SlideShare a Scribd company logo
1 of 69
ĐẠI HỌC QUỐC GIA HÀ NỘI
TRƯỜNG ĐẠI HỌC KHOA HỌC TỰ NHIÊN
NGUYỄN THỊ DIỆP
PHƯƠNG PHÁP ĐẠO HÀM VÀ CÁC BÀI TOÁN VỀ TÌM
GIÁ TRỊ LỚN NHẤT VÀ NHỎ NHẤT
Chuyên ngành : Phương pháp toán sơ cấp
Mã số: 60 46 01 13
LUẬN VĂN THẠC SỸ KHOA HỌC
Người hướng dẫn khoa học: PGS.TS. NGUYỄN MINH TUẤN
Hà Nội- 2015
Lời cám ơn
Trước khi trình bày nội dung chính của luận văn, tôi xin bày tỏ lòng cảm ơn chân
thành tới PGS.TS. Nguyễn Minh Tuấn, người thầy đã trực tiếp hướng dẫn, chỉ bảo
tận tình và giúp đỡ tôi trong suốt quá trình hoàn thành luận văn này.
Tôi cũng xin chân thành cảm ơn sự giúp đỡ của các thầy giáo, cô giáo trong khoa
Toán Cơ Tin học, Trương Đại học Khoa học Tự Nhiên-Đại học Quốc gia Hà Nội và
Khoa sau đại học, đã nhiệt tình giúp đỡ tôi hoàn thành khóa Cao học.
Tôi xin bày tỏ lòng biết ơn đến gia đình, bạn bè đã luôn động viên và khuyến khích
tôi rất nhiều trong thời gian nghiên cứu và học tập.
Do mới làm quen với công tác nghiên cứu khoa học nên luận văn còn nhiều thiếu
sót. Tác giả kính mong nhận được ý kiến đóng góp của các thầy cô và các bạn để luận
văn hoàn thiện hơn.
Hà Nội, năm 2015
Nguyễn Thị Diệp
2
Mục lục
Lời mở đầu 4
1 Một số kiến thức chuẩn bị 6
1.1 Định nghĩa đạo hàm tại một điểm . . . . . . . . . . . . . . . . . . . . . 6
1.2 Cực trị của hàm số . . . . . . . . . . . . . . . . . . . . . . . . . . . . . 7
1.3 Các định lí cơ bản về hàm khả vi . . . . . . . . . . . . . . . . . . . . . 8
1.4 Hàm lồi và hàm lõm . . . . . . . . . . . . . . . . . . . . . . . . . . . . 9
2 Ứng dụng đạo hàm giải các bài toán tìm giá trị lớn nhất, giá trị nhỏ
nhất của hàm số 11
2.1 Khảo sát trực tiếp hàm số trên miền xác định . . . . . . . . . . . . . . 11
2.2 Khảo sát hàm số theo từng biến . . . . . . . . . . . . . . . . . . . . . . 17
2.3 Đặt biến phụ chuyển về đánh giá hàm số một biến . . . . . . . . . . . . 30
2.4 Đánh giá gián tiếp thông qua biểu thức bậc nhất . . . . . . . . . . . . 44
2.5 Phương pháp sử dụng tính chất của hàm lồi, hàm lõm . . . . . . . . . 51
3 Cực trị hàm nhiều biến 59
3.1 Cực trị tự do . . . . . . . . . . . . . . . . . . . . . . . . . . . . . . . . 59
3.2 Cực trị có điều kiện . . . . . . . . . . . . . . . . . . . . . . . . . . . . . 63
3
Lời mở đầu
Trong những năm gần đây, các kỳ khảo sát chất lượng, thi học sinh giỏi bậc trung
học phổ thông thường gặp những bài toán yêu cầu tìm giá trị nhỏ nhất, giá trị lớn
nhất của một đại lượng nào đó. Các bài toán cực trị rất phong phú và đa dạng mang
nội dung vô cùng sâu sắc, có ý nghĩa rất quan trọng đối với các em học sinh. Các bài
toán về cực trị góp phần không nhỏ vào việc rèn luyện tư duy cho học sinh. Bài toán
đi tìm cái tốt nhất, rẻ nhất, ngắn nhất, dài nhất... trong một bài toán. Để dần dần
hình thành cho học sinh thói quen đi tìm giải pháp tối ưu cho một công việc nào đó
trong cuộc sống sau này.
Luận văn trình bày một số ứng dụng của đạo hàm để giải các bài toán cực trị. Luận
văn chỉ đề cập tới một số phương pháp giải một số loại toán cực trị đại số thường gặp
trong chương trình toán học trung học phổ thông. Luận văn hệ thống hóa, phân loại
toán và trình bày theo từng ý tưởng cũng như các kỹ năng vận dụng đạo hàm vào việc
giải một lớp các bài toán tìm giá trị lớn nhất và giá trị nhỏ nhất. Luận văn gồm có 3
chương với các nội dung sau:
Chương 1: Luận văn trình bày các kiến thức khái niệm cần thiết như đạo hàm, tính
đơn điệu và hàm lồi và được tham khảo trong [3].
Chương 2: Luận văn trình bày phương pháp sử dụng đạo hàm vào giải các bài toán
tìm giá trị lớn nhất, giá trị nhỏ nhất. Chương 2 luận văn trình bày phương pháp khảo
sát trực tiếp hàm số trên tập xác định của hàm số, khảo sát theo hàm số từng biến,
đặt biến phụ chuyển về đánh giá hàm một biến, đánh giá thông qua biểu thức bậc
nhất, hay phương pháp sử dụng tính chất hàm lồi, hàm lõm... được tham khảo trong
[1, 5, 6, 2, 7, 4].
4
5
Chương 3. Luận văn trình bày phương pháp để tìm cực trị tự do và cực trị có điều
kiện của hàm nhiều biến số. Từ đó tìm giá trị lớn nhất, giá trị nhỏ nhất của hàm số
và được tham khảo trong [3].
Chương 1
Một số kiến thức chuẩn bị
1.1 Định nghĩa đạo hàm tại một điểm
Định nghĩa 1.1. Cho hàm số y = f(x) xác định trên khoảng (a, b) và x0 ∈ (a, b). Nếu
giới hạn sau tồn tại và hữu hạn
lim
x→x0
f(x) − f(x0)
x − x0
thì giới hạn này được gọi là đạo hàm của hàm số f tại điểm x0 và được ký hiệu là
f (x0). Khi đó ta nói rằng f khả vi tại x0.
Chú ý. Nếu kí hiệu ∆x = x − x0, ∆y = f(x0 + ∆x) − f(x0) thì
f (x0) = lim
x→x0
f(x0 + ∆x) − f(x0)
x − x0
= lim
∆x→0
∆y
∆x
.
Nếu hàm số y = f(x) có đạo hàm tại x0 thì nó liên tục tại điểm đó.
Ý nghĩa hình học. Cho hàm số y = f(x) có đồ thị (C). Khi đó, f (x0) là hệ số góc
của tiếp tuyến đồ thị (C) của hàm số y = f(x) tại M(x0, y0) ∈ (C). Phương trình tiếp
tuyến của đồ thị hàm số y = f(x) tại điểm M(x0, y0) ∈ (C) là
y = f (x0)(x − x0) + y0.
6
7
1.2 Cực trị của hàm số
Định nghĩa 1.2. Cho hàm số y = f(x) xác định trên tập hợp D ⊂ R và x0 ∈ D. Điểm
x0 được gọi là một điểm cực đại của hàm số f(x) nếu tồn tại một khoảng (a, b) chứa
điểm x0 sao cho f(x) ≤ f(x0) với ∀x ∈ (a, b) ∩ D. Khi đó f(x0) được gọi là giá trị cực
đại của f(x) và điểm (x0, f(x0)) được gọi là điểm cực đại của đồ thị hàm số y = f(x).
Điểm x0 được gọi là một điểm cực tiểu của hàm số f(x) nếu tồn tại một khoảng (a, b)
chứa điểm x0 sao cho f(x) ≥ f(x0) với ∀x ∈ (a, b) ∩ D.
Khi đó f(x0) được gọi là giá trị cực tiểu của f(x) và điểm (x0, f(x0)) được gọi là điểm
cực tiểu của đồ thị hàm số y = f(x).
Điểm cực đại, cực tiểu được gọi chung là điểm cực trị. Giá trị cực đại, giá trị cực tiểu
được gọi chung là cực trị.
Định lý 1.3. Cho hàm số y = f(x) xác định và liên tục trên [a, b].
Nếu f (x) ≥ 0 ∀x ∈ [a, b] thì f(x) đồng biến trên [a, b] và khi đó ta có
min
x∈[a,b]
f(x) = f(a), max
x∈[a,b]
f(x) = f(b).
Nếu f (x) ≤ 0 ∀x ∈ [a, b] thì f(x) nghịch biến trên [a, b] và khi đó ta có
min
x∈[a,b]
f(x) = f(b), max
x∈[a,b]
f(x) = f(a).
Chú ý. Khái niệm cực đại cực tiểu của một hàm số có tính chất địa phương, chúng
chưa chắc đã là giá trị lớn nhất, giá trị nhỏ nhất của hàm số.
Ta có kết quả sau về điều kiện cần của cực trị.
Định lý 1.4. ( Định lý Fermat) Cho hàm f xác định trên (a, b) và x0 ∈ (a, b). Nếu
hàm số f có cực trị tại x0 và hàm f có đạo hàm tại x0 thì
f (x0) = 0.
Chú ý. Điều ngược lại không đúng: Nếu hàm f có f (x0) = 0 nhưng chưa chắc x0 là
điểm cực trị, ví dụ hàm y = x3
có y (0) = 0 nhưng hàm số không có cực trị tại x = 0.
8
Nếu hàm số f có cực trị tại x0 thì có thể tại x0 đạo hàm không xác định, ví dụ hàm
y = |x| có cực tiểu tại x = 0 nhưng dễ chứng minh được hàm số không có đạo hàm tại
x = 0.
Định lý 1.5. Giả sử hàm số f khả vi trên khoảng (a, b) chứa x0, và f (x0) = 0.
Nếu f (x) ≥ 0 với mọi x ∈ (a, x0) và f (x) ≤ 0 với mọi x ∈ (x0, b) ( tức đạo hàm đổi
dấu từ (+) sang (-) khi đi qua x0) thì x0 là điểm cực tiểu của hàm f.
Nếu f (x) ≤ 0 với mọi x ∈ (a, x0) và f (x) ≥ 0 với mọi x ∈ (x0, b) ( tức đạo hàm đổi
dấu từ (-) sang (+) khi đi qua x0) thì x0 là điểm cực đại của hàm f.
Định lý 1.6. Giả sử hàm số f có đạo hàm cấp một trên khoảng (a, b) chứa x0, có đạo
hàm cấp hai khác 0 tại x0.
Nếu f (x0) = 0 và f (x0) > 0 thì x0 là điểm cực tiểu của hàm f.
Nếu f (x0) = 0 và f (x0) < 0 thì x0 là điểm cực đại của hàm f.
1.3 Các định lí cơ bản về hàm khả vi
Trong phần này, luận văn trình bày hai định lý quan trọng về đạo hàm. Đó là định
lí Lagrange, định lí Rolle (xem [3]).
Định lý 1.7. (Định lý Rolle) Nếu f(x) là hàm liên tục trên đoạn [a, b], có đạo hàm
trên khoảng (a, b) và f(a) = f(b) thì tồn tại c ∈ (a, b) sao cho f (c) = 0.
Chứng minh. Vì f(x) liên tục trên [a, b] nên theo định lí Weierstrass f(x) nhận giá trị
lớn nhất M và giá trị nhỏ nhất m trên [a, b].
- Khi M = m ta có f(x) là hàm hằng trên [a, b], do đó với mọi c ∈ (a, b) luôn có
f (c) = 0.
- Khi M > m, vì f(a) = f(b) nên tồn tại c ∈ (a, b) sao cho f(c) = m hoặc f(c) = M,
theo Định lý Fermat suy ra f (c) = 0. Định lý được chứng minh.
Hệ quả 1.8. Nếu hàm số f(x) có đạo hàm trên (a, b) và f(x) có n nghiệm ( n là số
nguyên dương lớn hơn 1) trên (a, b) thì f (x) có ít nhất n − 1 nghiệm trên (a, b).
9
Hệ quả 1.9. Nếu hàm số f(x) có đạo hàm trên (a, b) và f (x) vô nghiệm trên (a, b)
thì f(x) có nhiều nhất 1 nghiệm trên (a, b).
Hệ quả 1.10. Nếu f(x) có đạo hàm trên (a, b) và f (x) có nhiều nhất n nghiệm (nlà
số nguyên dương) trên (a, b) thì f(x) có nhiều nhất n + 1 nghiệm trên (a, b).
Định lý 1.11. (Định lí Lagrange) Nếu f(x) là hàm liên tục trên đoạn [a, b], có đạo
hàm trên khoảng (a, b) thì tồn tại c ∈ (a, b) sao cho
f (c) =
f(b) − f(a)
b − a
.
Chứng minh. Xét hàm số
F(x) = f(x) −
f(b) − f(a)
b − a
x, x ∈ [a, b].
Khi đó F(x) là hàm liên tục trên đoạn[a, b], có đạo hàm trên khoảng (a, b) và
F(a) = F(b). Theo định lí Rolle tồn tại c ∈ (a, b) sao cho F (c) = 0. Mà
F (x) = f (x) −
f(b) − f(a)
b − a
,
suy ra
f (c) =
f(b) − f(a)
b − a
.
Định lý được chứng minh.
Định lí Rolle là một hệ quả của định lý Lagrange trong trường hợp f(a) = f(b).
1.4 Hàm lồi và hàm lõm
Ta ký hiệu I(a, b) là một tập hợp có một trong bốn dạng tập hợp sau (a, b), [a, b), (a, b]
và [a, b].
Định nghĩa 1.12. Hàm số f(x) được gọi là lồi trên tập I(a, b) nếu với mọi x1, x2 ∈
I(a, b) và với mọi cặp số dương α, β có tổng α + β = 1, ta đều có
f(αx1 + βx2) ≤ αf(x1) + βf(x2). (1.1)
Nếu dấu đẳng thức trong (1.1) xảy ra khi và chỉ khi x1 = x2 thì ta nói hàm số f(x) là
hàm lồi thực sự (chặt) trên I(a, b).
10
Định nghĩa 1.13. Hàm số f(x) được gọi là lõm trên tập I(a, b) nếu với mọi
x1, x2 ∈ I(a, b) và với mọi cặp số dương α, β có tổng α + β = 1, ta đều có
f(αx1 + βx2) ≥ αf(x1) + βf(x2). (1.2)
Nếu dấu đẳng thức trong (1.2) xảy ra khi và chỉ khi x1 = x2 thì ta nói hàm số f(x) là
hàm lõm thực sự (chặt) trên I(a, b).
Định lý 1.14. Nếu f(x) khả vi bậc hai trên I(a, b) thì f(x) lồi (lõm) trên I(a, b) khi
và chỉ khi f (x) ≥ 0(f (x) ≤ 0) trên I(a, b).
Nếu f(x) lồi khả vi trên I(a, b) thì với mọi cặp x0, x ∈ I(a, b), ta đều có
f(x) ≥ f(x0) + f (x0)(x − x0) (1.3)
Dễ nhận thấy rằng (1.3) xảy ra đẳng thức khi x0 = x. Vậy ta có thể viết (1.3) dưới
dạng
f(x) = min
u∈I(a,b)
f(u) + f (u)(x − u).
Chương 2
Ứng dụng đạo hàm giải các bài
toán tìm giá trị lớn nhất, giá trị
nhỏ nhất của hàm số
2.1 Khảo sát trực tiếp hàm số trên miền xác định
Bài toán 1. ( Thi HSG Quốc gia, 1992) Cho số tự nhiên n > 1. Tìm giá trị lớn nhất
và nhỏ nhất của hàm số
f(x) = n
√
1 + x + n
√
1 − x
với x thuộc [0, 1].
Chứng minh. Hàm số f(x) = n
√
1 + x + n
√
1 − x liên tục trên miền [0, 1], hơn nữa
f (x) =
1
n
1
n
(1 + x)n−1
−
1
n
(1 − x)n−1
< 0 ∀x ∈ (0, 1).
Vậy f(x) nghịch biến [0, 1] nên
n
√
2 = f(1) ≤ f(x) ≤ f(0) = 2 ∀x ∈ [0, 1].
Vậy giá trị lớn nhất của f(x) = n
√
1 + x + n
√
1 − x trên [0, 1] là 2 đạt được khi x = 0,
giá trị nhỏ nhất của f(x) trên [0, 1] là n
√
2 đạt được khi x = 1.
11
12
Bài toán 2. a. Tìm giá trị lớn nhất của biểu thức
x + 1
√
x2 − x + 1
với x ∈ R.
b. Tìm giá trị nhỏ nhất của biểu thức
√
x2 − x + 1 + y2 − y + 1 +
√
z2 − z + 1
trong đó các số x, y, z thỏa mãn x + y + z = 3.
Chứng minh. a) Xét hàm số
f(x) =
x + 1
√
x2 − x + 1
với x ∈ R.
Ta có
f (x) =
3(1 − x)
2(x2 − x + 1)
√
x2 − x + 1
và do đó f (x) = 0 khi và chỉ khi x = 1, limx→+∞ f(x) = 1, limx→−∞ f(x) = −1, và
f (x) đổi dấu từ + sang − khi đi qua 1. Do đó
f(x) ≤ f(1) = 2 ∀x ∈ R.
Vậy giá trị lớn nhất của f(x) trên R là 2 đạt được khi x = 1.
b) Áp dụng câu a) ta có
x + 1
√
x2 − x + 1
≤ 2 ∀x ∈ R.
Hay
√
x2 − x + 1 ≥
1
2
(x + 1). (2.1)
Tương tự ta có
y2 − y + 1 ≥
1
2
(y + 1), (2.2)
√
z2 − z + 1 ≥
1
2
(z + 1). (2.3)
Cộng từng vế các bất đẳng thức (2.1), (2.2) và (2.3) ta có
√
x2 − x + 1 + y2 − y + 1 +
√
z2 − z + 1 ≥
1
2
(3 + x + y + z).
13
Mà x + y + z = 3 nên ta có
√
x2 − x + 1 + y2 − y + 1 +
√
z2 − z + 1 ≥ 3.
Dấu bằng xảy ra khi x = y = z = 1. Vậy giá trị nhỏ nhất của
√
x2 − x + 1 +
y2 − y + 1 +
√
z2 − z + 1 là 3.
Bài toán 3. Giả sử A, B, C là ba góc của một tam giác nhọn. Tìm giá trị nhỏ nhất
của biểu thức
tan A + tan B + tan C + 6(sin A + sin B + sin C).
Chứng minh. Xét hàm số
f(x) = tan x + 6 sin x − 7x với x ∈ (0, π/2).
Ta có
f (x) =
1
cos2 x
+ 6 cos x − 7 =
(cos x − 1)(3 cos x + 1)(2 cos x − 1)
cos2 x
.
Vì x ∈ (0, π/2) nên f (x) = 0 khi 2 cos x − 1 = 0 hay x = π/3. Ta thấy f (x) đổi dấu
từ − sang + khi đi qua π/3. Nên ta được
min
x∈(0,π/2)
f(x) = f(π/3) = 4
√
3 −
7π
3
.
Áp dụng điều này lần lượt cho x = A, x = B, x = C ta được
f(A) + f(B) + f(C) ≥ 3(4
√
3 −
7π
3
).
Hay
tan A + tan B + tan C + 6(sin A + sin B + sin C) ≥ 12
√
3.
Vậy giá trị nhỏ nhất của tan A + tan B + tan C + 6(sin A + sin B + sin C) là 12
√
3 đạt
được khi A = B = C = π/3.
Bài toán 4. Giả sử x > 0, y > 0 và x + y = 1. Chứng minh rằng giá trị lớn nhất của
biểu thức
x
√
1 − x
+
y
√
1 − y
là
√
2.
14
Chứng minh. Theo giả thiết thì y = 1 − x. Ta sẽ chứng minh rằng
x
√
1 − x
+
1 − x
√
x
≥
√
2 ∀x ∈ (0, 1).
Xét
f(x) =
x
√
1 − x
+
1 − x
√
x
với x ∈ (0, 1).
Ta có
f (x) =
1
2
2 − x
(1 − x)
√
1 − x
−
x + 1
x
√
x
=
1
2
1 + (1 − x)
(1 − x)
√
1 − x
−
1 + x
x
√
x
=
1
2
(h(
√
1 − x) − h(
√
x))
trong đó
h(t) =
1 + t2
t3
=
1
t3
+
1
t
.
Ta thấy hàm h(t) là hàm nghịch biến trên (0, +∞), nên
f (x) = 0 ⇔ h(
√
1 − x) = h(
√
x) ⇔
√
1 − x =
√
x ⇔ x =
1
2
và
f (x) > 0 ⇔ h(
√
1 − x) > h(
√
x) ⇔
√
1 − x <
√
x ⇔ x ∈ (
1
2
, 1)
f (x) < 0 ⇔ x ∈ (0,
1
2
).
Vậy trên (0, 1) thì f (x) đổi dấu từ − sang + khi đi qua 1
2
. Nên ta được
min
x∈(0,1)
f(x) = f(
1
2
) =
√
2.
Suy ra f(x) ≥
√
2 ∀x ∈ (0, 1). Vậy giá trị nhỏ nhất của
x
√
1 − x
+
y
√
1 − y
là
√
2 đạt được khi x = y = 1
2
.
Bài toán 5. Tìm giá trị nhỏ nhất của hàm số
f(x) =
√
x2 + x + 1 +
√
x2 − x + 1.
15
Chứng minh. Tập xác định R. Xét hàm số
f(x) =
√
x2 + x + 1 +
√
x2 − x + 1
trên R. Ta có
f (x) =
x + 1
2
(x + 1
2
)2 + 3
4
+
x − 1
2
(x − 1
2
)2 + 3
4
= g(x +
1
2
) − g(
1
2
− x) ∀x ∈ R
trong đó
g(t) =
t
t2 + 3
4
với t ∈ R.
Vì hàm g đồng biến trên R nên
f (x) = 0 ⇔ x = 0,
f (x) > 0 ⇔ g(x +
1
2
) > g(
1
2
− x) ⇔ x > 0.
Vậy trên R thì f (x) đổi dấu từ − sang + khi đi qua 0. Từ đó giá trị nhỏ nhất của
f(x) là 2 khi x = 0.
Bài toán 6. Giả sử a, b ∈ R+ và a = b. Tìm giá trị nhỏ nhất của biểu thức
(
a + x
b + x
)b+x
với x ∈ [0, +∞).
Chứng minh. Xét hàm số
f(x) = (
a + x
b + x
)b+x
, x ≥ 0.
Khi đó
ln f(x) = (b + x) ln
a + x
b + x
.
Suy ra
[ln f(x)] = [(b + x) ln
a + x
b + x
] ,
và do đó
f (x)
f(x)
= ln
a + x
b + x
+ (b + x)
b + x
a + x
(
a + x
b + x
) = ln
a + x
b + x
+
b − a
a + x
.
16
Hay
f (x) = f(x) ln
a + x
b + x
+
b − a
a + x
= (
a + x
b + x
)b+x
g(x)
trong đó
g(x) = ln
a + x
b + x
+
b − a
a + x
.
Ta có
g (x) =
b + x
a + x
.
b − a
(b + x)2
+
a − b
(a + x)2
= −
(a − b)2
(a + x)2(b + x)
< 0.
Do đó g(x) nghịch biến trên (0, +∞). Suy ra
g(x) > lim
x→∞
g(x) = lim
x→∞
ln
a + x
b + x
+
b − a
a + x
= 0.
Vậy
f (x) > 0 ∀x > 0,
nên f(x) đồng biến trên [0, +∞). Suy ra f(x) ≥ f(0) ∀x ≥ 0. Vậy giá trị nhỏ nhất
của biểu thức (a+x
b+x
)b+x
với x ∈ [0, +∞) là (a
b
)b
.
Bài toán 7. Tìm giá trị nhỏ nhất của biểu thức
2sin x
+ 2tan x
− 2x+1
0 ≤ x ≤
π
2
.
Chứng minh. Áp dụng bất đẳng thức AM-GM ta có
2sin x
+ 2tan x
≥ 2
√
2sin x2tan x.
Ta chứng minh
sin x + tan x ≥ 2x.
Thật vậy, xét hàm số
f(x) = sin x + tan x − 2x
liên tục trên [0, π
2
], và có
f (x) = cos x +
1
cos2 x
− 2 > cos2
x +
1
cos2 x
− 2 ≥ 0 ∀x ∈ [0,
π
2
].
Do đó f(x) đồng biến trên [0, π
2
]. Suy ra f(x) ≥ f(0) = 0, hay
sin x + tan x ≥ 2x với mọi x ∈ [0,
π
2
]
17
Vậy giá trị nhỏ nhất của biểu thức
2sin x
+ 2tan x
− 2x+1
với 0 ≤ x ≤
π
2
là 0 đạt được khi x = 0.
2.2 Khảo sát hàm số theo từng biến
Đối với các BĐT nhiều biến, ta có thể chọn một biến là biến số biến thiên và cố
định các biến còn lại, bài toán lúc này trở thành BĐT một biến.
Bài toán 8. Giả sử A, B, C là ba góc của một tam giác. Tìm giá trị nhỏ nhất của biểu
thức
Q = 2
1
sin A
+
1
sin B
+
1
sin C
− (cot A + cot B + cot C).
Chứng minh. Ta viết
Q =
2
sin A
− cot A +
2
sin B
− cot B +
2
sin C
− cot C .
Xét hàm số
f(x) =
2
sin x
− cot x với x ∈ (0, π).
Ta có
f (x) =
1 − 2 cos x
sin2
x
.
Khi đó, hàm f (x) trong khoảng (0, π) sẽ chuyển từ − sang + khi đi qua điểm π/3. Từ
đó ta có
f(A) ≥
√
3
f(B) ≥
√
3
f(C) ≥
√
3.
Suy ra
Q = f(A) + f(B) + f(C) ≥ 3
√
3.
18
Dấu đẳng thức khi tam giác ABC đều. Vậy giá trị nhỏ nhất của biểu thức
Q = 2
1
sin A
+
1
sin B
+
1
sin C
− (cot A + cot B + cot C)
là 3
√
3.
Bài toán 9. Giả sử các số thực a, b, c > 0, thỏa mãn điều kiện a2
+ b2
+ c2
= 1. Tìm
giá trị nhỏ nhất của biểu thức
a
b2 + c2
+
b
c2 + a2
+
c
a2 + b2
.
Chứng minh. Ta sẽ chứng minh rằng
a
b2 + c2
+
b
c2 + a2
+
c
a2 + b2
≥
3
√
3
2
. (2.4)
Ta có (2.4) tương đương
a
1 − a2
+
b
1 − b2
+
c
1 − c2
≥
3
√
3
2
(a2
+ b2
+ c2
).
Từ đó gợi ý ta chứng minh các bất đẳng thức sau
a
1 − a2
≥
3
√
3
2
a2
,
b
1 − b2
≥
3
√
3
2
b2
,
c
1 − c2
≥
3
√
3
2
c2
.
Hay phải chứng minh
a(1 − a2
) ≤
2
3
√
3
, b(1 − b2
) ≤
2
3
√
3
, c(1 − c2
) ≤
2
3
√
3
.
Khảo sát đại diện là hàm số
f(x) = x(1 − x2
) với x ∈ (0, 1).
Ta có
f (x) = 1 − 3x2
.
Khi đó, hàm f (x) trong khoảng (0, 1) sẽ chuyển từ − sang + khi đi qua điểm 1/
√
3.
Nên giá trị lớn nhất của f(x) trên khoảng (0, 1) là 2
3
√
3
. Vậy giá trị nhỏ nhất của biểu
thức a
b2+c2 + b
c2+a2 + c
a2+b2 là 3
√
3
2
đạt được khi a = b = c = 1/
√
3.
19
Bài toán 10. Chứng minh rằng giá trị lớn nhất của biểu thức
2(x3
+ y3
+ z3
) − (x2
y + y2
z + z2
x) với x, y, z ∈ [0, 1]
là 3.
Chứng minh. Ta sẽ chứng minh rằng
f(x) = 2x3
− yx2
− z2
x + 2(y3
+ z3
) ≤ 3.
Thật vậy, ta có
f (x) = 6x2
− 2yz − z2
và f (x) = 0 khi
x = x1 =
1
6
(y − y2 + 6z2), x = x2 =
1
6
(y + y2 + 6z2).
Xét hai trường hợp
Trường hợp 1. Nếu x2 ∈ (0, 1) suy ra f (x) ≤ 0 ∀x ∈ [0, 1]. Suy ra f(x) giảm trên
[0, 1]. Do đó
max
x∈[0,1]
f(x) = max{f(0), f(1)}.
Trường hợp 2. Nếu x2 ∈ (0, 1) thì do x1 ≤ 0 < x2 nên trên [0, 1] hàm f (x) sẽ đổi dấu
từ − sang + khi đi qua x2. Do đó
max
x∈[0,1]
f(x) = max{f(0), f(1)}.
Như vậy trong cả hai trường hợp ta đều có
max
x∈[0,1]
f(x) = max{f(0), f(1)}.
Mặt khác
f(0) = 2(y3
+ z3
) − y2
z ≤ 2(y3
+ z3
) − y2
z + (2 − y − z2
) = f(1).
Nên ta được
max
x∈[0,1]
f(x) = f(1) = 2(y3
+ z3
) − y2
z + (2 − y − z2
).
20
Ta sẽ chứng minh f(1) ≤ 3. Thật vậy, đặt
f(1) = g(y) = 2(y3
+ z3
) − y2
z + (2 − y − z2
).
Khi đó
g (y) = 6y2
− 2zy − 1 = 0
và
g (y) = 0 ⇔ y = y1 =
1
6
(z −
√
z2 + 6) < 0, y = y2 =
1
6
(z +
√
z2 + 6).
• Nếu y2 ∈ (0, 1) thì g (y) ≤ 0 ∀y ∈ [0, 1]. Suy ra g(y) giảm trên [0, 1]. Do đó
max
y∈[0,1]
g(y) = max{g(0), g(1)}.
Nếu y2 ∈ (0, 1) thì trên [0, 1] hàm g (y) sẽ đổi dấu từ − sang + khi đi qua y2. Do vậy
max
y∈[0,1]
g(y) = max{g(0), g(1)}.
Như vậy trong cả hai trường hợp ta đều có
max
y∈[0,1]
g(y) = max{g(0), g(1)}.
Ta có
g(0) = 2z3
+ 2 − z2
≤ 2z3
+ 2 − z2
+ (1 − z) = g(1) = z(z − 1)(2z + 1) + 3 ≤ 3
với mọi x, y, z ∈ [0, 1]. Do đó
max
y∈[0,1]
g(y) = 3.
Khi x = y = z = 1 thì 2(x3
+ y3
+ z3
) − (x2
y + y2
z + z2
x) = 3. Vậy giá trị lớn nhất
của 2(x3
+ y3
+ z3
) − (x2
y + y2
z + z2
x) là 3.
Bài toán 11. Giả sử a, b, c ∈ [1
3
, 3]. Tìm giá trị lớn nhất của biểu thức
S(a, b, c) =
a
a + b
+
b
b + c
+
c
c + a
.
Chứng minh. Đặt
f(a) =
a
a + b
+
b
b + c
+
c
c + a
.
21
Xét hai trường hợp sau
Trường hợp 1. a ≥ b ≥ c. Ta có
f (a) =
b
(a + b)2
−
c
(a + c)2
=
(b − c)(a2
− bc)
(a + b)2(a + c)2
≥ 0.
Suy ra
f(a) ≤ f(3) =
3
3 + b
+
b
b + c
+
c
c + 3
= g(c).
Mặt khác
g (a) =
−b
(c + b)2
+
3
(c + 3)2
=
(b − 3)(3b − c2
)
(c + 3)2(b + c)2
≥ 0.
Suy ra
g(c) ≤ g(
1
3
) =
3
3 + b
+
3b
3b + 1
+
1
10
= h(b).
Ta có
h (b) =
3
(3b + 1)2
−
3
(b + 3)2
=
(1 − b)(1 + b)
(3b + 1)2(b + 3)2
.
Nên h(b) ≤ h(1). Suy ra
S(a, b, c) ≤ S(3, 1,
1
3
) =
8
5
.
Trường hợp 2. c ≥ b ≥ a. Áp dụng trường hợp 1 ta nhận được
S(c, b, a) ≤
8
5
.
Mặt khác
S(a, b, c) − S(c, b, a) =
(a − b)(b − c)(a − c)
(a + b)(b + c)(a + c)
≤ 0.
Suy ra
S(a, b, c) ≤
8
5
.
Vậy giá trị lớn nhất của S(a, b, c) là 8
5
, đạt được khi và chỉ khi (a, b, c) ∈
{(3, 1, 1
3
), (1
3
, 3, 1), (1, 1
3
, 3)}.
Bài toán 12. Giả sử a, b, c ∈ [0, 1]. Tìm giá trị lớn nhất của biểu thức
S =
a
b3 + c3 + 6
+
b
c3 + a3 + 6
+
c
a3 + b3 + 6
.
22
Chứng minh. Đặt
f(c) =
a
b3 + c3 + 6
+
b
c3 + a3 + 6
+
c
a3 + b3 + 6
.
Ta có
f (c) =
1
b3 + c3 + 6
−
3ac2
(b3 + c3 + 6)2
−
3c2
(a3 + c3 + 6)2
và
f (c) =
6ac(6 + b3
− 2c3
)
(b3 + c3 + 6)2
−
6bc(6 + a3
− 2c3
)
(a3 + c3 + 6)2
≤ 0.
Nên f (c) giảm trên [0, 1]. Suy ra
f (c) ≥ f (1) =
1
b3 + a3 + 6
−
3a
(7 + b3)2
−
3b
(7 + a3)2
≥
1
8
− 2
3
49
> 0.
Suy ra f(c) tăng trên [0, 1]. Do đó
S = f(c) ≤ f(1) =
a
b3 + 7
+
b
a3 + 7
+
1
a3 + b3 + 6
= g(a).
Ta có
g (a) =
1
b3 + 7
−
2a2
b
(a3 + 7)2
−
1
(a3 + b3 + 7)2
và
g (a) =
6ab(7 − 2a3
)
(a3 + 7)3
−
6a(b3
+ 6 − 2a3
)
(a3 + b3 + 7)3
≤ 0.
Nên g (a) giảm trên [0, 1]. Suy ra
g (a) ≥ g (1) =
1
b3 + 7
−
3b
64
−
3
(7 + b3)2
=
1
b3 + 7
−
1
8
5
8
−
3
b3 + 7
+
5 − 3b
64
> 0.
Suy ra g(a) tăng trên [0, 1]. Do đó
S = g(a) ≤ g(1) =
2
b3 + 7
+
b
8
= h(b).
Ta có
h (b) =
1
8
−
6b2
(b3 + 7)2
−
(b3
+ 7)2
− 48b2
8(b3 + 7)2
> 0 ∀b ∈ [0, 1].
Suy ra h(b) tăng trên [0, 1], nên
h(b) ≤ h(1) =
3
8
.
Hay S ≤ 3
8
. Với a = b = c = 1 thì S = 3
8
. Vậy giá trị lớn nhất của S là 3
8
.
23
Bài toán 13. Xét hàm số
f(x, y) = (1 − x)(2 − y)(4x − 2y)
trên miền D = {(x, y) : 0 ≤ x ≤ 1, 0 ≤ y ≤ 2}. Tìm giá trị nhỏ nhất của hàm f trên
miền D.
Chứng minh. Biến đổi hàm số đã cho thành
f(x, y) = 2(1 − x)(2 − y)((2 − y) − 2(1 − x))
Đặt u = 1 − x, v = 2 − y, thì bài toán đã cho trở thành tìm giá trị nhỏ nhất của hàm
số
F(u, v) = −2uv2
+ u2
v
trên miền E = {(u, v) : 0 ≤ u ≤ 2, 0 ≤ v ≤ 1}, nghĩa là
min
E
F(u, v) = min
0≤u≤2
[ min
0≤v≤1
(−2uv2
+ u2
v)].
Xét hàm số g(v) = −2uv2
+ u2
v với 0 ≤ v ≤ 1, coi u là tham số. Ta có
g (v) = −4uv + u2
= u(−4v + u).
Ta thấy g (v) = 0 khi v0 = u
4
, và qua v0 = u
4
thì g (v) đổi dấu từ dương sang âm, mà
0 ≤ u
4
≤ 1
2
, ta suy ra
max
0≤v≤1
g(v) = min{g(0), g(1)} = min{0, u2
− 2u}.
Do u2
− 2u ≤ 0 nên
min
E
F(u, v) = min
0≤u≤2
(u2
− 2u) = −1
khi u = 1, v = 1. Từ đó
min
D
f(x, y) = 2 min
E
F(u, v) = −2
khi x = 0, y = 1. Vậy giá trị nhỏ nhất của f(x, y) trên miền D là −2.
Bài toán 14. (Đề thi HSG THPT toàn quốc bảng A, 1999) Xét các số thực dương
a, b, c thỏa mãn abc + a + c = b. Tìm giá trị lớn nhất của biểu thức
P =
2
a2 + 1
−
2
b2 + 1
+
3
c2 + 1
.
24
Chứng minh. Biến đổi giả thiết thành a + c = b(1 − ac) > 0, suy ra
a <
1
c
, b =
a + c
1 − ac
. (2.5)
Thay (2.5) vào biểu thức P và biến đổi được
P =
2
a2 + 1
+
3
c2 + 1
+
2(a + c)2
(1 + a2)(1 + b2)
− 2. (2.6)
Xét hàm số
f(x) =
1
x2 + 1
+
2(x + c)2
(1 + x2)(1 + c2)
với 0 < x < 1
2
và coi c là tham số (c > 0). Ta có
f (x) =
2c(x2
+ 2cx − 1)
(1 + c2)(1 + x2)2
.
Trên (0, 1
c
) thì f (x) = 0 có nghiệm duy nhất là
x0 = −c +
√
c2 + 1 (2.7)
với 0 < x0 < 1
c
. Qua x0 thì f (x) đổi dấu từ dương sang âm nên f(x) đạt cực đại tại
x0 nên
f(x) ≤ f(x0) = 1 +
c
√
c2 + 1
.
Từ đó theo (2.6) ta có
P = 2f(x) − 2 +
3
c2 + 1
≤
2c
√
c2 + 1
+
3
c2 + 1
= g(c).
Xét hàm số g(c) với c > 0. Ta có
g (c) =
2(1 − 8c2
)
(c2 + 1)(3c +
√
c2 + 1)
.
Với c > 0, thì ta thấy g (c) = 0 tại c0 = 1√
8
và g (c) đổi dấu từ dương sang âm khi đi
qua c0 nên g(c0) là giá trị cực đại, suy ra
P ≤ g(
1
√
8
) =
10
3
.
Ta thấy, giá trị P = 10
3
đạt được khi c = 1√
8
, a = 1√
2
, b =
√
2. Vậy giá trị lớn nhất của
P là 10
3
.
25
Bài toán 15. (VMO, 2001) Xét các số thực dương x, y, z thỏa mãn hệ điều kiện



2
5
≤ z ≤ min{x, y}
xz ≥ 4
15
yz ≥ 1
5
.
(2.8)
Hãy tìm giá trị lớn nhất của biểu thức
P(x, y, z) =
1
x
+
2
y
+
3
z
.
Chứng minh. Từ hệ bất phương trình (2.8) suy ra
x ≥ max{z,
4
15z
} (2.9)
a) Xét hàm số
f(x) =
1
x
+
1
z
với x > 0 và tham số z ≥ 2
5
. Xét hai trường hợp
Trường hợp 1. Nếu z ≥ 2√
15
, khi đó theo (2.9) thì x ≥ z ≥ 4
15z
, nên
f(x) ≤
1
z
+
1
z
=
2
z
≤ 15. (2.10)
Trường hợp 2. Nếu 2
5
≤ z ≤ 2√
15
, khi đó theo (2.9) thì x ≥ 4
15z
≥ z, nên
f(x) ≤
15z
4
+
1
z
= g(z).
Xét hàm số g(z) với 2
5
≤ z ≤ 2√
15
. Ta có
g (z) =
15
4
−
1
z2
< 0
khi và chỉ khi z < 2√
15
. Do đó g(z) là hàm giảm và
f(z) ≤ g(z) ≤ g(
2
5
) = 4. (2.11)
So sánh (2.10) và (2.11) ta có
1
x
+
1
z
≤ 4
26
và
1
x
+
1
z
= 4 khi x =
2
3
, z =
2
5
.
Xét hàm số
h(y) =
1
y
+
1
z
với tham số z ≥ 2
5
. Từ điều kiện (2.8) suy ra
y ≥ max{z,
1
5z
}.
Lập luận tương tự phần a) ta được
• Nếu z ≥ 1√
5
thì
h(y) ≤ 2
√
5. (2.12)
• Nếu 2
5
≤ z ≤ 1√
5
thì
h(y) ≤
9
2
. (2.13)
So sánh (2.12) và (2.13) ta có
1
y
+
1
z
≤
9
2
và
1
y
+
1
z
=
9
2
khi x =
2
5
, y =
1
2
.
So sánh kết quả phần a) và b) ta có
P(x, y, z) = (
1
x
+
1
y
) + 2(
1
y
+
1
z
) ≤ 4 + 2
9
2
= 13.
Đẳng thức xảy ra khi và chỉ khi x = 2
3
, y = 1
2
, z = 2
5
. Vậy giá trị lớn nhất của P là
13.
Bài toán 16. (Đề thi chọn ĐTQG, 2001) Xét các số thực dương a, b, c thỏa mãn
21ab + 2bc + 8ac ≤ 12. Tìm giá trị nhỏ nhất của biểu thức
P(a, b, c) =
1
a
+
2
b
+
3
c
.
27
Chứng minh. Ta đổi biến
x =
1
a
, y =
1
b
, z =
1
c
thì giả thiết của bài toán đã cho trở thành 2x + 8y + 21z ≤ 12xyz. Ta cần tìm giá trị
nhỏ nhất của biểu thức
P(x, y, z) = x + 2y + 3z.
Từ giả thiết z(12xy − 21 ≥ 2x + 8y) > 0 suy ra
z ≥
2x + 8y
12xy − 21
với x >
7
4y
. (2.14)
Suy ra
P(x, y, z) ≥ x + 2y +
2x + 8y
4xy − 7
.
Xét hàm số
f(x) = x +
2x + 8y
4xy − 7
=
4x2
y − 5x + 8y
4xy − 7
với biến x > 7
4y
và y là tham số dương. Ta có
f (x) =
16x2
y2
− 56xy − 32y2
+ 35
(4xy − 7)2
.
Trên đoạn (7y
4
, +∞) thì f (x) = 0 có nghiệm duy nhất là
x0 = −
7y
4
+
32y2 + 14
4y
và qua x0 thì f (x) đổi dấu từ âm sang dương nên f(x) đạt cực tiểu tại x0 nên
f(x) ≥ f(x0) = 2x0 −
5
4y
.
Suy ra
P(x, y, z) ≥ f(x) + 2y ≥ f(x0) + 2y.
Xét hàm số
g(y) = 2y +
9
4y
+
1
2y
32y2 + 14.
Ta có
g (y) = 0 ⇔ (8y2
− 9) 32y2 + 14 − 28 = 0.
28
Đặt t = 32y2 + 14 với t > 0, thì phương trình trên trở thành
t3
− 50t − 112 = 0.
Hay
(t − 8)(t2
+ 8t + 14) = 0 ⇔ t = 8 hoặc t =
5
4
.
Từ đó g (5
4
) = 0 và g (y) đổi dấu từ âm sang dương khi đi qua điểm 5
4
nên g(y) đạt
cực tiểu tại y0 = 5
4
lúc đó ta có
P(x, y, z) ≥ g(y) ≥ g(
5
4
) =
15
2
.
Dấu đẳng thức xảy ra khi y = 5
4
, x = 3, z = 3
2
hay a = 1
3
, b = 4
3
, c = 2
3
. Vậy giá trị nhỏ
nhất của P là 15
2
.
Bài toán 17. Tìm giá trị nhỏ nhất của biểu thức
Q = −2xy2
+ x2
y
trên miền E = {(x, y) : 0 ≤ x ≤ 2, 0 ≤ y ≤ 1}.
Chứng minh. Coi x là tham số ta có hàm số
f(y) = −2xy2
+ x2
y với y ∈ [0, 1].
Ta có
f (y) = −4xy + x2
và
f (y) = 0 ⇔ y = x/4.
Ta suy ra
f(y) ≤ max{f(0), f(1)} ∀y ∈ [0, 1].
Hay
f(y) ≥ min{0, x2
− 2x} ∀y ∈ [0, 1].
Khi 0 ≤ x ≤ 2 thì x2
− 2x ≤ 0 suy ra
f(y) ≥ x2
− 2x = g(x).
29
Khảo sát hàm số g(x) = x2
− 2x x ∈ [0, 2] ta tìm được min g(x) = g(1) = −1. Kết
quả giá trị nhỏ nhất của Q là −1 đạt khi x = 1 , y = 1.
Bài toán 18. Xét các số thực dương x, y, z thỏa mãn điều kiện 12xyz ≥ 2x+8y +21z.
Tìm giá trị nhỏ nhất của biểu thức Q = x + 2y + 3z.
Chứng minh. Từ giả thiết z(12xy − 21) ≥ 2x + 8y > 0 suy ra
z ≥
2x + 8y
12xy − 21
, x >
7
4y
.
Do đó
Q ≥ x + 2y +
2x + 8y
4xy − 7
.
Xét hàm số
f(x) = x +
2x + 8y
4xy − 7
với x >
7
4y
.
Khi đó
f (x) = 1 −
32y2
+ 14
(4xy − 7)2
trên khoảng ( 7
4y
, +∞) thì
f (x) = 0 ⇔ x = x0 =
7
4y
+
32y2 + 14
4y
và f (x) đổi dấu từ âm qua dương khi x qua x0. Do vậy f(x) ≥ f(x0) = 2x0 − 5
4y
. Suy
ra
Q ≥ f(x) + 2y ≥ 2x0 −
5
4y
+ 2y.
Đặt
g(y) = 2x0 −
5
4y
+ 2y = 2y +
9
4y
+
32y2 + 14
2y
.
Khi đó
g (y) = 0 ⇔ (8y2
− 9) 32y2 + 14 = 28.
Đặt t = 32y2 + 14 thì t > 0 và ta có phương trình
t3
− 50t − 112 = 0.
Phương trình này chỉ có một nghiệm dương t = 8 từ đó y = y0 = 5
4
. Ta cũng có
g(y) ≥ g(y0) =
15
2
.
30
Vậy
Q ≥ g(y) ≥ g(y0) =
15
2
.
Đẳng thức xảy ra khi x = 3, y = 5/4, z = 2/3. Vậy giá trị nhỏ nhất của Q là 15
2
.
2.3 Đặt biến phụ chuyển về đánh giá hàm số một
biến
Bài toán 19. Giả sử x, y là hai số thực không âm thỏa mãn x + y = 4. Tìm giá trị
lớn nhất, giá trị nhỏ nhất của biểu thức
P = (x3
− 1)(y3
− 1).
Chứng minh. Ta có
P = (x3
− 1)(y3
− 1) = x3
y3
− (x3
+ y3
) + 1 = (xy)3
− [(x + y)3
− 3xy(x + y)] + 1.
Mà x + y = 4 nên
f = (xy)3
− [(x + y)3
− 3xy(x + y)] + 1 = (xy)3
+ 12xy − 63.
Đặt t = xy ta đưa về hàm theo một biến t là
f(t) = t3
+ 12t − 63.
Bây giờ ta cần tìm miền biến thiên của t như sau
0 ≤ t = xy ≤ (
x + y
2
)2
= 4.
Vậy t ∈ [0, 4]. Dùng đạo hàm ta tìm được giá trị lớn nhất và giá trị nhỏ nhất của hàm
f(t) = t3
+ 12t − 63, trên đoạn [0, 4] ta có
−63 ≤ f(t) ≤ 49 ∀t ∈ [0, 4].
Vậy giá trị lớn nhất, giá trị nhỏ nhất của biểu thức P = (x3
− 1)(y3
− 1) lần lượt là 49
khi t = 4, là −63 khi t = 0.
31
Bài toán 20. Giả sử x, y, z là hai số thực không âm thỏa mãn x2
+ y2
+ z2
= 3. Tìm
giá trị lớn nhất của biểu thức
P = xy + yz + zx +
5
x + y + z
.
Chứng minh. Đặt t = x + y + z. Khi đó t2
= x2
+ y2
+ z2
+ 2(xy + yz + zx). Kết hợp
với giả thiết x2
+ y2
+ z2
= 3 ta nhận được
xy + yz + zx =
t2
− 3
2
.
Khi đó
P =
t2
2
+
5
t
−
3
2
= f(t)
Do 0 ≤ xy + yz + zx ≤ x2
+ y2
+ z2
= 3 nên
√
3 ≤ t ≤ 3.
Ta thấy f (t) = t − 5
t2 > 0 ∀t ∈ [
√
3, 3]. Suy ra f(t) đồng biến trên [
√
3, 3]. Do đó
f(t) ≤ f(3) = 14
3
. Dấu đẳng thức xảy ra khi t = 3 hay x = y = z = 1. Vậy giá trị lớn
nhất của biểu thức P là 14
3
khi x = y = z = 1.
Bài toán 21. Giả sử x, y ≥ 0 là hai số thực thỏa mãn x+y = 1. Tìm giá trị lớn nhất,
giá trị nhỏ nhất của biểu thức
P = (x2
− 1)(y2
− 1) − x2 + y2 + 1.
Chứng minh. Đặt t = xy thì
0 ≤ t = xy ≤ (
x + y
2
)2
=
1
4
.
Do vậy t ∈ [0, 1
4
]. Khi đó
P = f(t) = t2
+ 2t −
√
2 − 2t.
Ta có f (t) > 0 nên hàm số đồng biến. Suy ra
max
t∈[0, 1
4
]
f(t) = f(
1
4
), min
t∈[0, 1
4
]
f(t) = f(0).
Vậy giá trị lớn nhất P là 9
16
−
√
3
2
đạt được khi x = y = 1
2
, giá trị nhỏ nhất của P là
−
√
2 đạt được khi (x, y) = (0, 1), (1, 0).
32
Bài toán 22. Giả sử hai số x, y khác 0 thay đổi thỏa mãn
(x + y)xy = x2
+ y2
− xy. (2.15)
Chứng minh rằng giá trị lớn nhất của
1
x3
+
1
y3
là 16.
Chứng minh. Ta đặt
x + y = u, xy = v.
Khi đó (2.15) trở thành uv = u2
− 3v. Suy ra
v =
u2
u + 3
( do u = 3).
Ta có
1
x3
+
1
y3
=
x3
+ y3
(xy)3
=
u3
− 3uv
u3
=
u(u2
− 3v)
v3
=
u2
v2
= (
u + 3
u
)2
.
Vì
u2
≥ 4v ⇒ u2
≥
4u2
u + 3
⇒
4
u + 3
≤ 1 ⇒
u − 1
u + 3
≥ 0.
Điều này dẫn đến
u ≥ 1 hoặc u ≤ −3.
Vì từ (2.15) suy ra u+3
u
> 0 nên ta chỉ cần chứng minh
u − 1
u + 3
≤ 4 với u ≥ 1 hoặc u ≤ −3.
Xét hàm số
f(u) =
u + 3
u
.
Khi đó
f (u) =
−3
u2
< 0
suy ra f(x) nghịch biến trên mỗi khoảng (−∞, −3) và [1, +∞) và do đó
f(u) ≤ f(1) = 4 ∀u ≥ 1.
33
Hơn nữa
0 < f(−3) < f(u) < 1 ∀u < −3.
Do đó
1
x3
+
1
y3
≤ 16.
Ta thấy khi x = y = 1
2
thì
1
x3
+
1
y3
= 16.
Vậy giá trị lớn nhất của
1
x3
+
1
y3
là 16.
Bài toán 23. Giả sử x, y là các số thực thay đổi thỏa mãn điều kiện x2
+ y2
+ xy = 1.
Tìm giá trị lớn nhất và giá trị nhỏ nhất của biểu thức
S = x2
y − xy2
.
Chứng minh. Ta có
S2
= (xy)2
(x2
+ y2
− 2xy) = (xy)2
(1 − 3xy).
Đặt t = xy. Khi đó, từ x2
+ y2
+ xy = 1, ta có
1 − 3xy = (x − y)2
≥ 0, (x + y)2
= 1 + xy.
Điều này dẫn đến
1
3
≥ t ≥ −1.
Ta có
S2
= f(t) = t2
(1 − 3t) với t ∈ [−1,
1
3
].
Khi đó
f (t) = 2t − 9t2
và do đó f (t) = 0 khi t = 0 hoặc t = 2
9
. Hơn nữa
f(−1) = 4, f(0) = f(
1
3
) = 0, f(
2
9
) =
4
243
.
34
Vậy
S2
≤ 4.
Hay −2 ≤ S ≤ 2. Ta thấy S = 2 khi x = −1, y = 1, và S = −2 khi x = 1, y = −1. Vậy
giá trị lớn nhất và giá trị nhỏ nhất của S lần lượt là 2 và −2.
Bài toán 24. Giả sử x, y ∈ R và x, y > 1. Tìm giá trị nhỏ nhất của
P =
(x3
+ y3
) − (x2
+ y2
)
(x − 1)(y − 1)
.
Chứng minh. Biến đổi ta có
P =
(x + y)3
− (x + y)2
− xy(3(x + y) − 2)
xy − (x + y) + 1
.
Đặt t = x + y. Vì x, y > 1 nên t > 2. Ta được
P =
t3
− t2
− xy(3t − 2)
xy − t + 1
.
Áp dụng bất đẳng thức xy ≤ (x+y)2
4
, ta có
xy ≤
t2
4
.
Mặt khác xy > 0 và 3t − 2 > 0 nên
P =
t3
− t2
− xy(3t − 2)
xy − t + 1
≥
t3
− t2
− t2
4
(3t − 2)
t2
4
− t + 1
=
t2
t − 2
.
Khảo sát hàm số
f(t) =
t2
t − 2
trên miền t ∈ (2, +∞)
ta được kết quả
min
t∈(2,+∞)
f(t) = 8
đạt được khi x = y = 2. Vậy giá trị nhỏ nhất của S là 8.
Bài toán 25. (Trích đề thi Đại học khối A năm 2006) Giả sử hai số thực x, y = 0
thay đổi thỏa mãn (x + y)xy = x2
+ y2
− xy. Tìm giá trị lớn nhất của biểu thức
P =
1
x3
+
1
y3
.
35
Chứng minh. Từ giả thiết ta có
1
x
+
1
y
=
1
x2
+
1
y2
−
1
x
1
y
.
Đặt
a =
1
x
, b =
1
y
.
Ta được
a + b = a2
+ b2
− ab ⇒ a + b = (a + b)2
− 3ab.
Đặt t = a + b thì
ab =
t2
− t
3
.
Khi đó
P = a3
+ b3
= (a + b)3
− 3ab(a + b) = t3
− (t2
− t)t = t2
.
Tìm miền của t
a + b = (a + b)2
− 3ab ⇒ (a + b)2
− (a + b) = 3ab ≤ 3
(a + b)2
4
.
Do vậy
t2
− t ≤
3t2
4
⇒ 0 ≤ t ≤ 4.
Suy ra P ≤ 16. Vậy giá trị lớn nhất của P là 16 đạt được khi x = y = 1
2
.
Bài toán 26. Giả sử các số thực dương x, y, z thỏa mãn



x + y + z = 3
xy + yz + zx = 1.
Tìm giá trị nhỏ nhất, giá trị lớn nhất của biểu thức
Q = x4
+ y4
+ z4
.
Chứng minh. Đặt s = xyz thì
Q = 47 + s.
Từ hệ (26) ta có 


y + z = 3 − x
yz = x2
− 3x + 1.
36
Vì (3 − x)2
≥ 4(x2
− 3x + 1) nên
3x2
− 6x − 5 ≤ 0
kết hợp x ≥ 0 ta được
x ∈ [0,
3 + 2
√
6
3
].
Khảo sát hàm số
s = g(x) = x3
− 3x2
+ x với x ∈ [0,
3 + 2
√
6
3
]
và kết hợp với s ≥ 0 ta được
0 ≤ s ≤
4
√
6 − 9
9
.
Vì Q = 47 + 12s nên
47 ≤ Q ≤
105 + 16
√
6
3
.
Ta thấy với x = 0, (y, z) = ((3 +
√
5)/2, (3 −
√
5)/2), ((3 −
√
5)/2, (3 +
√
5)/2) thì
Q = 47; với x = 3+2
√
6
3
, y = z = (3 −
√
6)/3 thì Q = 105+16
√
6
3
. Vậy giá trị nhỏ nhất của
Q là 47, giá trị lớn nhất là 105+16
√
6
3
.
Bài toán 27. Giả sử các số thực x, y, z > 0 thỏa mãn



xy + yz + zx = 8
xyz = 4.
Tìm giá trị nhỏ nhất, giá trị lớn nhất của biểu thức Q = x4
+ y4
+ z4
.
Chứng minh. Từ giả thiết ta có



y + z = 1
x
(8 − 4
x
)
yz = 4
x
.
Vì (y + z)2
≥ 4yz nên
1
x2
(8 −
4
x
)2
≥ 4
4
x
⇒
1
x
(8 −
4
x
)2
≥ 16 ( do x > 0).
Hay
x3
− 4x2
+ 4x − 1 ≤ 0 ⇒ (x − 1)(x2
− 3x + 1) ≤ 0.
37
Kết hợp với x > 0 ta được
x ∈ (0,
3 −
√
5
2
] ∪ [1,
3 +
√
5
2
].
Đặt
s = x + y + z
thì
s = g(x) = x +
8
x
−
4
x2
với x ∈ (0,
3 −
√
5
2
] ∪ [1,
3 +
√
5
2
].
Khảo sát hàm số g(x) với x ∈ (0, 3−
√
5
2
] ∪ [1, 3+
√
5
2
] và chú ý s > 0 ta được
5 ≤ s ≤
5
√
5 − 1
2
.
Mặt khác
Q = f(s) = s4
− 32s2
+ 16s + 128 với s ∈ [5,
5
√
5 − 1
2
].
Dùng đạo hàm lập bảng biến thiên hàm số f(s), ta được
g(5) ≤ f(s) ≤ f(
5
√
5 − 1
2
).
Vậy giá trị nhỏ nhất, giá trị lớn nhất của Q lần lượt là 33 và 271−75
√
5
2
.
Bài toán 28. Giả sử các số thực x, y, z > 0 thỏa mãn x + y + z ≤ 2. Tìm giá trị nhỏ
nhất của biểu thức
Q = 4x2 +
1
x2
+ 4y2 +
1
y2
+ 4z2 +
1
z2
.
Chứng minh. Trước hết ta chứng minh
Q ≥ 4(x + y + z)2 + (
1
x
+
1
y
+
1
z
)2.
Thật vậy, áp dụng bất đẳng thức
1
x
+
1
y
+
1
z
≥
9
x + y + z
ta suy ra
Q ≥ 4(x + y + z)2 +
81
(x + y + z)2
.
38
Đặt t = (x + y + z)2
thì t ∈ (0, 4]. Khảo sát hàm số
f(t) = 4t +
81
t
với t ∈ (0, 4].
ta được mint∈(0,4] f(t) = f(4) = 145
4
suy ra
Q ≥
√
145
2
.
Đẳng thức xảy ra khi x = y = z = 2
3
. Vậy giá trị nhỏ nhất của Q là
√
145
2
.
Bài toán 29. (Trích đề thi Đại học khối A năm 2003) Giả sử x, y, z > 0, x+y+z ≤ 1.
Chứng minh rằng giá trị nhỏ nhất của
x2 +
1
x2
+ y2 +
1
y2
+ z2 +
1
z2
là
√
82.
Chứng minh. Đặt
P = x2 +
1
x2
+ y2 +
1
y2
+ z2 +
1
z2
.
Ta có
P = x2 +
1
x2
+ y2 +
1
y2
+ z2 +
1
z2
≥ (x + y + z)2 + (
1
x
+
1
y
+
1
z
)2
≥ (3 3
√
xyz)2 + (
3
3
√
xyz
)2.
Đặt
t = 3
√
xyz
suy ra
P ≥ f(t) = 9t2 +
9
t2
Tìm miền của t
0 < t = 3
√
xyz ≤
x + y + z
3
≤
1
3
.
Vậy
t ∈ (0,
1
3
].
39
Khảo sát hàm
f(t) = 9t2 +
9
t2
với t ∈ (0, 1
3
] ta dễ dàng suy ra được
min P = min
t∈(0, 1
3
]
f(t) =
√
82.
Vậy giá trị nhỏ nhất của
x2 +
1
x2
+ y2 +
1
y2
+ z2 +
1
z2
là
√
82 đạt được khi x = y = z = 1
3
.
Bài toán 30. Giả sử ba số thực a, b, c > 0 thỏa mãn a + b + c ≤ 3
2
. Tìm giá trị nhỏ
nhất của
P = a2
+ b2
+ c2
+
1
a2b
+
1
b2c
+
1
c2a
.
Chứng minh. Ta đặt
t =
3
√
abc
khi đó 0 < t ≤ a+b+c
3
≤ 1
2
. Đưa về hàm một biến
P ≥ f(t) = 3t2
+
3
t3
với t ∈ (0, 1
2
]. Khi đó
f (t) = 6t −
9
t4
< 0 ∀t ∈ (0,
1
2
].
Hay f(t) là hàm nghịch biến trên (0, 1
2
], do đó
f(t) ≥ f(
1
2
) =
9
8
.
Vậy giá trị nhỏ nhất của P là 9
8
đạt được khi a = b = b = 1
2
.
Bài toán 31. (Thi thử đại 2012-2013. Trường THPT Kon Tum) Giả sử x, y, z là
các số thực không âm thỏa x + y + z = 1. Tìm giá trị lớn nhất của biểu thức
P = xy + yz + zx − 2xyz.
40
Chứng minh. Vì vai trò của x, y, z trong bài toán bình đẳng nên có thể giả sử x ≤ y ≤ z.
Mặt khác x + y + z = 1 nên ta suy ra x ∈ [0, 1
3
]. Ta thấy
P = xy + yz + zx − 2xyz = x(y + z) + yz(1 − 2x).
Ta có yz ≤ (y+z)2
4
mà x ∈ [0, 1
3
] nên 1 − 2x > 0. Suy ra
yz(1 − 2x) ≤
(y + z)2
4
(1 − 2x).
Điều này dẫn đến
P = x(1−x)+yz(1−2x) ≤ x(1−x)+
(y + z)2
4
(1−2x) = x(1−x)+
(1 − x)2
4
(1−2x).
Khảo sát hàm
f(x) = x(1 − x) +
(1 − x)2
4
(1 − 2x)
với x ∈ [0, 1
3
]. Ta tìm được
max P = max
x∈[0, 1
3
]
f(x) =
7
27
khi x = y = z = 1
3
. Vậy giá trị lớn nhất của P là 7
27
.
Bài toán 32. Giả sử x, y, z là các số thực không âm thỏa mãn x + y + z = 1. Tìm giá
trị nhỏ nhất của biểu thức
P = x3
+ y3
+ z3
+
15
4
xyz.
Chứng minh. Vai trò x, yz bình đẳng nên ta có thể giả sử x ≤ y ≤ z. Từ giả thiết suy
ra 0 ≤ x ≤ 1
3
và y + z = 1 − x. Áp dụng bất đẳng thức
yz ≤
(y + z)2
4
và
27x
4
− 3 < 0
ta nhận được
P = x3
+ y3
+ z3
+
15
4
xyz = x3
+ (y + z)3
− 3yz(y + z) +
15
4
xyz
41
= x3
+ (y + z)3
+ yz[
15x
4
− 3(y + z)] = x3
+ (1 − x)3
+ yz(
27x
4
− 3)
≥ x3
+ (1 − x)3
+
(y + z)2
4
(
27x
4
− 3) =
1
16
(27x3
− 18x2
+ 3x + 4).
Khảo sát hàm
f(x) =
1
16
(27x3
− 18x2
+ 3x + 4)
với 0 ≤ x ≤ 1
3
. Ta tìm được
min P = min
x∈[0, 1
3
]
f(x) =
1
4
khi (x, y, z) = (1
3
, 1
3
, 1
3
) hoặc (x, y, z) = (0, 1
2
, 1
2
) hoặc các hoán vị của nó. Vậy giá trị
nhỏ nhất của P là 1
4
.
Bài toán 33. (Trích đề thi thử đại học năm 2012-2013, trường THPT chuyên Nguyễn
Tất Thành, Kon Tum) Giả sử x, y là các số thực không âm thay đổi và thỏa mãn điều
kiện 4(x2
+ y2
+ xy) ≤ 1 + 2(x + y). Tìm giá trị lớn nhất của biểu thức
P = xy +
√
x + y − x2
− y2
.
Chứng minh. Ta biến đổi
P = xy+
√
x + y−(x2
+y2
) = xy+
√
x + y−[(x+y)2
−2xy] = 3xy+
√
x + y−(x+y)2
.
Đặt t = x + y. Khi đó
P ≤ 3
(x + y)2
4
+
√
x + y − (x + y)2
=
√
x + y −
(x + y)2
4
= f(t) =
√
t −
t2
4
.
Từ giả thiết 4(x2
+ y2
+ xy) ≤ 1 + 2(x + y) ta suy ra
4((x + y)2
− xy) ≤ 1 + 2(x + y)
và do đó
4((x + y)2
−
(x + y)2
4
) ≤ 4((x + y)2
− xy) ≤ 1 + 2(x + y).
Hay
3t2
− 2t − 1 ≤ 0.
42
Từ đó
−
1
3
≤ t ≤ t.
Mặt khác x, y không âm nên 0 ≤ t ≤ 1. Bây giờ ta đi tìm giá trị lớn nhất của
f(t) =
√
t −
t2
4
trên miền 0 ≤ t ≤ 1. Bằng cách dùng đạo hàm và khảo sát hàm f(t) ta được
max P = maxt∈[0,1] f(t) = 3
4
, khi x = y = 1
2
. Vậy giá trị lớn nhất của P là 3
4
.
Bài toán 34. (Trích đề thi học sinh giỏi Toán 12, bảng A, tỉnh Nghệ An, năm 2012-
2013 ) Giả sử a, b, c là các số thực dương. Tìm giá trị nhỏ nhất của biểu thức
P =
2
a +
√
ab + 3
√
abc
−
3
√
a + b + c
.
Chứng minh. Áp dụng bất đẳng thức AG-GM, ta có
a +
√
ab +
3
√
abc ≤ a +
1
2
√
a4b +
1
4
3
√
a4b16c
≤ a +
1
2
a + 4b
2
+
1
4
a + 4b + 16c
3
=
4
3
(a + b + c).
Đẳng thức xảy ra khi và chỉ khi a = 4b = 16c. Suy ra
P ≥
3
2(a + b + c)
−
3
√
a + b + c
.
Đặt t = a + b + c, t > 0. Khi đó ta có
P ≥ f(t) =
3
2t
−
3
√
t
.
Khảo sát hàm số
f(t) =
3
2t
−
3
√
t
với t > 0. Ta tìm được
min
t>0
f(t) = −
3
2
khi và chỉ khi t = 1. Vậy giá trị nhỏ nhất của P là −3
2
đẳng thức xảy ra khi và chỉ khi
a = 16
21
, b = 4
21
, c = 1
21
.
43
Bài toán 35. Giả sử x, y, z ∈ [1, 2]. Tìm giá trị lớn nhất của biểu thức
P = (x + y + z)(
1
x
+
1
y
+
1
z
).
Chứng minh. Ta có
P = (
x
y
+
y
x
) + (
y
z
+
z
x
) + (
x
z
+
z
x
) + 3.
Giả sử 1 ≤ x ≤ y ≤ z ≤ 2, suy ra
(1 −
x
y
)(1 −
y
z
) ≥ 0
và
(1 −
y
x
)(1 −
z
y
) ≥ 0.
Suy ra
P = (
x
y
+
y
x
) + (
y
z
+
z
x
) + (
x
z
+
z
x
) + 3 ≤ 5 + 2(
x
z
+
z
x
).
Đặt t = x
z
với t ∈ [1
2
, 1]. Khi đó
P ≤ f(t) = 5 + 2(t +
1
t
)
với t ∈ [1
2
, 1]. Dùng đạo hàm khảo sát hàm
f(t) = 5 + 2(t +
1
t
)
với t ∈ [1
2
, 1]. Ta thấy
max
t∈[1
2
,1]
f(t) = 10.
Vậy giá trị lớn nhất của P là 10 đạt được khi x = 1, y = z = 2.
Bài toán 36. Giả sử 1 ≤ x, y, z ≤ 3 và x + y + z = 6. Tìm giá trị lớn nhất của
P = x2
+ y2
+ z2
.
Chứng minh. Vai trò x, y, z trong bài toán bình đẳng nên có thể giả sử x =
max{x, y, z}. Khi đó
P = x2
+ y2
+ z2
≤ x2
+ y2
+ z2
+ 2(y − 1)(z − 1)
44
= x2
+ (y + z)2
− 2(y + z) + 2 = x2
+ (6 − x)2
− 2(6 − x) + 2.
Đẳng thức xảy ra khi y = 1 hoặc z = 1. Xét hàm số
f(x) = x2
+ (6 − x)2
− 2(6 − x) + 2
với 2 ≤ x ≤ 3. Ta dễ dàng tìm được giá trị lớn nhất của f(x) bằng 14 khi x = 2 hoặc
x = 3. Vậy giá trị lớn nhất của P là 14, chẳng hạn khi x = 3, y = 1, z = 2.
2.4 Đánh giá gián tiếp thông qua biểu thức bậc
nhất
Nếu bài toán có dạng sau cho n ∈ N và các số a1, a2, . . . an ∈ D thỏa mãn
a1 + a2 + · · · + an = nα, với α ∈ D. Hàm số y = f(x) trên khoảng D không lồi
và cũng không lõm trên D nhưng đồ thị vẫn “nằm trên” tiếp tuyến của nó tại D. Trong
bài này không thể áp dụng được BĐT hàm lồi được nhưng vẫn có thể dùng phương
pháp “tiếp tuyến” để giải quyết bài toán. Sau đây xin được trình bày một số bài toán
minh họa cho phương pháp trên được trích dẫn từ một số đề thi Olympic của nước
ta và các nước trên thế giới. Trong một số bài toán có thể chúng ta phải sử dụng linh
hoạt các giả thiết và tính chất của các biểu thức trong bài toán để vận dụng phương
pháp một cách hiệu quả nhất.
Bài toán 37. ( Olimpic 30/4- 2006). Giả sử a, b, c là các số thực dương. Tìm giá trị
lớn nhất của biểu thức
Q =
a(b + c)
(b + c)2 + a2
+
b(c + a)
(c + a)2 + b2
+
c(a + b)
(a + b)2 + c2
.
Chứng minh. Do Q có tính thuần nhất nên chỉ xét giá trị của Q với a + b + c = 1. Ta
có
Q =
a(1 − a)
(1 − a)2 + a2
+
b(1 − b)
(1 − b)2 + b2
+
c(1 − c)
(1 − c)2 + c2
= f(a) + f(b) + f(c)
trong đó
f(x) =
x(1 − x)
(1 − x)2 + x2
=
x − x2
1 − 2x + 2x2
với x ∈ (0, 1).
45
Khi đó tiếp tuyến tại x = 1
3
có phương trình
y =
27
25
(x −
1
3
) +
2
5
=
27
25
x +
1
25
.
Mặc dầu trong khoảng (0, 1) đồ thị (C) của hàm số y = f(x) không lồi, nhưng vẫn có
f(x) ≤
27
25
x +
1
25
∀x ∈ (0, 1). (2.16)
Thật vậy (2.16) tương đương với
x − x2
2x2 − 2x + 1
≤
27x + 1
25
⇒ 54x3
− 27x2
+ 1 ≥ 0.
Xét hàm số
g(x) = 54x3
− 27x2
+ 1
với x ∈ (0, 1). Khi đó
g (x) = 54x(3x − 1).
Lập bảng biến thiên của hàm số y = g(x) ta được kết quả
g(x) ≥ 0 ∀x ∈ (0, 1).
Áp dụng BĐT (2.16) cho các số a, b, c ∈ (0, 1) ta có
f(a) + f(b) + f(c) ≤
27
25
(a + b + c) +
3
25
=
6
5
.
Vậy giá trị lớn nhất của biểu thức
Q =
a(b + c)
(b + c)2 + a2
+
b(c + a)
(c + a)2 + b2
+
c(a + b)
(a + b)2 + c2
là 6
5
đẳng thức xảy ra khi a = b = c.
Bài toán 38. (Hồng Kong, 2005). Giả sử a, b, c, d là các số dương thỏa mãn a + b +
c + d = 1. Chứng minh rằng giá trị nhỏ nhất của
6(a3
+ b3
+ c3
+ d3
) − (a2
+ b2
+ c2
+ d2
)
là 1
8
.
46
Chứng minh. Từ giả thiết ta có a, b, c, d ∈ (0, 1). Bây giờ ta chứng minh rằng
f(a) + f(b) + f(c) + f(d) ≥
1
8
(2.17)
trong đó
f(x) = 6x3
− x2
.
Xét f(x) với x ∈ (0, 1). Tiếp tuyến của đồ thị hàm số y = f(x) tại x = 1
4
có phương
trình là y = 5
8
x − 1
8
. Mặt khác
f(x) − (
5
8
x −
1
8
) = 6x3
− x2
− (
5
8
x −
1
8
) =
1
8
(4x − 1)2
(3x + 1) ≥ 0
với mọi x ∈ (0, 1) hay
f(x) ≥
5
8
x −
1
8
.
Từ đó suy ra
f(a) + f(b) + f(c) + f(d) ≥
5
8
(a + b + c + d) − 4
1
8
=
1
8
.
Đẳng thức xảy ra khi a = b = c = d = 1
4
. Vậy giá trị nhỏ nhất của
6(a3
+ b3
+ c3
+ d3
) − (a2
+ b2
+ c2
+ d2
)
là 1
8
.
Bài toán 39. ( Mở rộng bài toán thi Olimpic Ba Lan, 1996 và Olimpic 30-4, 1999)
Giả sử các số thực a, b, c thỏa mãn a + b + c = 1. Tìm giá trị lớn nhất của biểu thức
a
1 + a2
+
b
1 + b2
+
c
1 + c2
.
Chứng minh. Đặt
f(x) =
x
1 + x2
.
Ta sẽ chứng minh rằng
f(a) + f(b) + f(c) ≤
9
10
. (2.18)
Thật vậy, có
f (x) =
1 − x2
(1 + x2)2
,
47
hơn nữa f (x) = 0 khi x = −1 hoặc x = 1. Xét các trường hợp xảy ra
Trường hợp 1. Có một số, giả sử a ∈ (−∞, −3] nên b + c ≥ 4 nên có một số, giả sử
b ≥ 2. Khi đó ta có
f(a) + f(b) + f(c) < 0 +
2
5
+
1
2
=
9
10
.
Trường hợp 2. Có một số, giả sử a ∈ (−3, −1
3
] . Khi đó
f(a) + f(b) + f(c) ≤ −
3
10
+
1
2
+
1
2
=
7
10
<
9
10
.
Trường hợp 3. Cả ba số a, b, c ∈ (−1
3
, +∞). Khi đó tiếp tuyến của đồ thị y = f(x) tại
x = 1
3
có phương trình
y =
18
25
x +
3
50
.
Ta có
f(x) − (
18
25
x +
3
50
) =
x
1 + x2
− (
18
25
x +
3
50
) =
(3x − 1)2
(4x + 3)
50(1 + x2)
≤ 0 ∀x > −
1
3
.
Áp dụng bất đẳng thức này cho các số a, b, c > −1
3
và a + b + c = 1 ta có
f(a) + f(b) + f(c) ≤
18
25
+ 3
3
50
=
9
10
.
Vậy trong mọi trường hợp bất đẳng thức (2.18) đều đúng. Đẳng thức xảy ra khi
a = b = c = 1
3
. Vậy giá trị lớn nhất của biểu thức
a
1 + a2
+
b
1 + b2
+
c
1 + c2
là 9
10
.
Bài toán 40. (Rumania, 2005). Giả sử các số thực dương a, b, c thỏa mãn a+b+c = 3.
Chứng minh rằng giá trị nhỏ nhất của
1
a2
+
1
b2
+
1
c2
− (a2
+ b2
+ c2
)
là 0.
Chứng minh. Ta cần chứng minh rằng
1
a2
+
1
b2
+
1
c2
− (a2
+ b2
+ c2
) ≥ 0. (2.19)
48
Theo giả thiết a, b, c > 0 ta được a2
+ b2
+ c2
< (a + b + c)2
= 9. Từ đó nếu có một
trong ba số, giả sử a < 1
3
thì
1
a2
+
1
b2
+
1
c2
> 9 > a2
+ b2
+ c2
nên (2.19) đúng. Bây giờ ta chỉ cần xét trường hợp a, b, c ≥ 1
3
. Vì a + b + c = 3 nên
a, b, c ≤ 7
3
. Vậy
a, b, c ∈ [
1
3
,
7
3
].
Xét hàm số
f(x) =
1
x2
− x2
trên [
1
3
,
7
3
].
Tiếp tuyến của đồ thị hàm số y = f(x) tại x = 1 là y = −4x+4. Do g(x) = x2
−2x−1 =
(x − 1)2
− 2 ≤ (4
3
)2
− 2 < 0 trên [1
3
, 7
3
], nên ta có
f(x) − (−4x + 4) = −
(x − 1)2
(x2
− 2x − 1)
x2
≥ 0 ∀x ∈ [
1
3
,
7
3
].
Hay
f(x) ≥ −4x + 4 ∀x ∈ [
1
3
,
7
3
].
Áp dụng cho các số a, b, c ∈ [1
3
, 7
3
] ta có
f(a) + f(b) + f(c) ≥ −4(a + b + c) + 4.3 = 0.
Nên bất đẳng thức (2.19) được chứng minh. Đẳng thức xảy ra khi và chỉ khi
a = b = c = 1. Vậy giá trị nhỏ nhất của
1
a2
+
1
b2
+
1
c2
− (a2
+ b2
+ c2
)
là 0.
Bài toán 41. (Trung Quốc, 2005). Giả sử các số không âm a, b, c thỏa mãn a+b+c = 1.
Chứng minh rằng giá trị nhỏ nhất của
10(a3
+ b3
+ c3
) − 9(a5
+ b5
+ c5
) (2.20)
là 1.
49
Chứng minh. Đặt
f(x) = 10x3
− 9x5
.
Nên để chứng minh (2.20) ta sẽ chứng minh rằng
f(a) + f(b) + f(c) ≥ 1. (2.21)
Để chứng minh điều này, ta xét 2 trường hợp.
Trường hợp 1. Trong ba số a, b, c có một số, giả sử a ≥ 9
10
. Khi đó thì
a ∈ [
9
10
, 1], b, c ∈ [0,
1
10
].
Xét hàm số f(x) trên đoạn [ 9
10
, 1] ta có
f (x) = 30x2
− 45x4
= 15x2
(2 − 3x2
) ≤ 0 với mọi x ∈ [
9
10
, 1].
Vậy f(x) nghịch biến trên đoạn này và từ đó
f(a) ≥ f(1) = 1 với a ∈ [
9
10
, 1].
Hơn nữa với b, c ∈ [0, 1
10
] thì f(b) = 10b3
− 9b5
≥ 0 và f(c) = 10c3
− 9c5
≥ 0 nên
f(a) + f(b) + f(c) ≥ 1 + 0 + 0 = 1
hay (2.21) đúng.
Trường hợp 2. Các số a, b, c ∈ [0, 9
10
]. Khi đó tiếp tuyến của đồ thị hàm số y = f(x) tại
x = 1
3
có phương trình
y =
25
9
x −
16
27
.
Ta có
f(x)−(
25
9
x−
16
27
) = 10x3
−9x5
−(
25
9
x−
16
27
) = −
1
27
(3x−1)2
(27x3
+18x2
−21x−16)).
Xét hàm số
g(x) = 27x3
+ 18x2
− 21x − 16 trên đoạn [0,
9
10
].
Ta có
g (x) = 81x2
+ 36x − 21
50
g (x) = 0 khi x = 1
3
hoặc x = −7
9
. Ta thấy trên đoạn [0, 9
10
] thì g(x) < 0 nên
f(x) − (
25
9
x −
16
27
) ≥ 0
hay
f(x) ≥
25
9
x −
16
27
∀x ∈ [0,
9
10
].
Áp dụng cho các số a, b, c ∈ [0, 9
10
] và a + b + c = 1 ta có
f(a) + f(b) + f(c) ≥
25
9
(a + b + c) − 3
16
27
= 1.
Hay (2.21) đúng. Vậy giá trị nhỏ nhất của 10(a3
+ b3
+ c3
) − 9(a5
+ b5
+ c5
) là 1. Đẳng
thức xảy ra khi a = b = c = 1
3
hoặc (a, b, c) là một hoán vị bất kì của bộ (1, 0, 0).
Bài toán 42. (Moldova,2005) . Giả sử các số dương a, b, c thỏa mãn a4
+ b4
+ c4
= 3.
Tìm giá trị lớn nhất của biểu thức
1
4 − ab
+
1
4 − bc
+
1
4 − ca
.
Chứng minh. Vì ab ≤ a2+b2
2
nên
1
4 − ab
≤
2
8 − (a2 + b2)
do đó
1
4 − ab
+
1
4 − bc
+
1
4 − ca
≤
2
8 − (a2 + b2)
+
2
8 − (b2 + c2)
+
2
8 − (c2 + a2)
.
Để vận dụng giả thiết a4
+ b4
+ c4
= 3 ta đặt
x = (b2
+ c2
)2
, y = (c2
+ a2
)2
, z = (a2
+ b2
)2
thì ta có x, y, z > 0 và
x + y + z = (b2
+ c2
)2
+ (c2
+ a2
)2
+ (a2
+ b2
)2
≤ 4(a4
+ b4
+ c4
) = 12.
Ta sẽ chứng minh
1
8 −
√
x
+
1
8 −
√
y
+
1
8 −
√
z
≤
1
2
. (2.22)
51
Xét hàm số
f(t) =
1
8 −
√
t
với t ∈ (0, 12).
Phương trình tiếp tuyến của đồ thị y = f(t) tại t = 4 có phương trình
y =
1
144
t +
5
36
.
Hơn nữa ta có
1
8 −
√
t
− (
1
144
t +
5
36
) = −
1
144
(
√
t − 2)2
(4 −
√
t) ≤ 0
với mọi t ∈ (0, 12). Vậy
f(t) ≤
1
144
t +
5
36
.
Từ đó
f(x) + f(y) + f(z) ≤
1
144
(x + y + z) + 3
5
36
≤
1
144
12 + 3
5
36
=
1
2
.
Vậy giá trị lớn nhất của biểu thức
1
4 − ab
+
1
4 − bc
+
1
4 − ca
là 1
2
. Đẳng thức xảy ra khi a = b = c = 1.
2.5 Phương pháp sử dụng tính chất của hàm lồi,
hàm lõm
Bài toán 43. (Bất đẳng thức Karamata). Cho hai dãy số {xk, yk ∈ I(a, b), k =
1, 2, . . . , n} thỏa mãn các điều kiện
x1 ≥ x2 ≥ · · · ≥ xn, y1 ≥ y2 ≥ · · · ≥ yn
và 


x1 ≥ y1
x1 + x2 ≥ y1 + y2
. . . . . .
x1 + x2 + . . . xn−1 ≥ y1 + y2 + . . . yn−1
x1 + x2 + . . . xn = y1 + y2 + . . . yn
(2.23)
52
Khi đó, ứng với mọi hàm lồi f(x) (f (x) ≥ 0) trên I(a, b), ta đều có
f(x1) + f(x2) + · · · + f(xn) ≥ f(y1) + f(y2) + · · · + f(yn). (2.24)
Ta cũng có phát biểu tương tự đối với hàm lõm bằng cách đổi chiều dấu bất đẳng thức.
Chứng minh. Sử dụng biểu diễn đối với hàm lồi
f(x1) + f(x2) + · · · + f(xn) = min
t1,...,tn∈I(a,b)
[
n
i=1
f(t1)
n
i=1
f(xi − t1)f (ti)]. (2.25)
Không mất tính tổng quát, ta giả thiết bộ số t1, . . . , tn ∈ I(a, b) cũng là một bộ số
giảm, tức là
t1 ≥ t1 ≥ ... ≥ tn.
Khi đó, để chứng minh (2.25), ta chỉ cần chứng minh rằng
x1f (t1) + x2f (t2) + · · · + xn1f (tn) ≥
≥ y1f (t1) + y2f (t2) + · · · + ynf (tn). (2.26)
Sử dụng biến đổi Abel
x1f (t1) + x2f (t2) + · · · + xn1f (tn) =
= S1[f (t1) − f (t2)] + S2[f (t2) − f (t3)] + · · · +
+ Sn−1[f (tn−1 − f (tn)] + Snf (tn) (2.27)
với
Sk(x) := x1 + x2 + · · · + xk.
Vì rằng f (x) ≥ 0 nên f (xk) ≤ f (xk−1). Mặt khác, do Sk(x) ≥ Sk(y) (k =
1, 2, . . . , n − 1) và Sn(x) = Sn(y), ta thu được ngay (2.26). Chứng minh được hoàn
thành.
Hệ quả 2.1 (Bất đẳng thức Jensen). Với mọi hàm lồi f(x) trên I(a, b) và với mọi
xi ∈ I(a, b) (i = 1, 2, . . . , n), ta luôn có bất đẳng thức
f(x1) + f(x2) + · · · + f(xn)
n
≥ f(
x1 + x2 + · · · + xn
n
).
53
Chứng minh. Do tính chất đối xứng, không mất tính tổng quát, ta có thể giả sử
x1 ≥ x2 ≥ · · · ≥ xn.
Khi đó, ta có 


x1 ≥ x
x1 + x2 ≥ 2x
. . . . . .
x1 + x2 + · · · + xn−1 ≥ (n − 1)x
x1 + x2 + · · · + xn = nx
(2.28)
trong đó
x =
x1 + x2 + · · · + xn
n
.
Theo bất đẳng thức Karamata, ta có
f(x1) + f(x2) + · · · + f(xn) ≥ nf(
x1 + x2 + · · · + xn
n
).
Chứng minh được hoàn thành.
Ở phần tiếp theo, luận văn trình bày một số áp dụng của bất đẳng thức Karamata
và các hệ quả của nó.
Bài toán 44. Cho 2n số thực dương ai, bi (i = 1, 2, . . . , n) thỏa mãn các điều kiện



a1 ≥ a2 ≥ · · · ≥ an
b1 ≥ b2 ≥ · · · ≥ bn
a1 ≥ b1, a1a2 ≥ b1b2, . . . , a1a2 . . . an = b1b2 . . . bn.
Chứng minh rằng
a1 + a2 + · · · + an ≥ b1 + b2 + · · · + bn.
Chứng minh. Đặt xi = ln ai, yi = ln bi (i = 1, 2, . . . , n). Với các điều kiện đã cho, ta
54
có 


x1 ≥ y1
x1 + x2 ≥ y1 + y2
. . . . . .
x1 + x2 + . . . xn−1 ≥ y1 + y2 + . . . yn−1
x1 + x2 + . . . xn = y1 + y2 + . . . yn
Xét hàm số f(x) = ex
với x ∈ (0, +∞). Ta có f (x) = ex
> 0 ∀x ∈ (0, +∞) nên hàm
số f(x) lồi trên khoảng (0, +∞). Khi đó, theo bất đẳng thức Karamata, ta có
ex1
+ ex2
+ · · · + exn
≥ ey1
+ ey2
+ · · · + eyn
hay
a1 + a2 + · · · + an ≥ b1 + b2 + · · · + bn.
Chứng minh được hoàn thành.
Bài toán 45. Giả sử các số thực a, b, c thỏa mãn



0 ≤ c ≤ b ≤ a ≤ 8
a + b ≤ 13
a + b + c = 15.
Tìm giá trị lớn nhất và nhỏ nhất của biểu thức
M = a2
+ b2
+ c2
.
Chứng minh. Từ giả thiết, ta có



0 ≤ a ≤ 8
a + b ≤ 8 + 5
a + b + c = 8 + 5 + 2.
Xét hàm số f(x) = x2
, ta có f (x) = 2 > 0 ∀x ∈ R nên hàm số f(x) lồi thực sự trên
R. Do đó, theo bất đẳng thức Karamata, ta có
f(a) + f(b) + f(c) ≤ f(8) + f(5) + f(2)
55
hay
a2
+ b2
+ c2
≤ 64 + 25 + 4 = 93.
Đẳng thức xảy ra khi a = 8, b = 5, c = 2 Vậy
max M = 93 đạt được khi a = 8, b = 5, c = 2.
Áp dụng bất đẳng thức B-C-S cho hai bộ số (a, b, c) và (1, 1, 1), ta có
(1a + 1b + 1c)2
≤ (12
+ 12
+ 12
)(a2
+ b2
+ c2
).
Suy ra
(a + b + c)2
≤ 3(a2
+ b2
+ c2
).
Nên ta được
152
≤ 3M
hay
M ≥ 75.
Đẳng thức xảy ra khi a = b = c = 5. Vậy
min M = 75 đạt được khi a = b = c = 5.
Bài toán 46. Giả sử A, B, C là 3 góc của một tam giác nhọn. Chứng minh rằng giá
trị lớn nhất của
cos A + cos B + cos C
là 3
2
.
Chứng minh. Không mất tính tổng quát, ta coi A ≥ B ≥ C. Khi đó A ≥ π
3
, C ≤ π
3
và
A + B = π − C ≥ 2π
3
nên



A ≥ π
3
A + B ≥ π
3
+ π
3
A + B + C = π
3
+ π
3
+ π
3
.
56
Xét hàm số f(x) = cos x với x ∈ [0, π
2
]. Ta có f (x) = − cos x < 0 ∀x ∈ [0, π
2
] nên
hàm số f(x) lõm trên đoạn [0, π
2
]. Khi đó, theo bất đẳng thức Karamata, ta có
f(A) + f(B) + f(C) ≤ 3f(
π
3
)
hay
cos A + cos B + cos C ≤
3
2
.
Vậy giá trị lớn nhất của
cos A + cos B + cos C
là 3
2
đạt được khi A = B = C = π
3
.
Bài toán 47. Giả sử tam giác ABC không nhọn. Chứng minh rằng giá trị nhỏ nhất
của
tan
A
2
+ tan
B
2
+ tan
C
2
là 2
√
2 − 1.
Chứng minh. Không mất tính tổng quát, ta coi A ≥ B ≥ C. Khi đó



A ≥ π
2
A + B ≥ π
2
+ π
4
A + B + C = π
2
+ π
4
+ π
4
.
hay 


A
2
≥ π
4
A
2
+ B
2
≥ π
4
+ π
8
A
2
+ B
2
+ C
2
= π
4
+ π
8
+ π
8
.
Xét hàm số f(x) = tan x với x ∈ (0, π
2
). Ta có
f (x) =
2 sin x
cos3 x
≥ 0 ∀x ∈ (0,
π
2
)
nên hàm số f(x) lồi trên khoảng (0, π
2
). Khi đó, áp dụng bất đẳng thức Karamata, ta
được
tan
A
2
+ tan
B
2
+ tan
C
2
≥ tan
π
4
+ tan
π
8
+ tan
π
8
.
57
Để ý rằng tan π
8
=
√
2 − 1 nên
tan
π
4
+ tan
π
8
+ tan
π
8
= 2
√
2 − 1.
Vậy
tan
A
2
+ tan
B
2
+ tan
C
2
≥ 2
√
2 − 1.
Vậy giá trị nhỏ nhất của
tan
A
2
+ tan
B
2
+ tan
C
2
là 2
√
2 − 1 đạt được khi (A, B, C) = (π
2
, π
4
, π
4
) và các hoán vị của nó.
Bài toán 48. (IMO 2000). Giả sử các số dương a, b, c thỏa mãn điều kiện abc = 1.
Chứng minh rằng
(a − 1 +
1
b
)(b − 1 +
1
c
)(c − 1 +
1
a
) ≤ 1.
Chứng minh. Vì abc = 1 nên ta đặt
a =
x
y
, b =
y
z
, c =
z
x
với x, y, z > 0. Ta viết bất đẳng thức đã cho theo x, y, z ta có
(
x
y
− 1 +
z
y
)(
y
z
b − 1 +
x
z
)(
z
x
− 1 +
y
x
) ≤ 1.
Hay
(x − y + z)(y − z + x)(z − x + y) ≤ xyz.
Để ý rằng (x−y+z)+(y−z+x) = 2x > 0 do đó trong ba số x−y+z, y−z+x, z−x+y
không thể có trường hợp hai số cùng âm. Nếu trong ba số trên có một hoặc ba số âm,
hiển nhiên ta có bất đẳng thức cần chứng minh. Trường hợp cả ba số đó đều dương,
bằng cách lấy lôgarit hai vế với cơ số e, ta được
ln(x − y + z) + ln(y − z + x) + ln(z − x + y) ≤ ln x + ln y + ln z.
Không mất tính tổng quát, ta coi x ≥ y ≥ z. Khi đó, ta có x + y − z ≥
x, (x+y−z)+(x−y+z) = 2x ≥ x+y, (x+y−z)+(x−y+z)+(z −x+y) = x+y+z.
58
Xét hàm số f(x) = ln x với x > 0. Ta có f (x) = − 1
x2 < 0 ∀x > 0 nên hàm số f(x)
lõm trên khoảng (0, +∞). Khi đó theo bất đẳng thức Karamata, ta có
ln(x − y + z) + ln(y − z + x) + ln(z − x + y) ≤ ln x + ln y + ln z.
Đẳng thức xảy ra khi và chỉ khi x = y = z hay a = b = c.
Chương 3
Cực trị hàm nhiều biến
3.1 Cực trị tự do
Sau đây, luận văn xin trình bày về cực trị tự do của hàm nhiều biến được tham
khảo trong [3]. Giả sử z = f(x1, . . . , xn) là một hàm xác định và liên tục ở trong miền
D mở, M(a1, . . . , an) ∈ D. Ta nói rằng hàm f(x1, . . . , xn) đạt được giá trị cực đại (cực
tiểu) tại M nếu tại mọi điểm (x1, . . . , xn) thuộc một lân cận nào đó của M(a1, . . . , an)
thì
f(x1, . . . , xn) ≤ f(a1, . . . , an)
( tương ứng f(x1, . . . , xn) ≥ f(a1, . . . , an)).
Giá trị cực đại và giá trị cực tiểu của hàm f(x1, . . . , xn) được gọi là cực trị của hàm
số. Tại M(a1, . . . , an) mà hàm đạt được cực trị gọi là điểm cực trị của hàm số.
Định lý 3.1. (Điều kiện cần của cực trị [3]) Nếu hàm z = f(x1, . . . , xn) đạt
được cực trị tại M(a1, . . . , an) và tại đây hàm số có các đạo hàm riêng hữu hạn,
fxj
(a1, . . . , an), j = 1, 2, . . . , n thì các đạo hàm riêng đó phải triệt tiêu
fxj
(a1, . . . , an) = 0 với mọi j = 1, 2, . . . , n.
Định lý 3.2. (xem [3]). Giả sử M(x0, y0) là điểm thỏa mãn zx(x0, y0) = 0, zy(x0, y0) =
0 của hàm z = f(x, y) và tại đây hàm z = f(x, y) có các đạo hàm riêng cấp 2 liên tục
59
60
và ta gọi
A =
∂2
z
∂x2
(x0, y0), B =
∂2
z
∂x∂y
(x0, y0), C =
∂2
z
∂y2
(x0, y0).
1. Nếu B2
− AC < 0 thì z = f(x, y) có cực trị tại M(x0, y0). Hơn nữa hàm z = f(x, y)
đạt cực đại tại M(x0, y0) nếu A < 0, z = f(x, y) đạt cực tiểu tại M(x0, y0) nếu A > 0.
2. Nếu B2
− AC > 0 thì z = f(x, y) không có cực trị tại M(x0, y0).
3. Nếu B2
− AC = 0: chưa kết luận được cực trị của hàm z = f(x, y) tại M(x0, y0).
Bài toán 49. Tìm giá trị nhỏ nhất của các hàm số sau
z = x3
+ y3
− 3xy trong đó 0 ≤ x, y ≤ 2.
Chứng minh. Ta thấy 


zx = 3x2
− 3y = 0
zy = 3y2
− 3x = 0.
Tương đương với 


x2
= y
y2
= x.
Hay (x, y) = (0, 0), (1, 1). Tọa độ các điểm dừng là M1(1, 1), M2(0, 0). Hơn nữa



zxx = 6x
zxy = −3
zyy = 6y.
Do vậy
A1 = 6.1 = 6 > 0, A2 = 0, B1 = −3, B2 = −3, C1 = 6, C2 = 0.
Ta có
B2
1 − A1C1 = 9 − 36 = −27 < 0
hàm số đạt cực tiểu tại M1(1, 1). Vậy giá trị nhỏ nhất của z là z(M1) = −1. Dễ thấy
tại biên của D = {(x, y) : 0 ≤ x, y ≤ 2}, thì z ≥ −1. Vậy giá trị nhỏ nhất của các hàm
số z = x3
+ y3
− 3xy trong đó 0 ≤ x, y ≤ 2 là −1 đạt được khi x = y = 1.
61
Bài toán 50. Tìm giá trị nhỏ nhất và lớn nhất của hàm số sau
z = x3
+ 2y3
− 3x − 6y trong đó − 2 ≤ x, y ≤ 2.
Chứng minh. Ta thấy 


zx = 3x2
− 3 = 0
zy = 6y2
− 6 = 0.
Tương đương với 


x2
= 1
y2
= 1.
Hay (x, y) = (1, 1), (−1, −1), (1, −1), (−1, 1). Tọa độ các điểm dừng là
M1(1, 1), M2(−1, −1), M3(−1, 1), M4(1, −1).
Hơn nữa 


zxx = 6x
zxy = −3
zyy = 6y.
Do vậy, tại M1 có
A1 = 6.1 = 6 > 0, B1 = 0, C1 = 12.
Ta có
B2
1 − A1C1 = −72 < 0
hàm số đạt cực tiểu tại M1. Tại M2 có
A2 = −6 < 0, B2 = 0, C2 = −12.
Ta có
B2
2 − A2C2 = −72 < 0
hàm số đạt cực đại tại M3. Tại M3 có
A3 = −6 < 0, B3 = 0, C3 = 12.
62
Ta có
B2
3 − A3C3 = 72 > 0
suy ra M3 không là điểm cực trị. Tại M4 có
A4 = 6 > 0, B4 = 0, C4 = −12.
Ta có
B2
4 − A4C4 = 72 > 0
suy ra M4 không là điểm cực trị. Dễ thấy tại biên của tập D = {(x, y) : −2 ≤ x, y ≤ 2},
tức x hoặc y thuộc {2, −2} thì −6 ≤ z ≤ 6. Vậy giá trị nhỏ nhất và lớn nhất của
z = x3
+ 2y3
− 3x − 6y lần lượt là là −6 và 6.
Bài toán 51. Tìm giá trị lớn nhất và nhỏ nhất của hàm số
z = 8x2
+ 3y2
+ 1 − (2x2
+ y2
+ 1)2
trong miền tròn đóng D xác định bởi x2
+ y2
≤ 1.
Chứng minh. Rõ ràng z liên tục với mọi x, y nên nó đạt giá trị lớn nhất M và giá trị
nhỏ nhất m trên miền D. Ta có



zx = 16x − 2(2x2
+ y2
+ 1)4x = 8x(1 − 2x2
− y2
) = 0
zy = 6y − 2(2x2
+ y2
+ 1)2y = 2y(1 − 4x2
− 2y2
) = 0.
Hay (x, y) = (0, 0), (0, 1√
2
), (0, −1√
2
), ( 1√
2
, 0), (−1√
2
, 0). Tọa độ các điểm dừng là
O(0, 0), A1(0,
1
√
2
), A2(0,
−1
√
2
), A3(
1
√
2
, 0), A4(
−1
√
2
, 0),
và cả 5 điểm dừng này đều nằm trong miền D. Tính giá trị của z tại các điểm ấy ta
được z(O) = 0, z(A1) = z(A2) = 1
4
, z(A3) = z(A4) = 1. Bây giờ, ta xét giá trị của z
trên biên của miền D. Trên biên ấy x2
+ y2
= 1, vậy y2
= 1 − x2
, do đó
z = 8x2
+ 3(1 − x2
) + 1 − (2x2
+ 1 = x2
+ 1)2
= x2
(1 − x2
)
trong đó −1 ≤ x ≤ 1. Hàm này bằng 0 khi x = 1, −1 và đạt giá trị lớn nhất bằng 1
4
khi x = 1√
2
, −1√
2
. Vậy hàm số đạt giá trị nhỏ nhất m = 0 tại gốc O và đạt giá trị lớn
nhất M = 1 tại các điểm A3, A4.
63
3.2 Cực trị có điều kiện
Xét bài toán: Tìm cực trị của hàm số f(x1, . . . , xn) với điều kiện φj(x1, . . . , xn) =
0, j = 1, . . . , m. Phương pháp làm như sau (xem [3]): Xét hàm Lagrange
L(x1, . . . , xn, λ1, . . . , λm) = f(x1, . . . , xn) +
m
j=1
λjφj(x1, . . . , xn).
Giải hệ 


Lxj
(x1, . . . , xn, λ1, . . . , λm) = 0 ∀j = 1, . . . , n
φj(x1, . . . , xn) = 0, j = 1, . . . , m
để tìm các điểm dừng. Sau đó xét dấu của dạng vi phân cấp 2 là d2
L để tìm cực trị
của hàm số ban đầu.
Bài toán 52. Tìm giá trị lớn nhất và nhỏ nhất của hàm số u = x − 2y + 2z với điều
kiện
x2
+ y2
+ z2
− 1 = 0.
Chứng minh. Rõ ràng u liên tục với mọi x, y, z nên nó đạt giá trị lớn nhất M và giá
trị nhỏ nhất m trên miền D. Ta lập hàm Lagrange
L(x, y, z, λ) = u = x − 2y + 2z + λ(x2
+ y2
+ z2
− 1).
Xét hệ phương trình 


Lx = 1 + 2λx = 0
Ly = −2 + 2λy = 0
Lz = 2 + 2λz = 0
x2
+ y2
+ z2
− 1 = 0.
Hay 


x
1
= y
−2
= z
2
x2
+ y2
+ z2
= 1.
64
Từ đây, ta tìm được 2 điểm dừng là M1(1
3
, −2
3
, 2
3
) ứng với λ = −3
2
và M2(−1
3
, 2
3
, −2
3
) ứng
với λ = 3
2
. Tính
d2
L = Lxxdx2
+ Lyydy2
+ Lzzdz2
+ 2Lxydxdy + 2Lyzdydz + 2Lzxdzdx
trong đó Lxx = 2λ, Lyy = 2λ, Lzz = 2λ, Lxy = Lyz = Lzx = 0. Do đó
d2
L = 2λ(dx2
+ dy2
+ dz2
).
Từ đó suy ra giá trị lớn nhất và nhỏ nhất của hàm số u = x − 2y + 2z với điều kiện
x2
+ y2
+ z2
− 1 = 0 lần lượt là 1
3
( đạt được khi (x, y, z) = (1
3
, −2
3
, 2
3
)) và −1
3
( đạt được
khi (x, y, z) = (−1
3
, 2
3
, −2
3
)).
Bài toán 53. Tìm giá trị nhỏ nhất của hàm số u = x2
+ y2
với điều kiện
x + y = 1.
Chứng minh. Rõ ràng z liên tục với mọi x, y nên nó đạt giá trị nhỏ nhất m trên miền
D. Lập hàm Lagrange
L(x, y) = x2
+ y2
+ λ(x + y − 1).
Xét hệ phương trình 


Lx = 2x + λ = 0
Ly = 2y + λ = 0
x + y − 1 = 0
ta tìm được điểm dừng là M(1
2
, 1
2
) với λ = −1. Tính
d2
L(
1
2
,
1
2
, −1) = Lxxdx2
+ 2Lxydxdy + Lyydy2
|( 1
2
, 1
2
,−1)
trong đó Lxx = 2, Lxy = 0, Lyy = 2. Do đó
d2
L(
1
2
,
1
2
, −1) = 2dx2
+ 2dy2
> 0.
Vậy giá trị nhỏ nhất u = x2
+ y2
với điều kiện x + y = 1 là 1
2
.
65
Bài toán 54. Tìm giá trị lớn nhất của hàm số u = sin x sin y sin z với điều kiện
x + y + z = π
2
, x, y, z ≥ 0.
Chứng minh. Rõ ràng u liên tục với mọi x, y, z nên nó đạt giá trị lớn nhất m trên miền
D. Lập hàm Lagrange
L(x, y, z, λ) = ln sin x + ln sin y + ln sin z + λ(x + y + z −
π
2
).
Ta thấy 


Lx = cot x + λ = 0
Ly = cot y + λ = 0
Lz = cot z + λ = 0
x + y + z − π
2
= 0
ta tìm được điểm dừng là M(π
6
, π
6
, π
6
) với λ = −
√
3
2
. Tính
d2
L = −(
dx2
sin2
x
+
dy2
sin2
y
+
dz2
sin2
z
) < 0.
Do đó tại điểm (π
6
, π
6
, π
6
) hàm số đạt cực đại có điều kiện là 1
8
. Vậy giá trị lớn nhất
u = sin x sin y sin z với điều kiện x + y + z = π
2
, x, y, z ≥ 0 là 1
8
.
Bài toán 55. Tìm giá trị lớn nhất của hàm số u = xyz với điều kiện x2
+ y2
+ z2
=
1, x + y + z = 0.
Chứng minh. Rõ ràng u liên tục với mọi x, y, z nên nó đạt giá trị lớn nhất m trên miền
D. Lập hàm Lagrange
L(x, y, z, λ) = xyz − λ1(x2
+ y2
+ z2
− 1) − λ2(x + y + z).
Ta thấy 


Lx = yz − 2λ1x − λ2 = 0
Ly = xz − 2λ1y − λ2 = 0
Lz = xy − 2λ1z − λ2 = 0
x2
+ y2
+ z2
= 1
x + y + z = 0
66
ta tìm được các điểm dừng là
M1(
1
√
6
,
1
√
6
, −
2
√
6
), M2(
1
√
6
, −
2
√
6
,
1
√
6
), M3(−
2
√
6
,
1
√
6
,
1
√
6
) với λ2 = −
1
2
√
6
,
M4(−
1
√
6
, −
1
√
6
,
2
√
6
), M5(−
1
√
6
,
2
√
6
, −
1
√
6
), M6(
2
√
6
, −
1
√
6
, −
1
√
6
) với λ2 =
1
2
√
6
.
Tiếp tục tìm vi phân bậc 2 của hàm Lagrange
d2
L = −2λ1(dx2
+ dy2
+ dz2
) + 2zdxdy + 2ydxdz + 2xdydz
trong đó dx, dy, dz liên hệ với nhau bởi 2 hệ thức
xdx + ydy + zdz = 0, dx + dy + dz = 0.
Tại các điểm M1, M4 thì
x = y = −2λ1, z = 4λ1.
Khi đó
xdx + ydy + zdz = −2λ1dx − 2λ1dy + 4λ1dz = 0
hay
dz =
1
2
(dx + dy).
Thay vào biểu thức của d2
L tại M1 ta có
d2
L(M1) =
1
√
6
(dx2
+ dy2
+ dz2
) +
1
√
6
(dx − dy)2
> 0.
Vậy
u(M1) = umin = −
1
3
√
6
,
d2
L(M4) = −
1
√
6
(dx2
+ dy2
+ dz2
) −
1
√
6
(dx − dy)2
< 0.
Vậy
u(M4) = umax =
1
3
√
6
.
Tương tự
u(M5) = u(M6) = umax =
1
3
√
6
, u(M2) = u(M3) = umin = −
1
3
√
6
.
Vậy giá trị nhỏ nhất và lớn nhất của u = xyz với điều kiện x2
+y2
+z2
= 1, x+y+z = 0
lần lượt là − 1
3
√
6
và 1
3
√
6
.
67
Bài toán 56. Tìm giá trị lớn nhất của hàm số u = xy + yz với điều kiện x2
+ y2
=
2, y + z = 2, x, y, z ≥ 0.
Chứng minh. Rõ ràng u liên tục với mọi x, y, z nên nó đạt giá trị lớn nhất m trên miền
D. Lập hàm Lagrange
L(x, y, z, λ1, λ2) = xy + yz + λ1(x2
+ y2
− 2) − λ2(y + z − 2).
Ta thấy 


Lx = y + 2λ1x = 0
Ly = x + z + 2λ1y + λ2 = 0
Lz = y + λ2 = 0
x2
+ y2
= 2
y + z = 2
ta tìm được điểm dừng là M(1, 1, 1) với λ1 = −1
2
, λ2 = −1.
Tiếp tục tìm vi phân bậc 2 của hàm Lagrange
d2
L = 2λ1(dx2
+ dy2
) + 2dxdy + 2dydz
và thay λ1 = −1
2
ta nhận được
d2
L(1, 1, 1, −
1
2
) = −(dx2
+ dy2
) + 2dxdy + 2dydz.
Từ phương trình y + z = 2 ta suy ra dy = −dz và từ 2xdx + 2ydy = 0 với x = y = 1
ta có dx = −dy. Vậy nên
d2
L(1, 1, 1, −
1
2
) = −(dx2
+ dy2
) − 2dy2
− 2dz2
= −dx2
− 3dy2
− 2dz2
< 0.
Vậy u(1, 1, 1) = umax = 2. Vậy giá trị lớn nhất của u = xy + yz với điều kiện
x2
+ y2
= 2, y + z = 2, x, y, z ≥ 0 là 2.
Kết luận
Luận văn đề cập tới nghiên cứu một số phương pháp đạo hàm để tìm giá trị lớn
nhất và giá trị nhỏ nhất của hàm số với ứng dụng vào giải quyết những bài toán khác
nhau. Luận văn đã trình bày các vấn đề sau:
- Phương pháp khảo sát trực tiếp hàm số trên miền xác định
- Phương pháp khảo sát hàm số theo từng biến
- Phương pháp đặt biến phụ
- Phương pháp đánh giá thông qua biểu thức bậc nhất
- Phương pháp sử dụng tính chất của hàm lồi, hàm lõm
- Cực trị tự do của hàm nhiều biến, và cực trị có điều kiện của hàm nhiều biến.
68
Tài liệu tham khảo
[1] Phạm Văn Dũng, Phương pháp sử dụng đạo hàm chứng minh bất đẳng thức.
[2] Phạm Kim Hùng, Sáng tạo bất đẳng thức, NXB tri thức, 2006.
[3] Trần Đức Long, Nguyễn Đình Sang, Hoàng Quốc Toàn, Giáo trình giải tích 1,
NXB ĐHQG Hà Nội, 2004.
[4] Nguyễn Văn Mậu, Bất đẳng thức: Định lý và áp dụng, NXB Giáo dục, 2006.
[5] Trần Phương, Vẻ đẹp Bất đẳng thức trong các kì thi Olympic Toán học, NXB
ĐHQG Hà Nội, 2010.
[6] Trần Phương, Những viên kim cương trong bất đẳng thức Toán học, 2009.
[7] Nguyễn Minh Tuấn, Lý thuyết cơ sở của hàm lồi và các bất đẳng thức cổ điển,
NXB ĐHQG Hà Nội, 2013.
69

More Related Content

What's hot

Giaitichcoso(PGS.TS.NguyenBichHuy).pdf
Giaitichcoso(PGS.TS.NguyenBichHuy).pdfGiaitichcoso(PGS.TS.NguyenBichHuy).pdf
Giaitichcoso(PGS.TS.NguyenBichHuy).pdfBui Loi
 
Luận văn: Hàm số mũ trong dạy học vật lý ở trung học phổ thông, HAY
Luận văn: Hàm số mũ trong dạy học vật lý ở trung học phổ thông, HAYLuận văn: Hàm số mũ trong dạy học vật lý ở trung học phổ thông, HAY
Luận văn: Hàm số mũ trong dạy học vật lý ở trung học phổ thông, HAYViết thuê trọn gói ZALO 0934573149
 
kỹ thuật giải phương trình hàm
kỹ thuật giải phương trình hàmkỹ thuật giải phương trình hàm
kỹ thuật giải phương trình hàmljmonking
 
Phương trình năng lượng tổng quát - Trịnh Văn Quang
Phương trình năng lượng tổng quát - Trịnh Văn Quang Phương trình năng lượng tổng quát - Trịnh Văn Quang
Phương trình năng lượng tổng quát - Trịnh Văn Quang Trinh Van Quang
 
Gt khong gian_metric Nguyen Hoang
Gt khong gian_metric Nguyen HoangGt khong gian_metric Nguyen Hoang
Gt khong gian_metric Nguyen HoangBui Loi
 
Một số vấn đề về không gian Sobolev
Một số vấn đề về không gian SobolevMột số vấn đề về không gian Sobolev
Một số vấn đề về không gian Sobolevnataliej4
 
Topo daicuong1[1]
Topo daicuong1[1]Topo daicuong1[1]
Topo daicuong1[1]Bui Loi
 
Bài Giảng Đại Số Tuyến Tính - ĐH Thăng Long
Bài Giảng Đại Số Tuyến Tính - ĐH Thăng LongBài Giảng Đại Số Tuyến Tính - ĐH Thăng Long
Bài Giảng Đại Số Tuyến Tính - ĐH Thăng LongHoàng Như Mộc Miên
 
Luận văn: Dạy học phân hoá qua tổ chức ôn tập một số chủ đề phương trình, bất...
Luận văn: Dạy học phân hoá qua tổ chức ôn tập một số chủ đề phương trình, bất...Luận văn: Dạy học phân hoá qua tổ chức ôn tập một số chủ đề phương trình, bất...
Luận văn: Dạy học phân hoá qua tổ chức ôn tập một số chủ đề phương trình, bất...Viết thuê trọn gói ZALO 0934573149
 

What's hot (20)

Luận văn: Một số lớp bài toán về phương trình hàm, HAY, 9đ
Luận văn: Một số lớp bài toán về phương trình hàm, HAY, 9đLuận văn: Một số lớp bài toán về phương trình hàm, HAY, 9đ
Luận văn: Một số lớp bài toán về phương trình hàm, HAY, 9đ
 
Giaitichcoso(PGS.TS.NguyenBichHuy).pdf
Giaitichcoso(PGS.TS.NguyenBichHuy).pdfGiaitichcoso(PGS.TS.NguyenBichHuy).pdf
Giaitichcoso(PGS.TS.NguyenBichHuy).pdf
 
Bt dai so hoang
Bt dai so hoangBt dai so hoang
Bt dai so hoang
 
Luận văn: Nghiên cứu didactic về khái niệm bất đẳng thức, HAY
Luận văn: Nghiên cứu didactic về khái niệm bất đẳng thức, HAYLuận văn: Nghiên cứu didactic về khái niệm bất đẳng thức, HAY
Luận văn: Nghiên cứu didactic về khái niệm bất đẳng thức, HAY
 
Luận văn: Lý thuyết đồ thị với các bài toán phổ thông, HAY, 9đ
Luận văn: Lý thuyết đồ thị với các bài toán phổ thông, HAY, 9đLuận văn: Lý thuyết đồ thị với các bài toán phổ thông, HAY, 9đ
Luận văn: Lý thuyết đồ thị với các bài toán phổ thông, HAY, 9đ
 
Bài tập hàm biến phức
Bài tập hàm biến phứcBài tập hàm biến phức
Bài tập hàm biến phức
 
Luận văn: Hàm số mũ trong dạy học vật lý ở trung học phổ thông, HAY
Luận văn: Hàm số mũ trong dạy học vật lý ở trung học phổ thông, HAYLuận văn: Hàm số mũ trong dạy học vật lý ở trung học phổ thông, HAY
Luận văn: Hàm số mũ trong dạy học vật lý ở trung học phổ thông, HAY
 
kỹ thuật giải phương trình hàm
kỹ thuật giải phương trình hàmkỹ thuật giải phương trình hàm
kỹ thuật giải phương trình hàm
 
Phương trình năng lượng tổng quát - Trịnh Văn Quang
Phương trình năng lượng tổng quát - Trịnh Văn Quang Phương trình năng lượng tổng quát - Trịnh Văn Quang
Phương trình năng lượng tổng quát - Trịnh Văn Quang
 
Đề tài: Dạy học tích hợp chủ đề hàm số bậc hai, HAY
Đề tài: Dạy học tích hợp chủ đề hàm số bậc hai, HAYĐề tài: Dạy học tích hợp chủ đề hàm số bậc hai, HAY
Đề tài: Dạy học tích hợp chủ đề hàm số bậc hai, HAY
 
Gt khong gian_metric Nguyen Hoang
Gt khong gian_metric Nguyen HoangGt khong gian_metric Nguyen Hoang
Gt khong gian_metric Nguyen Hoang
 
Bài mẫu Khóa luận tốt nghiệp ngành sư phạm toán, HAY, 9 ĐIỂM
Bài mẫu Khóa luận tốt nghiệp ngành sư phạm toán, HAY, 9 ĐIỂMBài mẫu Khóa luận tốt nghiệp ngành sư phạm toán, HAY, 9 ĐIỂM
Bài mẫu Khóa luận tốt nghiệp ngành sư phạm toán, HAY, 9 ĐIỂM
 
Một số vấn đề về không gian Sobolev
Một số vấn đề về không gian SobolevMột số vấn đề về không gian Sobolev
Một số vấn đề về không gian Sobolev
 
Topo daicuong1[1]
Topo daicuong1[1]Topo daicuong1[1]
Topo daicuong1[1]
 
BÀI MẪU Khóa luận tốt nghiệp ngành sư phạm toán, HAY
BÀI MẪU Khóa luận tốt nghiệp ngành sư phạm toán, HAYBÀI MẪU Khóa luận tốt nghiệp ngành sư phạm toán, HAY
BÀI MẪU Khóa luận tốt nghiệp ngành sư phạm toán, HAY
 
Luận văn: Giải bài toán Dirichlet đối với phương trình Elliptic, 9đ
Luận văn: Giải bài toán Dirichlet đối với phương trình Elliptic, 9đLuận văn: Giải bài toán Dirichlet đối với phương trình Elliptic, 9đ
Luận văn: Giải bài toán Dirichlet đối với phương trình Elliptic, 9đ
 
Bài Giảng Đại Số Tuyến Tính - ĐH Thăng Long
Bài Giảng Đại Số Tuyến Tính - ĐH Thăng LongBài Giảng Đại Số Tuyến Tính - ĐH Thăng Long
Bài Giảng Đại Số Tuyến Tính - ĐH Thăng Long
 
Luận văn: Dạy học phân hoá qua tổ chức ôn tập một số chủ đề phương trình, bất...
Luận văn: Dạy học phân hoá qua tổ chức ôn tập một số chủ đề phương trình, bất...Luận văn: Dạy học phân hoá qua tổ chức ôn tập một số chủ đề phương trình, bất...
Luận văn: Dạy học phân hoá qua tổ chức ôn tập một số chủ đề phương trình, bất...
 
Đề tài: Bài toán phương trình đạo hàm riêng dạng elliptic, HAY
Đề tài: Bài toán phương trình đạo hàm riêng dạng elliptic, HAYĐề tài: Bài toán phương trình đạo hàm riêng dạng elliptic, HAY
Đề tài: Bài toán phương trình đạo hàm riêng dạng elliptic, HAY
 
Luận văn: Giải số phương trình vi phân đại số bằng đa bước, 9đ
Luận văn: Giải số phương trình vi phân đại số bằng đa bước, 9đLuận văn: Giải số phương trình vi phân đại số bằng đa bước, 9đ
Luận văn: Giải số phương trình vi phân đại số bằng đa bước, 9đ
 

Similar to Luận văn: Lớp bài toán tìm giá trị lớn nhất giá trị nhỏ nhất, HOT

06 mat101 bai2_v2.3013101225
06 mat101 bai2_v2.301310122506 mat101 bai2_v2.3013101225
06 mat101 bai2_v2.3013101225Yen Dang
 
Luận văn: Điểm bất động của một số lớp ánh xạ đa trị, HAY
Luận văn: Điểm bất động của một số lớp ánh xạ đa trị, HAYLuận văn: Điểm bất động của một số lớp ánh xạ đa trị, HAY
Luận văn: Điểm bất động của một số lớp ánh xạ đa trị, HAYViết thuê trọn gói ZALO 0934573149
 
Vận dụng giới hạn dãy số trong giải phương trình hàm.pdf
Vận dụng giới hạn dãy số trong giải phương trình hàm.pdfVận dụng giới hạn dãy số trong giải phương trình hàm.pdf
Vận dụng giới hạn dãy số trong giải phương trình hàm.pdfvongoccuong
 
Một số dạng toán về đa thức qua các kỳ thi Olympic 6732069.pdf
Một số dạng toán về đa thức qua các kỳ thi Olympic 6732069.pdfMột số dạng toán về đa thức qua các kỳ thi Olympic 6732069.pdf
Một số dạng toán về đa thức qua các kỳ thi Olympic 6732069.pdfTieuNgocLy
 
Luận văn: Một số lớp bài toán về loại phương trình hàm, HAY - Gửi miễn phí qu...
Luận văn: Một số lớp bài toán về loại phương trình hàm, HAY - Gửi miễn phí qu...Luận văn: Một số lớp bài toán về loại phương trình hàm, HAY - Gửi miễn phí qu...
Luận văn: Một số lớp bài toán về loại phương trình hàm, HAY - Gửi miễn phí qu...Dịch vụ viết bài trọn gói ZALO: 0909232620
 
Bài giảng chi tiết giải tích 1 07 2013 bo môn toán
Bài giảng chi tiết giải tích 1  07 2013 bo môn toánBài giảng chi tiết giải tích 1  07 2013 bo môn toán
Bài giảng chi tiết giải tích 1 07 2013 bo môn toánLaurent Koscielny
 
05 mat101 bai1_v2.3013101225
 05 mat101 bai1_v2.3013101225 05 mat101 bai1_v2.3013101225
05 mat101 bai1_v2.3013101225Yen Dang
 
Tổng hợp công thức giải nhanh trắc nghiệm toán THPT Quốc gia 2018
Tổng hợp công thức giải nhanh trắc nghiệm toán THPT Quốc gia 2018Tổng hợp công thức giải nhanh trắc nghiệm toán THPT Quốc gia 2018
Tổng hợp công thức giải nhanh trắc nghiệm toán THPT Quốc gia 2018Maloda
 
Hàm số - 5. Cực trị hàm số
Hàm số - 5. Cực trị hàm sốHàm số - 5. Cực trị hàm số
Hàm số - 5. Cực trị hàm sốlovestem
 

Similar to Luận văn: Lớp bài toán tìm giá trị lớn nhất giá trị nhỏ nhất, HOT (20)

06 mat101 bai2_v2.3013101225
06 mat101 bai2_v2.301310122506 mat101 bai2_v2.3013101225
06 mat101 bai2_v2.3013101225
 
Luận văn: Phương pháp giải bài toán cực trị và ứng dụng, HAY
Luận văn: Phương pháp giải bài toán cực trị và ứng dụng, HAYLuận văn: Phương pháp giải bài toán cực trị và ứng dụng, HAY
Luận văn: Phương pháp giải bài toán cực trị và ứng dụng, HAY
 
Luận văn: Phương pháp giải bài toán cực trị, HAY
Luận văn: Phương pháp giải bài toán cực trị, HAYLuận văn: Phương pháp giải bài toán cực trị, HAY
Luận văn: Phương pháp giải bài toán cực trị, HAY
 
Luận văn: Bất đẳng thức trong lớp hàm siêu việt, HAY, 9đ
Luận văn: Bất đẳng thức trong lớp hàm siêu việt, HAY, 9đLuận văn: Bất đẳng thức trong lớp hàm siêu việt, HAY, 9đ
Luận văn: Bất đẳng thức trong lớp hàm siêu việt, HAY, 9đ
 
Luận văn: Điểm bất động của một số lớp ánh xạ đa trị, HAY
Luận văn: Điểm bất động của một số lớp ánh xạ đa trị, HAYLuận văn: Điểm bất động của một số lớp ánh xạ đa trị, HAY
Luận văn: Điểm bất động của một số lớp ánh xạ đa trị, HAY
 
Luận văn: Tính chất định tính của hàm đa trị và ứng dụng, HOT
Luận văn: Tính chất định tính của hàm đa trị và ứng dụng, HOTLuận văn: Tính chất định tính của hàm đa trị và ứng dụng, HOT
Luận văn: Tính chất định tính của hàm đa trị và ứng dụng, HOT
 
Đề tài: Tính ổn định của lớp phương trình hàm với cặp biến tự do
Đề tài: Tính ổn định của lớp phương trình hàm với cặp biến tự doĐề tài: Tính ổn định của lớp phương trình hàm với cặp biến tự do
Đề tài: Tính ổn định của lớp phương trình hàm với cặp biến tự do
 
Luận văn: Một số phương pháp giải phương trình hàm, HOT, 9đ
Luận văn: Một số phương pháp giải phương trình hàm, HOT, 9đLuận văn: Một số phương pháp giải phương trình hàm, HOT, 9đ
Luận văn: Một số phương pháp giải phương trình hàm, HOT, 9đ
 
Ứng Dụng Công Thức Viète Trong Giải Toán Bậc Phổ Thông.doc
Ứng Dụng Công Thức Viète Trong Giải Toán Bậc Phổ Thông.docỨng Dụng Công Thức Viète Trong Giải Toán Bậc Phổ Thông.doc
Ứng Dụng Công Thức Viète Trong Giải Toán Bậc Phổ Thông.doc
 
Vận dụng giới hạn dãy số trong giải phương trình hàm.pdf
Vận dụng giới hạn dãy số trong giải phương trình hàm.pdfVận dụng giới hạn dãy số trong giải phương trình hàm.pdf
Vận dụng giới hạn dãy số trong giải phương trình hàm.pdf
 
Một số dạng toán về đa thức qua các kỳ thi Olympic 6732069.pdf
Một số dạng toán về đa thức qua các kỳ thi Olympic 6732069.pdfMột số dạng toán về đa thức qua các kỳ thi Olympic 6732069.pdf
Một số dạng toán về đa thức qua các kỳ thi Olympic 6732069.pdf
 
Luận văn: Phép biến đổi phân tuyến tính, HAY, 9đ
Luận văn: Phép biến đổi phân tuyến tính, HAY, 9đLuận văn: Phép biến đổi phân tuyến tính, HAY, 9đ
Luận văn: Phép biến đổi phân tuyến tính, HAY, 9đ
 
Luận văn: Một số lớp bài toán về loại phương trình hàm, HAY - Gửi miễn phí qu...
Luận văn: Một số lớp bài toán về loại phương trình hàm, HAY - Gửi miễn phí qu...Luận văn: Một số lớp bài toán về loại phương trình hàm, HAY - Gửi miễn phí qu...
Luận văn: Một số lớp bài toán về loại phương trình hàm, HAY - Gửi miễn phí qu...
 
Luận văn: Ứng dụng của định lý minimax, HAY, 9đ
Luận văn: Ứng dụng của định lý minimax, HAY, 9đLuận văn: Ứng dụng của định lý minimax, HAY, 9đ
Luận văn: Ứng dụng của định lý minimax, HAY, 9đ
 
Luận văn: phương pháp nghiên cứu sự phân nhánh, HAY, 9đ
Luận văn: phương pháp nghiên cứu sự phân nhánh, HAY, 9đLuận văn: phương pháp nghiên cứu sự phân nhánh, HAY, 9đ
Luận văn: phương pháp nghiên cứu sự phân nhánh, HAY, 9đ
 
Đang thức, bat đang thức tích phân trong l p đa thức và phân thức hữu ty và m...
Đang thức, bat đang thức tích phân trong l p đa thức và phân thức hữu ty và m...Đang thức, bat đang thức tích phân trong l p đa thức và phân thức hữu ty và m...
Đang thức, bat đang thức tích phân trong l p đa thức và phân thức hữu ty và m...
 
Bài giảng chi tiết giải tích 1 07 2013 bo môn toán
Bài giảng chi tiết giải tích 1  07 2013 bo môn toánBài giảng chi tiết giải tích 1  07 2013 bo môn toán
Bài giảng chi tiết giải tích 1 07 2013 bo môn toán
 
05 mat101 bai1_v2.3013101225
 05 mat101 bai1_v2.3013101225 05 mat101 bai1_v2.3013101225
05 mat101 bai1_v2.3013101225
 
Tổng hợp công thức giải nhanh trắc nghiệm toán THPT Quốc gia 2018
Tổng hợp công thức giải nhanh trắc nghiệm toán THPT Quốc gia 2018Tổng hợp công thức giải nhanh trắc nghiệm toán THPT Quốc gia 2018
Tổng hợp công thức giải nhanh trắc nghiệm toán THPT Quốc gia 2018
 
Hàm số - 5. Cực trị hàm số
Hàm số - 5. Cực trị hàm sốHàm số - 5. Cực trị hàm số
Hàm số - 5. Cực trị hàm số
 

More from Dịch vụ viết bài trọn gói ZALO: 0909232620

Danh Sách 200 Đề Tài Tiểu Luận Chuyên Viên Chính Về Bảo Hiểm Xã Hội Mới Nhất
Danh Sách 200 Đề Tài Tiểu Luận Chuyên Viên Chính Về Bảo Hiểm Xã Hội Mới NhấtDanh Sách 200 Đề Tài Tiểu Luận Chuyên Viên Chính Về Bảo Hiểm Xã Hội Mới Nhất
Danh Sách 200 Đề Tài Tiểu Luận Chuyên Viên Chính Về Bảo Hiểm Xã Hội Mới NhấtDịch vụ viết bài trọn gói ZALO: 0909232620
 
Danh Sách 200 Đề Tài Báo Cáo Thực Tập Luật Phòng, Chống Hiv, Mới Nhất, Điểm Cao
Danh Sách 200 Đề Tài Báo Cáo Thực Tập Luật Phòng, Chống Hiv, Mới Nhất, Điểm CaoDanh Sách 200 Đề Tài Báo Cáo Thực Tập Luật Phòng, Chống Hiv, Mới Nhất, Điểm Cao
Danh Sách 200 Đề Tài Báo Cáo Thực Tập Luật Phòng, Chống Hiv, Mới Nhất, Điểm CaoDịch vụ viết bài trọn gói ZALO: 0909232620
 

More from Dịch vụ viết bài trọn gói ZALO: 0909232620 (20)

Danh Sách 200 Đề Tài Tiểu Luận Chuyên Viên Chính Về Bảo Hiểm Xã Hội Mới Nhất
Danh Sách 200 Đề Tài Tiểu Luận Chuyên Viên Chính Về Bảo Hiểm Xã Hội Mới NhấtDanh Sách 200 Đề Tài Tiểu Luận Chuyên Viên Chính Về Bảo Hiểm Xã Hội Mới Nhất
Danh Sách 200 Đề Tài Tiểu Luận Chuyên Viên Chính Về Bảo Hiểm Xã Hội Mới Nhất
 
Danh Sách 200 Đề Tài Luận Văn Thạc Sĩ Quản Trị Nguồn Nhân Lực, 9 Điểm
Danh Sách 200 Đề Tài Luận Văn Thạc Sĩ Quản Trị Nguồn Nhân Lực, 9 ĐiểmDanh Sách 200 Đề Tài Luận Văn Thạc Sĩ Quản Trị Nguồn Nhân Lực, 9 Điểm
Danh Sách 200 Đề Tài Luận Văn Thạc Sĩ Quản Trị Nguồn Nhân Lực, 9 Điểm
 
Danh Sách 200 Đề Tài Luận Văn Thạc Sĩ Quản Lý Văn Hóa Giúp Bạn Thêm Ý Tưởng
Danh Sách 200 Đề Tài Luận Văn Thạc Sĩ Quản Lý Văn Hóa Giúp Bạn Thêm Ý TưởngDanh Sách 200 Đề Tài Luận Văn Thạc Sĩ Quản Lý Văn Hóa Giúp Bạn Thêm Ý Tưởng
Danh Sách 200 Đề Tài Luận Văn Thạc Sĩ Quản Lý Văn Hóa Giúp Bạn Thêm Ý Tưởng
 
Danh Sách 200 Đề Tài Báo Cáo Thực Tập Quản Lý Giáo Dục Dễ Làm Điểm Cao
Danh Sách 200 Đề Tài Báo Cáo Thực Tập Quản Lý Giáo Dục Dễ Làm Điểm CaoDanh Sách 200 Đề Tài Báo Cáo Thực Tập Quản Lý Giáo Dục Dễ Làm Điểm Cao
Danh Sách 200 Đề Tài Báo Cáo Thực Tập Quản Lý Giáo Dục Dễ Làm Điểm Cao
 
Danh Sách 200 Đề Tài Báo Cáo Thực Tập Quan Hệ Lao Động Từ Sinh Viên Giỏi
Danh Sách 200 Đề Tài Báo Cáo Thực Tập Quan Hệ Lao Động Từ Sinh Viên GiỏiDanh Sách 200 Đề Tài Báo Cáo Thực Tập Quan Hệ Lao Động Từ Sinh Viên Giỏi
Danh Sách 200 Đề Tài Báo Cáo Thực Tập Quan Hệ Lao Động Từ Sinh Viên Giỏi
 
Danh Sách 200 Đề Tài Báo Cáo Thực Tập Nuôi Trồng Thủy Sản Dễ Làm Nhất
Danh Sách 200 Đề Tài Báo Cáo Thực Tập Nuôi Trồng Thủy Sản Dễ Làm NhấtDanh Sách 200 Đề Tài Báo Cáo Thực Tập Nuôi Trồng Thủy Sản Dễ Làm Nhất
Danh Sách 200 Đề Tài Báo Cáo Thực Tập Nuôi Trồng Thủy Sản Dễ Làm Nhất
 
Danh Sách 200 Đề Tài Báo Cáo Thực Tập Luật Sư, Mới Nhất, Điểm Cao
Danh Sách 200 Đề Tài Báo Cáo Thực Tập Luật Sư, Mới Nhất, Điểm CaoDanh Sách 200 Đề Tài Báo Cáo Thực Tập Luật Sư, Mới Nhất, Điểm Cao
Danh Sách 200 Đề Tài Báo Cáo Thực Tập Luật Sư, Mới Nhất, Điểm Cao
 
Danh Sách 200 Đề Tài Báo Cáo Thực Tập Luật Phòng, Chống Hiv, Mới Nhất, Điểm Cao
Danh Sách 200 Đề Tài Báo Cáo Thực Tập Luật Phòng, Chống Hiv, Mới Nhất, Điểm CaoDanh Sách 200 Đề Tài Báo Cáo Thực Tập Luật Phòng, Chống Hiv, Mới Nhất, Điểm Cao
Danh Sách 200 Đề Tài Báo Cáo Thực Tập Luật Phòng, Chống Hiv, Mới Nhất, Điểm Cao
 
Danh Sách 200 Đề Tài Báo Cáo Thực Tập Luật Phá Sản, Mới Nhất
Danh Sách 200 Đề Tài Báo Cáo Thực Tập Luật Phá Sản, Mới NhấtDanh Sách 200 Đề Tài Báo Cáo Thực Tập Luật Phá Sản, Mới Nhất
Danh Sách 200 Đề Tài Báo Cáo Thực Tập Luật Phá Sản, Mới Nhất
 
Danh Sách 200 Đề Tài Báo Cáo Thực Tập Luật Nhà Ở, Điểm Cao
Danh Sách 200 Đề Tài Báo Cáo Thực Tập Luật Nhà Ở, Điểm CaoDanh Sách 200 Đề Tài Báo Cáo Thực Tập Luật Nhà Ở, Điểm Cao
Danh Sách 200 Đề Tài Báo Cáo Thực Tập Luật Nhà Ở, Điểm Cao
 
Danh Sách 200 Đề Tài Báo Cáo Thực Tập Luật Ngân Hàng, Mới Nhất
Danh Sách 200 Đề Tài Báo Cáo Thực Tập Luật Ngân Hàng, Mới NhấtDanh Sách 200 Đề Tài Báo Cáo Thực Tập Luật Ngân Hàng, Mới Nhất
Danh Sách 200 Đề Tài Báo Cáo Thực Tập Luật Ngân Hàng, Mới Nhất
 
Danh Sách 200 Đề Tài Báo Cáo Thực Tập Luật Môi Trường, Mới Nhất
Danh Sách 200 Đề Tài Báo Cáo Thực Tập Luật Môi Trường, Mới NhấtDanh Sách 200 Đề Tài Báo Cáo Thực Tập Luật Môi Trường, Mới Nhất
Danh Sách 200 Đề Tài Báo Cáo Thực Tập Luật Môi Trường, Mới Nhất
 
Danh Sách 200 Đề Tài Báo Cáo Thực Tập Luật Hộ Tịch, Điểm Cao
Danh Sách 200 Đề Tài Báo Cáo Thực Tập Luật Hộ Tịch, Điểm CaoDanh Sách 200 Đề Tài Báo Cáo Thực Tập Luật Hộ Tịch, Điểm Cao
Danh Sách 200 Đề Tài Báo Cáo Thực Tập Luật Hộ Tịch, Điểm Cao
 
Danh Sách 200 Đề Tài Báo Cáo Thực Tập Luật Hình Sự , Dễ Làm Điểm Cao
Danh Sách 200 Đề Tài Báo Cáo Thực Tập Luật Hình Sự , Dễ Làm Điểm CaoDanh Sách 200 Đề Tài Báo Cáo Thực Tập Luật Hình Sự , Dễ Làm Điểm Cao
Danh Sách 200 Đề Tài Báo Cáo Thực Tập Luật Hình Sự , Dễ Làm Điểm Cao
 
Danh Sách 200 Đề Tài Báo Cáo Thực Tập Luật Hành Chính, Dễ Làm Điểm Cao
Danh Sách 200 Đề Tài Báo Cáo Thực Tập Luật Hành Chính, Dễ Làm Điểm CaoDanh Sách 200 Đề Tài Báo Cáo Thực Tập Luật Hành Chính, Dễ Làm Điểm Cao
Danh Sách 200 Đề Tài Báo Cáo Thực Tập Luật Hành Chính, Dễ Làm Điểm Cao
 
Danh Sách 200 Đề Tài Báo Cáo Thực Tập Luật Giáo Dục, Điểm Cao
Danh Sách 200 Đề Tài Báo Cáo Thực Tập Luật Giáo Dục, Điểm CaoDanh Sách 200 Đề Tài Báo Cáo Thực Tập Luật Giáo Dục, Điểm Cao
Danh Sách 200 Đề Tài Báo Cáo Thực Tập Luật Giáo Dục, Điểm Cao
 
Danh Sách 200 Đề Tài Báo Cáo Thực Tập Luật Đấu Thầu, Từ Sinh Viên Khá Giỏi
Danh Sách 200 Đề Tài Báo Cáo Thực Tập Luật Đấu Thầu, Từ Sinh Viên Khá GiỏiDanh Sách 200 Đề Tài Báo Cáo Thực Tập Luật Đấu Thầu, Từ Sinh Viên Khá Giỏi
Danh Sách 200 Đề Tài Báo Cáo Thực Tập Luật Đấu Thầu, Từ Sinh Viên Khá Giỏi
 
Danh Sách 200 Đề Tài Báo Cáo Thực Tập Luật Đầu Tư, Dễ Làm Điểm Cao
Danh Sách 200 Đề Tài Báo Cáo Thực Tập Luật Đầu Tư, Dễ Làm Điểm CaoDanh Sách 200 Đề Tài Báo Cáo Thực Tập Luật Đầu Tư, Dễ Làm Điểm Cao
Danh Sách 200 Đề Tài Báo Cáo Thực Tập Luật Đầu Tư, Dễ Làm Điểm Cao
 
Danh Sách 200 Đề Tài Báo Cáo Thực Tập Luật Đầu Tư Công, Dễ Làm Điểm Cao
Danh Sách 200 Đề Tài Báo Cáo Thực Tập Luật Đầu Tư Công, Dễ Làm Điểm CaoDanh Sách 200 Đề Tài Báo Cáo Thực Tập Luật Đầu Tư Công, Dễ Làm Điểm Cao
Danh Sách 200 Đề Tài Báo Cáo Thực Tập Luật Đầu Tư Công, Dễ Làm Điểm Cao
 
Danh Sách 200 Đề Tài Báo Cáo Thực Tập Luật Đất Đai, Từ Sinh Viên Khá Giỏi
Danh Sách 200 Đề Tài Báo Cáo Thực Tập Luật Đất Đai, Từ Sinh Viên Khá GiỏiDanh Sách 200 Đề Tài Báo Cáo Thực Tập Luật Đất Đai, Từ Sinh Viên Khá Giỏi
Danh Sách 200 Đề Tài Báo Cáo Thực Tập Luật Đất Đai, Từ Sinh Viên Khá Giỏi
 

Recently uploaded

Sáng kiến Dạy học theo định hướng STEM một số chủ đề phần “vật sống”, Khoa họ...
Sáng kiến Dạy học theo định hướng STEM một số chủ đề phần “vật sống”, Khoa họ...Sáng kiến Dạy học theo định hướng STEM một số chủ đề phần “vật sống”, Khoa họ...
Sáng kiến Dạy học theo định hướng STEM một số chủ đề phần “vật sống”, Khoa họ...Nguyen Thanh Tu Collection
 
[GIẢI PHẪU BỆNH] Tổn thương cơ bản của tb bào mô
[GIẢI PHẪU BỆNH] Tổn thương cơ bản của tb bào mô[GIẢI PHẪU BỆNH] Tổn thương cơ bản của tb bào mô
[GIẢI PHẪU BỆNH] Tổn thương cơ bản của tb bào môBryan Williams
 
Ma trận - định thức và các ứng dụng trong kinh tế
Ma trận - định thức và các ứng dụng trong kinh tếMa trận - định thức và các ứng dụng trong kinh tế
Ma trận - định thức và các ứng dụng trong kinh tếngTonH1
 
Sáng kiến “Sử dụng ứng dụng Quizizz nhằm nâng cao chất lượng ôn thi tốt nghiệ...
Sáng kiến “Sử dụng ứng dụng Quizizz nhằm nâng cao chất lượng ôn thi tốt nghiệ...Sáng kiến “Sử dụng ứng dụng Quizizz nhằm nâng cao chất lượng ôn thi tốt nghiệ...
Sáng kiến “Sử dụng ứng dụng Quizizz nhằm nâng cao chất lượng ôn thi tốt nghiệ...Nguyen Thanh Tu Collection
 
bài 5.1.docx Sinh học di truyền đại cương năm nhất của học sinh y đa khoa
bài 5.1.docx Sinh học di truyền đại cương năm nhất của học sinh y đa khoabài 5.1.docx Sinh học di truyền đại cương năm nhất của học sinh y đa khoa
bài 5.1.docx Sinh học di truyền đại cương năm nhất của học sinh y đa khoa2353020138
 
SÁNG KIẾN “THIẾT KẾ VÀ SỬ DỤNG INFOGRAPHIC TRONG DẠY HỌC ĐỊA LÍ 11 (BỘ SÁCH K...
SÁNG KIẾN “THIẾT KẾ VÀ SỬ DỤNG INFOGRAPHIC TRONG DẠY HỌC ĐỊA LÍ 11 (BỘ SÁCH K...SÁNG KIẾN “THIẾT KẾ VÀ SỬ DỤNG INFOGRAPHIC TRONG DẠY HỌC ĐỊA LÍ 11 (BỘ SÁCH K...
SÁNG KIẾN “THIẾT KẾ VÀ SỬ DỤNG INFOGRAPHIC TRONG DẠY HỌC ĐỊA LÍ 11 (BỘ SÁCH K...Nguyen Thanh Tu Collection
 
Tư tưởng Hồ Chí Minh về độc lập dân tộc và CNXH
Tư tưởng Hồ Chí Minh về độc lập dân tộc và CNXHTư tưởng Hồ Chí Minh về độc lập dân tộc và CNXH
Tư tưởng Hồ Chí Minh về độc lập dân tộc và CNXHThaoPhuong154017
 
Chàm - Bệnh án (da liễu - bvdlct ctump) .pptx
Chàm - Bệnh án (da liễu - bvdlct ctump) .pptxChàm - Bệnh án (da liễu - bvdlct ctump) .pptx
Chàm - Bệnh án (da liễu - bvdlct ctump) .pptxendkay31
 
ĐỀ THAM KHẢO THEO HƯỚNG MINH HỌA 2025 KIỂM TRA CUỐI HỌC KÌ 2 NĂM HỌC 2023-202...
ĐỀ THAM KHẢO THEO HƯỚNG MINH HỌA 2025 KIỂM TRA CUỐI HỌC KÌ 2 NĂM HỌC 2023-202...ĐỀ THAM KHẢO THEO HƯỚNG MINH HỌA 2025 KIỂM TRA CUỐI HỌC KÌ 2 NĂM HỌC 2023-202...
ĐỀ THAM KHẢO THEO HƯỚNG MINH HỌA 2025 KIỂM TRA CUỐI HỌC KÌ 2 NĂM HỌC 2023-202...Nguyen Thanh Tu Collection
 
10 ĐỀ KIỂM TRA + 6 ĐỀ ÔN TẬP CUỐI KÌ 2 VẬT LÝ 11 - KẾT NỐI TRI THỨC - THEO C...
10 ĐỀ KIỂM TRA + 6 ĐỀ ÔN TẬP CUỐI KÌ 2 VẬT LÝ 11 - KẾT NỐI TRI THỨC - THEO C...10 ĐỀ KIỂM TRA + 6 ĐỀ ÔN TẬP CUỐI KÌ 2 VẬT LÝ 11 - KẾT NỐI TRI THỨC - THEO C...
10 ĐỀ KIỂM TRA + 6 ĐỀ ÔN TẬP CUỐI KÌ 2 VẬT LÝ 11 - KẾT NỐI TRI THỨC - THEO C...Nguyen Thanh Tu Collection
 
50 ĐỀ ĐỀ XUẤT THI VÀO 10 THPT SỞ GIÁO DỤC THANH HÓA MÔN TIẾNG ANH 9 CÓ TỰ LUẬ...
50 ĐỀ ĐỀ XUẤT THI VÀO 10 THPT SỞ GIÁO DỤC THANH HÓA MÔN TIẾNG ANH 9 CÓ TỰ LUẬ...50 ĐỀ ĐỀ XUẤT THI VÀO 10 THPT SỞ GIÁO DỤC THANH HÓA MÔN TIẾNG ANH 9 CÓ TỰ LUẬ...
50 ĐỀ ĐỀ XUẤT THI VÀO 10 THPT SỞ GIÁO DỤC THANH HÓA MÔN TIẾNG ANH 9 CÓ TỰ LUẬ...Nguyen Thanh Tu Collection
 
Slide Webinar Hướng dẫn sử dụng ChatGPT cho người mới bắt đầ...
Slide Webinar Hướng dẫn sử dụng ChatGPT cho người mới bắt đầ...Slide Webinar Hướng dẫn sử dụng ChatGPT cho người mới bắt đầ...
Slide Webinar Hướng dẫn sử dụng ChatGPT cho người mới bắt đầ...Học viện Kstudy
 
TỔNG HỢP ĐỀ THI CHÍNH THỨC KỲ THI TUYỂN SINH VÀO LỚP 10 THPT MÔN NGỮ VĂN NĂM ...
TỔNG HỢP ĐỀ THI CHÍNH THỨC KỲ THI TUYỂN SINH VÀO LỚP 10 THPT MÔN NGỮ VĂN NĂM ...TỔNG HỢP ĐỀ THI CHÍNH THỨC KỲ THI TUYỂN SINH VÀO LỚP 10 THPT MÔN NGỮ VĂN NĂM ...
TỔNG HỢP ĐỀ THI CHÍNH THỨC KỲ THI TUYỂN SINH VÀO LỚP 10 THPT MÔN NGỮ VĂN NĂM ...Nguyen Thanh Tu Collection
 
BỘ ĐỀ KIỂM TRA CUỐI KÌ 2 VẬT LÝ 11 - KẾT NỐI TRI THỨC - THEO CẤU TRÚC ĐỀ MIN...
BỘ ĐỀ KIỂM TRA CUỐI KÌ 2 VẬT LÝ 11 - KẾT NỐI TRI THỨC - THEO CẤU TRÚC ĐỀ MIN...BỘ ĐỀ KIỂM TRA CUỐI KÌ 2 VẬT LÝ 11 - KẾT NỐI TRI THỨC - THEO CẤU TRÚC ĐỀ MIN...
BỘ ĐỀ KIỂM TRA CUỐI KÌ 2 VẬT LÝ 11 - KẾT NỐI TRI THỨC - THEO CẤU TRÚC ĐỀ MIN...Nguyen Thanh Tu Collection
 
cuộc cải cách của Lê Thánh Tông - Sử 11
cuộc cải cách của Lê Thánh Tông -  Sử 11cuộc cải cách của Lê Thánh Tông -  Sử 11
cuộc cải cách của Lê Thánh Tông - Sử 11zedgaming208
 
Bài giảng về vật liệu ceramic ( sứ vệ sinh, gạch ốp lát )
Bài giảng về vật liệu ceramic ( sứ vệ sinh, gạch ốp lát )Bài giảng về vật liệu ceramic ( sứ vệ sinh, gạch ốp lát )
Bài giảng về vật liệu ceramic ( sứ vệ sinh, gạch ốp lát )lamdapoet123
 
CHƯƠNG VII LUẬT DÂN SỰ (2) Pháp luật đại cương.pptx
CHƯƠNG VII LUẬT DÂN SỰ (2) Pháp luật đại cương.pptxCHƯƠNG VII LUẬT DÂN SỰ (2) Pháp luật đại cương.pptx
CHƯƠNG VII LUẬT DÂN SỰ (2) Pháp luật đại cương.pptx22146042
 
BỘ ĐỀ PHÁT TRIỂN THEO CẤU TRÚC ĐỀ MINH HỌA BGD NGÀY 22-3-2024 KỲ THI TỐT NGHI...
BỘ ĐỀ PHÁT TRIỂN THEO CẤU TRÚC ĐỀ MINH HỌA BGD NGÀY 22-3-2024 KỲ THI TỐT NGHI...BỘ ĐỀ PHÁT TRIỂN THEO CẤU TRÚC ĐỀ MINH HỌA BGD NGÀY 22-3-2024 KỲ THI TỐT NGHI...
BỘ ĐỀ PHÁT TRIỂN THEO CẤU TRÚC ĐỀ MINH HỌA BGD NGÀY 22-3-2024 KỲ THI TỐT NGHI...Nguyen Thanh Tu Collection
 
Sơ đồ tư duy môn sinh học bậc THPT.pdf
Sơ đồ tư duy môn sinh học bậc THPT.pdfSơ đồ tư duy môn sinh học bậc THPT.pdf
Sơ đồ tư duy môn sinh học bậc THPT.pdftohoanggiabao81
 
ôn tập lịch sử hhhhhhhhhhhhhhhhhhhhhhhhhh
ôn tập lịch sử hhhhhhhhhhhhhhhhhhhhhhhhhhôn tập lịch sử hhhhhhhhhhhhhhhhhhhhhhhhhh
ôn tập lịch sử hhhhhhhhhhhhhhhhhhhhhhhhhhvanhathvc
 

Recently uploaded (20)

Sáng kiến Dạy học theo định hướng STEM một số chủ đề phần “vật sống”, Khoa họ...
Sáng kiến Dạy học theo định hướng STEM một số chủ đề phần “vật sống”, Khoa họ...Sáng kiến Dạy học theo định hướng STEM một số chủ đề phần “vật sống”, Khoa họ...
Sáng kiến Dạy học theo định hướng STEM một số chủ đề phần “vật sống”, Khoa họ...
 
[GIẢI PHẪU BỆNH] Tổn thương cơ bản của tb bào mô
[GIẢI PHẪU BỆNH] Tổn thương cơ bản của tb bào mô[GIẢI PHẪU BỆNH] Tổn thương cơ bản của tb bào mô
[GIẢI PHẪU BỆNH] Tổn thương cơ bản của tb bào mô
 
Ma trận - định thức và các ứng dụng trong kinh tế
Ma trận - định thức và các ứng dụng trong kinh tếMa trận - định thức và các ứng dụng trong kinh tế
Ma trận - định thức và các ứng dụng trong kinh tế
 
Sáng kiến “Sử dụng ứng dụng Quizizz nhằm nâng cao chất lượng ôn thi tốt nghiệ...
Sáng kiến “Sử dụng ứng dụng Quizizz nhằm nâng cao chất lượng ôn thi tốt nghiệ...Sáng kiến “Sử dụng ứng dụng Quizizz nhằm nâng cao chất lượng ôn thi tốt nghiệ...
Sáng kiến “Sử dụng ứng dụng Quizizz nhằm nâng cao chất lượng ôn thi tốt nghiệ...
 
bài 5.1.docx Sinh học di truyền đại cương năm nhất của học sinh y đa khoa
bài 5.1.docx Sinh học di truyền đại cương năm nhất của học sinh y đa khoabài 5.1.docx Sinh học di truyền đại cương năm nhất của học sinh y đa khoa
bài 5.1.docx Sinh học di truyền đại cương năm nhất của học sinh y đa khoa
 
SÁNG KIẾN “THIẾT KẾ VÀ SỬ DỤNG INFOGRAPHIC TRONG DẠY HỌC ĐỊA LÍ 11 (BỘ SÁCH K...
SÁNG KIẾN “THIẾT KẾ VÀ SỬ DỤNG INFOGRAPHIC TRONG DẠY HỌC ĐỊA LÍ 11 (BỘ SÁCH K...SÁNG KIẾN “THIẾT KẾ VÀ SỬ DỤNG INFOGRAPHIC TRONG DẠY HỌC ĐỊA LÍ 11 (BỘ SÁCH K...
SÁNG KIẾN “THIẾT KẾ VÀ SỬ DỤNG INFOGRAPHIC TRONG DẠY HỌC ĐỊA LÍ 11 (BỘ SÁCH K...
 
Tư tưởng Hồ Chí Minh về độc lập dân tộc và CNXH
Tư tưởng Hồ Chí Minh về độc lập dân tộc và CNXHTư tưởng Hồ Chí Minh về độc lập dân tộc và CNXH
Tư tưởng Hồ Chí Minh về độc lập dân tộc và CNXH
 
Chàm - Bệnh án (da liễu - bvdlct ctump) .pptx
Chàm - Bệnh án (da liễu - bvdlct ctump) .pptxChàm - Bệnh án (da liễu - bvdlct ctump) .pptx
Chàm - Bệnh án (da liễu - bvdlct ctump) .pptx
 
ĐỀ THAM KHẢO THEO HƯỚNG MINH HỌA 2025 KIỂM TRA CUỐI HỌC KÌ 2 NĂM HỌC 2023-202...
ĐỀ THAM KHẢO THEO HƯỚNG MINH HỌA 2025 KIỂM TRA CUỐI HỌC KÌ 2 NĂM HỌC 2023-202...ĐỀ THAM KHẢO THEO HƯỚNG MINH HỌA 2025 KIỂM TRA CUỐI HỌC KÌ 2 NĂM HỌC 2023-202...
ĐỀ THAM KHẢO THEO HƯỚNG MINH HỌA 2025 KIỂM TRA CUỐI HỌC KÌ 2 NĂM HỌC 2023-202...
 
10 ĐỀ KIỂM TRA + 6 ĐỀ ÔN TẬP CUỐI KÌ 2 VẬT LÝ 11 - KẾT NỐI TRI THỨC - THEO C...
10 ĐỀ KIỂM TRA + 6 ĐỀ ÔN TẬP CUỐI KÌ 2 VẬT LÝ 11 - KẾT NỐI TRI THỨC - THEO C...10 ĐỀ KIỂM TRA + 6 ĐỀ ÔN TẬP CUỐI KÌ 2 VẬT LÝ 11 - KẾT NỐI TRI THỨC - THEO C...
10 ĐỀ KIỂM TRA + 6 ĐỀ ÔN TẬP CUỐI KÌ 2 VẬT LÝ 11 - KẾT NỐI TRI THỨC - THEO C...
 
50 ĐỀ ĐỀ XUẤT THI VÀO 10 THPT SỞ GIÁO DỤC THANH HÓA MÔN TIẾNG ANH 9 CÓ TỰ LUẬ...
50 ĐỀ ĐỀ XUẤT THI VÀO 10 THPT SỞ GIÁO DỤC THANH HÓA MÔN TIẾNG ANH 9 CÓ TỰ LUẬ...50 ĐỀ ĐỀ XUẤT THI VÀO 10 THPT SỞ GIÁO DỤC THANH HÓA MÔN TIẾNG ANH 9 CÓ TỰ LUẬ...
50 ĐỀ ĐỀ XUẤT THI VÀO 10 THPT SỞ GIÁO DỤC THANH HÓA MÔN TIẾNG ANH 9 CÓ TỰ LUẬ...
 
Slide Webinar Hướng dẫn sử dụng ChatGPT cho người mới bắt đầ...
Slide Webinar Hướng dẫn sử dụng ChatGPT cho người mới bắt đầ...Slide Webinar Hướng dẫn sử dụng ChatGPT cho người mới bắt đầ...
Slide Webinar Hướng dẫn sử dụng ChatGPT cho người mới bắt đầ...
 
TỔNG HỢP ĐỀ THI CHÍNH THỨC KỲ THI TUYỂN SINH VÀO LỚP 10 THPT MÔN NGỮ VĂN NĂM ...
TỔNG HỢP ĐỀ THI CHÍNH THỨC KỲ THI TUYỂN SINH VÀO LỚP 10 THPT MÔN NGỮ VĂN NĂM ...TỔNG HỢP ĐỀ THI CHÍNH THỨC KỲ THI TUYỂN SINH VÀO LỚP 10 THPT MÔN NGỮ VĂN NĂM ...
TỔNG HỢP ĐỀ THI CHÍNH THỨC KỲ THI TUYỂN SINH VÀO LỚP 10 THPT MÔN NGỮ VĂN NĂM ...
 
BỘ ĐỀ KIỂM TRA CUỐI KÌ 2 VẬT LÝ 11 - KẾT NỐI TRI THỨC - THEO CẤU TRÚC ĐỀ MIN...
BỘ ĐỀ KIỂM TRA CUỐI KÌ 2 VẬT LÝ 11 - KẾT NỐI TRI THỨC - THEO CẤU TRÚC ĐỀ MIN...BỘ ĐỀ KIỂM TRA CUỐI KÌ 2 VẬT LÝ 11 - KẾT NỐI TRI THỨC - THEO CẤU TRÚC ĐỀ MIN...
BỘ ĐỀ KIỂM TRA CUỐI KÌ 2 VẬT LÝ 11 - KẾT NỐI TRI THỨC - THEO CẤU TRÚC ĐỀ MIN...
 
cuộc cải cách của Lê Thánh Tông - Sử 11
cuộc cải cách của Lê Thánh Tông -  Sử 11cuộc cải cách của Lê Thánh Tông -  Sử 11
cuộc cải cách của Lê Thánh Tông - Sử 11
 
Bài giảng về vật liệu ceramic ( sứ vệ sinh, gạch ốp lát )
Bài giảng về vật liệu ceramic ( sứ vệ sinh, gạch ốp lát )Bài giảng về vật liệu ceramic ( sứ vệ sinh, gạch ốp lát )
Bài giảng về vật liệu ceramic ( sứ vệ sinh, gạch ốp lát )
 
CHƯƠNG VII LUẬT DÂN SỰ (2) Pháp luật đại cương.pptx
CHƯƠNG VII LUẬT DÂN SỰ (2) Pháp luật đại cương.pptxCHƯƠNG VII LUẬT DÂN SỰ (2) Pháp luật đại cương.pptx
CHƯƠNG VII LUẬT DÂN SỰ (2) Pháp luật đại cương.pptx
 
BỘ ĐỀ PHÁT TRIỂN THEO CẤU TRÚC ĐỀ MINH HỌA BGD NGÀY 22-3-2024 KỲ THI TỐT NGHI...
BỘ ĐỀ PHÁT TRIỂN THEO CẤU TRÚC ĐỀ MINH HỌA BGD NGÀY 22-3-2024 KỲ THI TỐT NGHI...BỘ ĐỀ PHÁT TRIỂN THEO CẤU TRÚC ĐỀ MINH HỌA BGD NGÀY 22-3-2024 KỲ THI TỐT NGHI...
BỘ ĐỀ PHÁT TRIỂN THEO CẤU TRÚC ĐỀ MINH HỌA BGD NGÀY 22-3-2024 KỲ THI TỐT NGHI...
 
Sơ đồ tư duy môn sinh học bậc THPT.pdf
Sơ đồ tư duy môn sinh học bậc THPT.pdfSơ đồ tư duy môn sinh học bậc THPT.pdf
Sơ đồ tư duy môn sinh học bậc THPT.pdf
 
ôn tập lịch sử hhhhhhhhhhhhhhhhhhhhhhhhhh
ôn tập lịch sử hhhhhhhhhhhhhhhhhhhhhhhhhhôn tập lịch sử hhhhhhhhhhhhhhhhhhhhhhhhhh
ôn tập lịch sử hhhhhhhhhhhhhhhhhhhhhhhhhh
 

Luận văn: Lớp bài toán tìm giá trị lớn nhất giá trị nhỏ nhất, HOT

  • 1. ĐẠI HỌC QUỐC GIA HÀ NỘI TRƯỜNG ĐẠI HỌC KHOA HỌC TỰ NHIÊN NGUYỄN THỊ DIỆP PHƯƠNG PHÁP ĐẠO HÀM VÀ CÁC BÀI TOÁN VỀ TÌM GIÁ TRỊ LỚN NHẤT VÀ NHỎ NHẤT Chuyên ngành : Phương pháp toán sơ cấp Mã số: 60 46 01 13 LUẬN VĂN THẠC SỸ KHOA HỌC Người hướng dẫn khoa học: PGS.TS. NGUYỄN MINH TUẤN Hà Nội- 2015
  • 2. Lời cám ơn Trước khi trình bày nội dung chính của luận văn, tôi xin bày tỏ lòng cảm ơn chân thành tới PGS.TS. Nguyễn Minh Tuấn, người thầy đã trực tiếp hướng dẫn, chỉ bảo tận tình và giúp đỡ tôi trong suốt quá trình hoàn thành luận văn này. Tôi cũng xin chân thành cảm ơn sự giúp đỡ của các thầy giáo, cô giáo trong khoa Toán Cơ Tin học, Trương Đại học Khoa học Tự Nhiên-Đại học Quốc gia Hà Nội và Khoa sau đại học, đã nhiệt tình giúp đỡ tôi hoàn thành khóa Cao học. Tôi xin bày tỏ lòng biết ơn đến gia đình, bạn bè đã luôn động viên và khuyến khích tôi rất nhiều trong thời gian nghiên cứu và học tập. Do mới làm quen với công tác nghiên cứu khoa học nên luận văn còn nhiều thiếu sót. Tác giả kính mong nhận được ý kiến đóng góp của các thầy cô và các bạn để luận văn hoàn thiện hơn. Hà Nội, năm 2015 Nguyễn Thị Diệp 2
  • 3. Mục lục Lời mở đầu 4 1 Một số kiến thức chuẩn bị 6 1.1 Định nghĩa đạo hàm tại một điểm . . . . . . . . . . . . . . . . . . . . . 6 1.2 Cực trị của hàm số . . . . . . . . . . . . . . . . . . . . . . . . . . . . . 7 1.3 Các định lí cơ bản về hàm khả vi . . . . . . . . . . . . . . . . . . . . . 8 1.4 Hàm lồi và hàm lõm . . . . . . . . . . . . . . . . . . . . . . . . . . . . 9 2 Ứng dụng đạo hàm giải các bài toán tìm giá trị lớn nhất, giá trị nhỏ nhất của hàm số 11 2.1 Khảo sát trực tiếp hàm số trên miền xác định . . . . . . . . . . . . . . 11 2.2 Khảo sát hàm số theo từng biến . . . . . . . . . . . . . . . . . . . . . . 17 2.3 Đặt biến phụ chuyển về đánh giá hàm số một biến . . . . . . . . . . . . 30 2.4 Đánh giá gián tiếp thông qua biểu thức bậc nhất . . . . . . . . . . . . 44 2.5 Phương pháp sử dụng tính chất của hàm lồi, hàm lõm . . . . . . . . . 51 3 Cực trị hàm nhiều biến 59 3.1 Cực trị tự do . . . . . . . . . . . . . . . . . . . . . . . . . . . . . . . . 59 3.2 Cực trị có điều kiện . . . . . . . . . . . . . . . . . . . . . . . . . . . . . 63 3
  • 4. Lời mở đầu Trong những năm gần đây, các kỳ khảo sát chất lượng, thi học sinh giỏi bậc trung học phổ thông thường gặp những bài toán yêu cầu tìm giá trị nhỏ nhất, giá trị lớn nhất của một đại lượng nào đó. Các bài toán cực trị rất phong phú và đa dạng mang nội dung vô cùng sâu sắc, có ý nghĩa rất quan trọng đối với các em học sinh. Các bài toán về cực trị góp phần không nhỏ vào việc rèn luyện tư duy cho học sinh. Bài toán đi tìm cái tốt nhất, rẻ nhất, ngắn nhất, dài nhất... trong một bài toán. Để dần dần hình thành cho học sinh thói quen đi tìm giải pháp tối ưu cho một công việc nào đó trong cuộc sống sau này. Luận văn trình bày một số ứng dụng của đạo hàm để giải các bài toán cực trị. Luận văn chỉ đề cập tới một số phương pháp giải một số loại toán cực trị đại số thường gặp trong chương trình toán học trung học phổ thông. Luận văn hệ thống hóa, phân loại toán và trình bày theo từng ý tưởng cũng như các kỹ năng vận dụng đạo hàm vào việc giải một lớp các bài toán tìm giá trị lớn nhất và giá trị nhỏ nhất. Luận văn gồm có 3 chương với các nội dung sau: Chương 1: Luận văn trình bày các kiến thức khái niệm cần thiết như đạo hàm, tính đơn điệu và hàm lồi và được tham khảo trong [3]. Chương 2: Luận văn trình bày phương pháp sử dụng đạo hàm vào giải các bài toán tìm giá trị lớn nhất, giá trị nhỏ nhất. Chương 2 luận văn trình bày phương pháp khảo sát trực tiếp hàm số trên tập xác định của hàm số, khảo sát theo hàm số từng biến, đặt biến phụ chuyển về đánh giá hàm một biến, đánh giá thông qua biểu thức bậc nhất, hay phương pháp sử dụng tính chất hàm lồi, hàm lõm... được tham khảo trong [1, 5, 6, 2, 7, 4]. 4
  • 5. 5 Chương 3. Luận văn trình bày phương pháp để tìm cực trị tự do và cực trị có điều kiện của hàm nhiều biến số. Từ đó tìm giá trị lớn nhất, giá trị nhỏ nhất của hàm số và được tham khảo trong [3].
  • 6. Chương 1 Một số kiến thức chuẩn bị 1.1 Định nghĩa đạo hàm tại một điểm Định nghĩa 1.1. Cho hàm số y = f(x) xác định trên khoảng (a, b) và x0 ∈ (a, b). Nếu giới hạn sau tồn tại và hữu hạn lim x→x0 f(x) − f(x0) x − x0 thì giới hạn này được gọi là đạo hàm của hàm số f tại điểm x0 và được ký hiệu là f (x0). Khi đó ta nói rằng f khả vi tại x0. Chú ý. Nếu kí hiệu ∆x = x − x0, ∆y = f(x0 + ∆x) − f(x0) thì f (x0) = lim x→x0 f(x0 + ∆x) − f(x0) x − x0 = lim ∆x→0 ∆y ∆x . Nếu hàm số y = f(x) có đạo hàm tại x0 thì nó liên tục tại điểm đó. Ý nghĩa hình học. Cho hàm số y = f(x) có đồ thị (C). Khi đó, f (x0) là hệ số góc của tiếp tuyến đồ thị (C) của hàm số y = f(x) tại M(x0, y0) ∈ (C). Phương trình tiếp tuyến của đồ thị hàm số y = f(x) tại điểm M(x0, y0) ∈ (C) là y = f (x0)(x − x0) + y0. 6
  • 7. 7 1.2 Cực trị của hàm số Định nghĩa 1.2. Cho hàm số y = f(x) xác định trên tập hợp D ⊂ R và x0 ∈ D. Điểm x0 được gọi là một điểm cực đại của hàm số f(x) nếu tồn tại một khoảng (a, b) chứa điểm x0 sao cho f(x) ≤ f(x0) với ∀x ∈ (a, b) ∩ D. Khi đó f(x0) được gọi là giá trị cực đại của f(x) và điểm (x0, f(x0)) được gọi là điểm cực đại của đồ thị hàm số y = f(x). Điểm x0 được gọi là một điểm cực tiểu của hàm số f(x) nếu tồn tại một khoảng (a, b) chứa điểm x0 sao cho f(x) ≥ f(x0) với ∀x ∈ (a, b) ∩ D. Khi đó f(x0) được gọi là giá trị cực tiểu của f(x) và điểm (x0, f(x0)) được gọi là điểm cực tiểu của đồ thị hàm số y = f(x). Điểm cực đại, cực tiểu được gọi chung là điểm cực trị. Giá trị cực đại, giá trị cực tiểu được gọi chung là cực trị. Định lý 1.3. Cho hàm số y = f(x) xác định và liên tục trên [a, b]. Nếu f (x) ≥ 0 ∀x ∈ [a, b] thì f(x) đồng biến trên [a, b] và khi đó ta có min x∈[a,b] f(x) = f(a), max x∈[a,b] f(x) = f(b). Nếu f (x) ≤ 0 ∀x ∈ [a, b] thì f(x) nghịch biến trên [a, b] và khi đó ta có min x∈[a,b] f(x) = f(b), max x∈[a,b] f(x) = f(a). Chú ý. Khái niệm cực đại cực tiểu của một hàm số có tính chất địa phương, chúng chưa chắc đã là giá trị lớn nhất, giá trị nhỏ nhất của hàm số. Ta có kết quả sau về điều kiện cần của cực trị. Định lý 1.4. ( Định lý Fermat) Cho hàm f xác định trên (a, b) và x0 ∈ (a, b). Nếu hàm số f có cực trị tại x0 và hàm f có đạo hàm tại x0 thì f (x0) = 0. Chú ý. Điều ngược lại không đúng: Nếu hàm f có f (x0) = 0 nhưng chưa chắc x0 là điểm cực trị, ví dụ hàm y = x3 có y (0) = 0 nhưng hàm số không có cực trị tại x = 0.
  • 8. 8 Nếu hàm số f có cực trị tại x0 thì có thể tại x0 đạo hàm không xác định, ví dụ hàm y = |x| có cực tiểu tại x = 0 nhưng dễ chứng minh được hàm số không có đạo hàm tại x = 0. Định lý 1.5. Giả sử hàm số f khả vi trên khoảng (a, b) chứa x0, và f (x0) = 0. Nếu f (x) ≥ 0 với mọi x ∈ (a, x0) và f (x) ≤ 0 với mọi x ∈ (x0, b) ( tức đạo hàm đổi dấu từ (+) sang (-) khi đi qua x0) thì x0 là điểm cực tiểu của hàm f. Nếu f (x) ≤ 0 với mọi x ∈ (a, x0) và f (x) ≥ 0 với mọi x ∈ (x0, b) ( tức đạo hàm đổi dấu từ (-) sang (+) khi đi qua x0) thì x0 là điểm cực đại của hàm f. Định lý 1.6. Giả sử hàm số f có đạo hàm cấp một trên khoảng (a, b) chứa x0, có đạo hàm cấp hai khác 0 tại x0. Nếu f (x0) = 0 và f (x0) > 0 thì x0 là điểm cực tiểu của hàm f. Nếu f (x0) = 0 và f (x0) < 0 thì x0 là điểm cực đại của hàm f. 1.3 Các định lí cơ bản về hàm khả vi Trong phần này, luận văn trình bày hai định lý quan trọng về đạo hàm. Đó là định lí Lagrange, định lí Rolle (xem [3]). Định lý 1.7. (Định lý Rolle) Nếu f(x) là hàm liên tục trên đoạn [a, b], có đạo hàm trên khoảng (a, b) và f(a) = f(b) thì tồn tại c ∈ (a, b) sao cho f (c) = 0. Chứng minh. Vì f(x) liên tục trên [a, b] nên theo định lí Weierstrass f(x) nhận giá trị lớn nhất M và giá trị nhỏ nhất m trên [a, b]. - Khi M = m ta có f(x) là hàm hằng trên [a, b], do đó với mọi c ∈ (a, b) luôn có f (c) = 0. - Khi M > m, vì f(a) = f(b) nên tồn tại c ∈ (a, b) sao cho f(c) = m hoặc f(c) = M, theo Định lý Fermat suy ra f (c) = 0. Định lý được chứng minh. Hệ quả 1.8. Nếu hàm số f(x) có đạo hàm trên (a, b) và f(x) có n nghiệm ( n là số nguyên dương lớn hơn 1) trên (a, b) thì f (x) có ít nhất n − 1 nghiệm trên (a, b).
  • 9. 9 Hệ quả 1.9. Nếu hàm số f(x) có đạo hàm trên (a, b) và f (x) vô nghiệm trên (a, b) thì f(x) có nhiều nhất 1 nghiệm trên (a, b). Hệ quả 1.10. Nếu f(x) có đạo hàm trên (a, b) và f (x) có nhiều nhất n nghiệm (nlà số nguyên dương) trên (a, b) thì f(x) có nhiều nhất n + 1 nghiệm trên (a, b). Định lý 1.11. (Định lí Lagrange) Nếu f(x) là hàm liên tục trên đoạn [a, b], có đạo hàm trên khoảng (a, b) thì tồn tại c ∈ (a, b) sao cho f (c) = f(b) − f(a) b − a . Chứng minh. Xét hàm số F(x) = f(x) − f(b) − f(a) b − a x, x ∈ [a, b]. Khi đó F(x) là hàm liên tục trên đoạn[a, b], có đạo hàm trên khoảng (a, b) và F(a) = F(b). Theo định lí Rolle tồn tại c ∈ (a, b) sao cho F (c) = 0. Mà F (x) = f (x) − f(b) − f(a) b − a , suy ra f (c) = f(b) − f(a) b − a . Định lý được chứng minh. Định lí Rolle là một hệ quả của định lý Lagrange trong trường hợp f(a) = f(b). 1.4 Hàm lồi và hàm lõm Ta ký hiệu I(a, b) là một tập hợp có một trong bốn dạng tập hợp sau (a, b), [a, b), (a, b] và [a, b]. Định nghĩa 1.12. Hàm số f(x) được gọi là lồi trên tập I(a, b) nếu với mọi x1, x2 ∈ I(a, b) và với mọi cặp số dương α, β có tổng α + β = 1, ta đều có f(αx1 + βx2) ≤ αf(x1) + βf(x2). (1.1) Nếu dấu đẳng thức trong (1.1) xảy ra khi và chỉ khi x1 = x2 thì ta nói hàm số f(x) là hàm lồi thực sự (chặt) trên I(a, b).
  • 10. 10 Định nghĩa 1.13. Hàm số f(x) được gọi là lõm trên tập I(a, b) nếu với mọi x1, x2 ∈ I(a, b) và với mọi cặp số dương α, β có tổng α + β = 1, ta đều có f(αx1 + βx2) ≥ αf(x1) + βf(x2). (1.2) Nếu dấu đẳng thức trong (1.2) xảy ra khi và chỉ khi x1 = x2 thì ta nói hàm số f(x) là hàm lõm thực sự (chặt) trên I(a, b). Định lý 1.14. Nếu f(x) khả vi bậc hai trên I(a, b) thì f(x) lồi (lõm) trên I(a, b) khi và chỉ khi f (x) ≥ 0(f (x) ≤ 0) trên I(a, b). Nếu f(x) lồi khả vi trên I(a, b) thì với mọi cặp x0, x ∈ I(a, b), ta đều có f(x) ≥ f(x0) + f (x0)(x − x0) (1.3) Dễ nhận thấy rằng (1.3) xảy ra đẳng thức khi x0 = x. Vậy ta có thể viết (1.3) dưới dạng f(x) = min u∈I(a,b) f(u) + f (u)(x − u).
  • 11. Chương 2 Ứng dụng đạo hàm giải các bài toán tìm giá trị lớn nhất, giá trị nhỏ nhất của hàm số 2.1 Khảo sát trực tiếp hàm số trên miền xác định Bài toán 1. ( Thi HSG Quốc gia, 1992) Cho số tự nhiên n > 1. Tìm giá trị lớn nhất và nhỏ nhất của hàm số f(x) = n √ 1 + x + n √ 1 − x với x thuộc [0, 1]. Chứng minh. Hàm số f(x) = n √ 1 + x + n √ 1 − x liên tục trên miền [0, 1], hơn nữa f (x) = 1 n 1 n (1 + x)n−1 − 1 n (1 − x)n−1 < 0 ∀x ∈ (0, 1). Vậy f(x) nghịch biến [0, 1] nên n √ 2 = f(1) ≤ f(x) ≤ f(0) = 2 ∀x ∈ [0, 1]. Vậy giá trị lớn nhất của f(x) = n √ 1 + x + n √ 1 − x trên [0, 1] là 2 đạt được khi x = 0, giá trị nhỏ nhất của f(x) trên [0, 1] là n √ 2 đạt được khi x = 1. 11
  • 12. 12 Bài toán 2. a. Tìm giá trị lớn nhất của biểu thức x + 1 √ x2 − x + 1 với x ∈ R. b. Tìm giá trị nhỏ nhất của biểu thức √ x2 − x + 1 + y2 − y + 1 + √ z2 − z + 1 trong đó các số x, y, z thỏa mãn x + y + z = 3. Chứng minh. a) Xét hàm số f(x) = x + 1 √ x2 − x + 1 với x ∈ R. Ta có f (x) = 3(1 − x) 2(x2 − x + 1) √ x2 − x + 1 và do đó f (x) = 0 khi và chỉ khi x = 1, limx→+∞ f(x) = 1, limx→−∞ f(x) = −1, và f (x) đổi dấu từ + sang − khi đi qua 1. Do đó f(x) ≤ f(1) = 2 ∀x ∈ R. Vậy giá trị lớn nhất của f(x) trên R là 2 đạt được khi x = 1. b) Áp dụng câu a) ta có x + 1 √ x2 − x + 1 ≤ 2 ∀x ∈ R. Hay √ x2 − x + 1 ≥ 1 2 (x + 1). (2.1) Tương tự ta có y2 − y + 1 ≥ 1 2 (y + 1), (2.2) √ z2 − z + 1 ≥ 1 2 (z + 1). (2.3) Cộng từng vế các bất đẳng thức (2.1), (2.2) và (2.3) ta có √ x2 − x + 1 + y2 − y + 1 + √ z2 − z + 1 ≥ 1 2 (3 + x + y + z).
  • 13. 13 Mà x + y + z = 3 nên ta có √ x2 − x + 1 + y2 − y + 1 + √ z2 − z + 1 ≥ 3. Dấu bằng xảy ra khi x = y = z = 1. Vậy giá trị nhỏ nhất của √ x2 − x + 1 + y2 − y + 1 + √ z2 − z + 1 là 3. Bài toán 3. Giả sử A, B, C là ba góc của một tam giác nhọn. Tìm giá trị nhỏ nhất của biểu thức tan A + tan B + tan C + 6(sin A + sin B + sin C). Chứng minh. Xét hàm số f(x) = tan x + 6 sin x − 7x với x ∈ (0, π/2). Ta có f (x) = 1 cos2 x + 6 cos x − 7 = (cos x − 1)(3 cos x + 1)(2 cos x − 1) cos2 x . Vì x ∈ (0, π/2) nên f (x) = 0 khi 2 cos x − 1 = 0 hay x = π/3. Ta thấy f (x) đổi dấu từ − sang + khi đi qua π/3. Nên ta được min x∈(0,π/2) f(x) = f(π/3) = 4 √ 3 − 7π 3 . Áp dụng điều này lần lượt cho x = A, x = B, x = C ta được f(A) + f(B) + f(C) ≥ 3(4 √ 3 − 7π 3 ). Hay tan A + tan B + tan C + 6(sin A + sin B + sin C) ≥ 12 √ 3. Vậy giá trị nhỏ nhất của tan A + tan B + tan C + 6(sin A + sin B + sin C) là 12 √ 3 đạt được khi A = B = C = π/3. Bài toán 4. Giả sử x > 0, y > 0 và x + y = 1. Chứng minh rằng giá trị lớn nhất của biểu thức x √ 1 − x + y √ 1 − y là √ 2.
  • 14. 14 Chứng minh. Theo giả thiết thì y = 1 − x. Ta sẽ chứng minh rằng x √ 1 − x + 1 − x √ x ≥ √ 2 ∀x ∈ (0, 1). Xét f(x) = x √ 1 − x + 1 − x √ x với x ∈ (0, 1). Ta có f (x) = 1 2 2 − x (1 − x) √ 1 − x − x + 1 x √ x = 1 2 1 + (1 − x) (1 − x) √ 1 − x − 1 + x x √ x = 1 2 (h( √ 1 − x) − h( √ x)) trong đó h(t) = 1 + t2 t3 = 1 t3 + 1 t . Ta thấy hàm h(t) là hàm nghịch biến trên (0, +∞), nên f (x) = 0 ⇔ h( √ 1 − x) = h( √ x) ⇔ √ 1 − x = √ x ⇔ x = 1 2 và f (x) > 0 ⇔ h( √ 1 − x) > h( √ x) ⇔ √ 1 − x < √ x ⇔ x ∈ ( 1 2 , 1) f (x) < 0 ⇔ x ∈ (0, 1 2 ). Vậy trên (0, 1) thì f (x) đổi dấu từ − sang + khi đi qua 1 2 . Nên ta được min x∈(0,1) f(x) = f( 1 2 ) = √ 2. Suy ra f(x) ≥ √ 2 ∀x ∈ (0, 1). Vậy giá trị nhỏ nhất của x √ 1 − x + y √ 1 − y là √ 2 đạt được khi x = y = 1 2 . Bài toán 5. Tìm giá trị nhỏ nhất của hàm số f(x) = √ x2 + x + 1 + √ x2 − x + 1.
  • 15. 15 Chứng minh. Tập xác định R. Xét hàm số f(x) = √ x2 + x + 1 + √ x2 − x + 1 trên R. Ta có f (x) = x + 1 2 (x + 1 2 )2 + 3 4 + x − 1 2 (x − 1 2 )2 + 3 4 = g(x + 1 2 ) − g( 1 2 − x) ∀x ∈ R trong đó g(t) = t t2 + 3 4 với t ∈ R. Vì hàm g đồng biến trên R nên f (x) = 0 ⇔ x = 0, f (x) > 0 ⇔ g(x + 1 2 ) > g( 1 2 − x) ⇔ x > 0. Vậy trên R thì f (x) đổi dấu từ − sang + khi đi qua 0. Từ đó giá trị nhỏ nhất của f(x) là 2 khi x = 0. Bài toán 6. Giả sử a, b ∈ R+ và a = b. Tìm giá trị nhỏ nhất của biểu thức ( a + x b + x )b+x với x ∈ [0, +∞). Chứng minh. Xét hàm số f(x) = ( a + x b + x )b+x , x ≥ 0. Khi đó ln f(x) = (b + x) ln a + x b + x . Suy ra [ln f(x)] = [(b + x) ln a + x b + x ] , và do đó f (x) f(x) = ln a + x b + x + (b + x) b + x a + x ( a + x b + x ) = ln a + x b + x + b − a a + x .
  • 16. 16 Hay f (x) = f(x) ln a + x b + x + b − a a + x = ( a + x b + x )b+x g(x) trong đó g(x) = ln a + x b + x + b − a a + x . Ta có g (x) = b + x a + x . b − a (b + x)2 + a − b (a + x)2 = − (a − b)2 (a + x)2(b + x) < 0. Do đó g(x) nghịch biến trên (0, +∞). Suy ra g(x) > lim x→∞ g(x) = lim x→∞ ln a + x b + x + b − a a + x = 0. Vậy f (x) > 0 ∀x > 0, nên f(x) đồng biến trên [0, +∞). Suy ra f(x) ≥ f(0) ∀x ≥ 0. Vậy giá trị nhỏ nhất của biểu thức (a+x b+x )b+x với x ∈ [0, +∞) là (a b )b . Bài toán 7. Tìm giá trị nhỏ nhất của biểu thức 2sin x + 2tan x − 2x+1 0 ≤ x ≤ π 2 . Chứng minh. Áp dụng bất đẳng thức AM-GM ta có 2sin x + 2tan x ≥ 2 √ 2sin x2tan x. Ta chứng minh sin x + tan x ≥ 2x. Thật vậy, xét hàm số f(x) = sin x + tan x − 2x liên tục trên [0, π 2 ], và có f (x) = cos x + 1 cos2 x − 2 > cos2 x + 1 cos2 x − 2 ≥ 0 ∀x ∈ [0, π 2 ]. Do đó f(x) đồng biến trên [0, π 2 ]. Suy ra f(x) ≥ f(0) = 0, hay sin x + tan x ≥ 2x với mọi x ∈ [0, π 2 ]
  • 17. 17 Vậy giá trị nhỏ nhất của biểu thức 2sin x + 2tan x − 2x+1 với 0 ≤ x ≤ π 2 là 0 đạt được khi x = 0. 2.2 Khảo sát hàm số theo từng biến Đối với các BĐT nhiều biến, ta có thể chọn một biến là biến số biến thiên và cố định các biến còn lại, bài toán lúc này trở thành BĐT một biến. Bài toán 8. Giả sử A, B, C là ba góc của một tam giác. Tìm giá trị nhỏ nhất của biểu thức Q = 2 1 sin A + 1 sin B + 1 sin C − (cot A + cot B + cot C). Chứng minh. Ta viết Q = 2 sin A − cot A + 2 sin B − cot B + 2 sin C − cot C . Xét hàm số f(x) = 2 sin x − cot x với x ∈ (0, π). Ta có f (x) = 1 − 2 cos x sin2 x . Khi đó, hàm f (x) trong khoảng (0, π) sẽ chuyển từ − sang + khi đi qua điểm π/3. Từ đó ta có f(A) ≥ √ 3 f(B) ≥ √ 3 f(C) ≥ √ 3. Suy ra Q = f(A) + f(B) + f(C) ≥ 3 √ 3.
  • 18. 18 Dấu đẳng thức khi tam giác ABC đều. Vậy giá trị nhỏ nhất của biểu thức Q = 2 1 sin A + 1 sin B + 1 sin C − (cot A + cot B + cot C) là 3 √ 3. Bài toán 9. Giả sử các số thực a, b, c > 0, thỏa mãn điều kiện a2 + b2 + c2 = 1. Tìm giá trị nhỏ nhất của biểu thức a b2 + c2 + b c2 + a2 + c a2 + b2 . Chứng minh. Ta sẽ chứng minh rằng a b2 + c2 + b c2 + a2 + c a2 + b2 ≥ 3 √ 3 2 . (2.4) Ta có (2.4) tương đương a 1 − a2 + b 1 − b2 + c 1 − c2 ≥ 3 √ 3 2 (a2 + b2 + c2 ). Từ đó gợi ý ta chứng minh các bất đẳng thức sau a 1 − a2 ≥ 3 √ 3 2 a2 , b 1 − b2 ≥ 3 √ 3 2 b2 , c 1 − c2 ≥ 3 √ 3 2 c2 . Hay phải chứng minh a(1 − a2 ) ≤ 2 3 √ 3 , b(1 − b2 ) ≤ 2 3 √ 3 , c(1 − c2 ) ≤ 2 3 √ 3 . Khảo sát đại diện là hàm số f(x) = x(1 − x2 ) với x ∈ (0, 1). Ta có f (x) = 1 − 3x2 . Khi đó, hàm f (x) trong khoảng (0, 1) sẽ chuyển từ − sang + khi đi qua điểm 1/ √ 3. Nên giá trị lớn nhất của f(x) trên khoảng (0, 1) là 2 3 √ 3 . Vậy giá trị nhỏ nhất của biểu thức a b2+c2 + b c2+a2 + c a2+b2 là 3 √ 3 2 đạt được khi a = b = c = 1/ √ 3.
  • 19. 19 Bài toán 10. Chứng minh rằng giá trị lớn nhất của biểu thức 2(x3 + y3 + z3 ) − (x2 y + y2 z + z2 x) với x, y, z ∈ [0, 1] là 3. Chứng minh. Ta sẽ chứng minh rằng f(x) = 2x3 − yx2 − z2 x + 2(y3 + z3 ) ≤ 3. Thật vậy, ta có f (x) = 6x2 − 2yz − z2 và f (x) = 0 khi x = x1 = 1 6 (y − y2 + 6z2), x = x2 = 1 6 (y + y2 + 6z2). Xét hai trường hợp Trường hợp 1. Nếu x2 ∈ (0, 1) suy ra f (x) ≤ 0 ∀x ∈ [0, 1]. Suy ra f(x) giảm trên [0, 1]. Do đó max x∈[0,1] f(x) = max{f(0), f(1)}. Trường hợp 2. Nếu x2 ∈ (0, 1) thì do x1 ≤ 0 < x2 nên trên [0, 1] hàm f (x) sẽ đổi dấu từ − sang + khi đi qua x2. Do đó max x∈[0,1] f(x) = max{f(0), f(1)}. Như vậy trong cả hai trường hợp ta đều có max x∈[0,1] f(x) = max{f(0), f(1)}. Mặt khác f(0) = 2(y3 + z3 ) − y2 z ≤ 2(y3 + z3 ) − y2 z + (2 − y − z2 ) = f(1). Nên ta được max x∈[0,1] f(x) = f(1) = 2(y3 + z3 ) − y2 z + (2 − y − z2 ).
  • 20. 20 Ta sẽ chứng minh f(1) ≤ 3. Thật vậy, đặt f(1) = g(y) = 2(y3 + z3 ) − y2 z + (2 − y − z2 ). Khi đó g (y) = 6y2 − 2zy − 1 = 0 và g (y) = 0 ⇔ y = y1 = 1 6 (z − √ z2 + 6) < 0, y = y2 = 1 6 (z + √ z2 + 6). • Nếu y2 ∈ (0, 1) thì g (y) ≤ 0 ∀y ∈ [0, 1]. Suy ra g(y) giảm trên [0, 1]. Do đó max y∈[0,1] g(y) = max{g(0), g(1)}. Nếu y2 ∈ (0, 1) thì trên [0, 1] hàm g (y) sẽ đổi dấu từ − sang + khi đi qua y2. Do vậy max y∈[0,1] g(y) = max{g(0), g(1)}. Như vậy trong cả hai trường hợp ta đều có max y∈[0,1] g(y) = max{g(0), g(1)}. Ta có g(0) = 2z3 + 2 − z2 ≤ 2z3 + 2 − z2 + (1 − z) = g(1) = z(z − 1)(2z + 1) + 3 ≤ 3 với mọi x, y, z ∈ [0, 1]. Do đó max y∈[0,1] g(y) = 3. Khi x = y = z = 1 thì 2(x3 + y3 + z3 ) − (x2 y + y2 z + z2 x) = 3. Vậy giá trị lớn nhất của 2(x3 + y3 + z3 ) − (x2 y + y2 z + z2 x) là 3. Bài toán 11. Giả sử a, b, c ∈ [1 3 , 3]. Tìm giá trị lớn nhất của biểu thức S(a, b, c) = a a + b + b b + c + c c + a . Chứng minh. Đặt f(a) = a a + b + b b + c + c c + a .
  • 21. 21 Xét hai trường hợp sau Trường hợp 1. a ≥ b ≥ c. Ta có f (a) = b (a + b)2 − c (a + c)2 = (b − c)(a2 − bc) (a + b)2(a + c)2 ≥ 0. Suy ra f(a) ≤ f(3) = 3 3 + b + b b + c + c c + 3 = g(c). Mặt khác g (a) = −b (c + b)2 + 3 (c + 3)2 = (b − 3)(3b − c2 ) (c + 3)2(b + c)2 ≥ 0. Suy ra g(c) ≤ g( 1 3 ) = 3 3 + b + 3b 3b + 1 + 1 10 = h(b). Ta có h (b) = 3 (3b + 1)2 − 3 (b + 3)2 = (1 − b)(1 + b) (3b + 1)2(b + 3)2 . Nên h(b) ≤ h(1). Suy ra S(a, b, c) ≤ S(3, 1, 1 3 ) = 8 5 . Trường hợp 2. c ≥ b ≥ a. Áp dụng trường hợp 1 ta nhận được S(c, b, a) ≤ 8 5 . Mặt khác S(a, b, c) − S(c, b, a) = (a − b)(b − c)(a − c) (a + b)(b + c)(a + c) ≤ 0. Suy ra S(a, b, c) ≤ 8 5 . Vậy giá trị lớn nhất của S(a, b, c) là 8 5 , đạt được khi và chỉ khi (a, b, c) ∈ {(3, 1, 1 3 ), (1 3 , 3, 1), (1, 1 3 , 3)}. Bài toán 12. Giả sử a, b, c ∈ [0, 1]. Tìm giá trị lớn nhất của biểu thức S = a b3 + c3 + 6 + b c3 + a3 + 6 + c a3 + b3 + 6 .
  • 22. 22 Chứng minh. Đặt f(c) = a b3 + c3 + 6 + b c3 + a3 + 6 + c a3 + b3 + 6 . Ta có f (c) = 1 b3 + c3 + 6 − 3ac2 (b3 + c3 + 6)2 − 3c2 (a3 + c3 + 6)2 và f (c) = 6ac(6 + b3 − 2c3 ) (b3 + c3 + 6)2 − 6bc(6 + a3 − 2c3 ) (a3 + c3 + 6)2 ≤ 0. Nên f (c) giảm trên [0, 1]. Suy ra f (c) ≥ f (1) = 1 b3 + a3 + 6 − 3a (7 + b3)2 − 3b (7 + a3)2 ≥ 1 8 − 2 3 49 > 0. Suy ra f(c) tăng trên [0, 1]. Do đó S = f(c) ≤ f(1) = a b3 + 7 + b a3 + 7 + 1 a3 + b3 + 6 = g(a). Ta có g (a) = 1 b3 + 7 − 2a2 b (a3 + 7)2 − 1 (a3 + b3 + 7)2 và g (a) = 6ab(7 − 2a3 ) (a3 + 7)3 − 6a(b3 + 6 − 2a3 ) (a3 + b3 + 7)3 ≤ 0. Nên g (a) giảm trên [0, 1]. Suy ra g (a) ≥ g (1) = 1 b3 + 7 − 3b 64 − 3 (7 + b3)2 = 1 b3 + 7 − 1 8 5 8 − 3 b3 + 7 + 5 − 3b 64 > 0. Suy ra g(a) tăng trên [0, 1]. Do đó S = g(a) ≤ g(1) = 2 b3 + 7 + b 8 = h(b). Ta có h (b) = 1 8 − 6b2 (b3 + 7)2 − (b3 + 7)2 − 48b2 8(b3 + 7)2 > 0 ∀b ∈ [0, 1]. Suy ra h(b) tăng trên [0, 1], nên h(b) ≤ h(1) = 3 8 . Hay S ≤ 3 8 . Với a = b = c = 1 thì S = 3 8 . Vậy giá trị lớn nhất của S là 3 8 .
  • 23. 23 Bài toán 13. Xét hàm số f(x, y) = (1 − x)(2 − y)(4x − 2y) trên miền D = {(x, y) : 0 ≤ x ≤ 1, 0 ≤ y ≤ 2}. Tìm giá trị nhỏ nhất của hàm f trên miền D. Chứng minh. Biến đổi hàm số đã cho thành f(x, y) = 2(1 − x)(2 − y)((2 − y) − 2(1 − x)) Đặt u = 1 − x, v = 2 − y, thì bài toán đã cho trở thành tìm giá trị nhỏ nhất của hàm số F(u, v) = −2uv2 + u2 v trên miền E = {(u, v) : 0 ≤ u ≤ 2, 0 ≤ v ≤ 1}, nghĩa là min E F(u, v) = min 0≤u≤2 [ min 0≤v≤1 (−2uv2 + u2 v)]. Xét hàm số g(v) = −2uv2 + u2 v với 0 ≤ v ≤ 1, coi u là tham số. Ta có g (v) = −4uv + u2 = u(−4v + u). Ta thấy g (v) = 0 khi v0 = u 4 , và qua v0 = u 4 thì g (v) đổi dấu từ dương sang âm, mà 0 ≤ u 4 ≤ 1 2 , ta suy ra max 0≤v≤1 g(v) = min{g(0), g(1)} = min{0, u2 − 2u}. Do u2 − 2u ≤ 0 nên min E F(u, v) = min 0≤u≤2 (u2 − 2u) = −1 khi u = 1, v = 1. Từ đó min D f(x, y) = 2 min E F(u, v) = −2 khi x = 0, y = 1. Vậy giá trị nhỏ nhất của f(x, y) trên miền D là −2. Bài toán 14. (Đề thi HSG THPT toàn quốc bảng A, 1999) Xét các số thực dương a, b, c thỏa mãn abc + a + c = b. Tìm giá trị lớn nhất của biểu thức P = 2 a2 + 1 − 2 b2 + 1 + 3 c2 + 1 .
  • 24. 24 Chứng minh. Biến đổi giả thiết thành a + c = b(1 − ac) > 0, suy ra a < 1 c , b = a + c 1 − ac . (2.5) Thay (2.5) vào biểu thức P và biến đổi được P = 2 a2 + 1 + 3 c2 + 1 + 2(a + c)2 (1 + a2)(1 + b2) − 2. (2.6) Xét hàm số f(x) = 1 x2 + 1 + 2(x + c)2 (1 + x2)(1 + c2) với 0 < x < 1 2 và coi c là tham số (c > 0). Ta có f (x) = 2c(x2 + 2cx − 1) (1 + c2)(1 + x2)2 . Trên (0, 1 c ) thì f (x) = 0 có nghiệm duy nhất là x0 = −c + √ c2 + 1 (2.7) với 0 < x0 < 1 c . Qua x0 thì f (x) đổi dấu từ dương sang âm nên f(x) đạt cực đại tại x0 nên f(x) ≤ f(x0) = 1 + c √ c2 + 1 . Từ đó theo (2.6) ta có P = 2f(x) − 2 + 3 c2 + 1 ≤ 2c √ c2 + 1 + 3 c2 + 1 = g(c). Xét hàm số g(c) với c > 0. Ta có g (c) = 2(1 − 8c2 ) (c2 + 1)(3c + √ c2 + 1) . Với c > 0, thì ta thấy g (c) = 0 tại c0 = 1√ 8 và g (c) đổi dấu từ dương sang âm khi đi qua c0 nên g(c0) là giá trị cực đại, suy ra P ≤ g( 1 √ 8 ) = 10 3 . Ta thấy, giá trị P = 10 3 đạt được khi c = 1√ 8 , a = 1√ 2 , b = √ 2. Vậy giá trị lớn nhất của P là 10 3 .
  • 25. 25 Bài toán 15. (VMO, 2001) Xét các số thực dương x, y, z thỏa mãn hệ điều kiện    2 5 ≤ z ≤ min{x, y} xz ≥ 4 15 yz ≥ 1 5 . (2.8) Hãy tìm giá trị lớn nhất của biểu thức P(x, y, z) = 1 x + 2 y + 3 z . Chứng minh. Từ hệ bất phương trình (2.8) suy ra x ≥ max{z, 4 15z } (2.9) a) Xét hàm số f(x) = 1 x + 1 z với x > 0 và tham số z ≥ 2 5 . Xét hai trường hợp Trường hợp 1. Nếu z ≥ 2√ 15 , khi đó theo (2.9) thì x ≥ z ≥ 4 15z , nên f(x) ≤ 1 z + 1 z = 2 z ≤ 15. (2.10) Trường hợp 2. Nếu 2 5 ≤ z ≤ 2√ 15 , khi đó theo (2.9) thì x ≥ 4 15z ≥ z, nên f(x) ≤ 15z 4 + 1 z = g(z). Xét hàm số g(z) với 2 5 ≤ z ≤ 2√ 15 . Ta có g (z) = 15 4 − 1 z2 < 0 khi và chỉ khi z < 2√ 15 . Do đó g(z) là hàm giảm và f(z) ≤ g(z) ≤ g( 2 5 ) = 4. (2.11) So sánh (2.10) và (2.11) ta có 1 x + 1 z ≤ 4
  • 26. 26 và 1 x + 1 z = 4 khi x = 2 3 , z = 2 5 . Xét hàm số h(y) = 1 y + 1 z với tham số z ≥ 2 5 . Từ điều kiện (2.8) suy ra y ≥ max{z, 1 5z }. Lập luận tương tự phần a) ta được • Nếu z ≥ 1√ 5 thì h(y) ≤ 2 √ 5. (2.12) • Nếu 2 5 ≤ z ≤ 1√ 5 thì h(y) ≤ 9 2 . (2.13) So sánh (2.12) và (2.13) ta có 1 y + 1 z ≤ 9 2 và 1 y + 1 z = 9 2 khi x = 2 5 , y = 1 2 . So sánh kết quả phần a) và b) ta có P(x, y, z) = ( 1 x + 1 y ) + 2( 1 y + 1 z ) ≤ 4 + 2 9 2 = 13. Đẳng thức xảy ra khi và chỉ khi x = 2 3 , y = 1 2 , z = 2 5 . Vậy giá trị lớn nhất của P là 13. Bài toán 16. (Đề thi chọn ĐTQG, 2001) Xét các số thực dương a, b, c thỏa mãn 21ab + 2bc + 8ac ≤ 12. Tìm giá trị nhỏ nhất của biểu thức P(a, b, c) = 1 a + 2 b + 3 c .
  • 27. 27 Chứng minh. Ta đổi biến x = 1 a , y = 1 b , z = 1 c thì giả thiết của bài toán đã cho trở thành 2x + 8y + 21z ≤ 12xyz. Ta cần tìm giá trị nhỏ nhất của biểu thức P(x, y, z) = x + 2y + 3z. Từ giả thiết z(12xy − 21 ≥ 2x + 8y) > 0 suy ra z ≥ 2x + 8y 12xy − 21 với x > 7 4y . (2.14) Suy ra P(x, y, z) ≥ x + 2y + 2x + 8y 4xy − 7 . Xét hàm số f(x) = x + 2x + 8y 4xy − 7 = 4x2 y − 5x + 8y 4xy − 7 với biến x > 7 4y và y là tham số dương. Ta có f (x) = 16x2 y2 − 56xy − 32y2 + 35 (4xy − 7)2 . Trên đoạn (7y 4 , +∞) thì f (x) = 0 có nghiệm duy nhất là x0 = − 7y 4 + 32y2 + 14 4y và qua x0 thì f (x) đổi dấu từ âm sang dương nên f(x) đạt cực tiểu tại x0 nên f(x) ≥ f(x0) = 2x0 − 5 4y . Suy ra P(x, y, z) ≥ f(x) + 2y ≥ f(x0) + 2y. Xét hàm số g(y) = 2y + 9 4y + 1 2y 32y2 + 14. Ta có g (y) = 0 ⇔ (8y2 − 9) 32y2 + 14 − 28 = 0.
  • 28. 28 Đặt t = 32y2 + 14 với t > 0, thì phương trình trên trở thành t3 − 50t − 112 = 0. Hay (t − 8)(t2 + 8t + 14) = 0 ⇔ t = 8 hoặc t = 5 4 . Từ đó g (5 4 ) = 0 và g (y) đổi dấu từ âm sang dương khi đi qua điểm 5 4 nên g(y) đạt cực tiểu tại y0 = 5 4 lúc đó ta có P(x, y, z) ≥ g(y) ≥ g( 5 4 ) = 15 2 . Dấu đẳng thức xảy ra khi y = 5 4 , x = 3, z = 3 2 hay a = 1 3 , b = 4 3 , c = 2 3 . Vậy giá trị nhỏ nhất của P là 15 2 . Bài toán 17. Tìm giá trị nhỏ nhất của biểu thức Q = −2xy2 + x2 y trên miền E = {(x, y) : 0 ≤ x ≤ 2, 0 ≤ y ≤ 1}. Chứng minh. Coi x là tham số ta có hàm số f(y) = −2xy2 + x2 y với y ∈ [0, 1]. Ta có f (y) = −4xy + x2 và f (y) = 0 ⇔ y = x/4. Ta suy ra f(y) ≤ max{f(0), f(1)} ∀y ∈ [0, 1]. Hay f(y) ≥ min{0, x2 − 2x} ∀y ∈ [0, 1]. Khi 0 ≤ x ≤ 2 thì x2 − 2x ≤ 0 suy ra f(y) ≥ x2 − 2x = g(x).
  • 29. 29 Khảo sát hàm số g(x) = x2 − 2x x ∈ [0, 2] ta tìm được min g(x) = g(1) = −1. Kết quả giá trị nhỏ nhất của Q là −1 đạt khi x = 1 , y = 1. Bài toán 18. Xét các số thực dương x, y, z thỏa mãn điều kiện 12xyz ≥ 2x+8y +21z. Tìm giá trị nhỏ nhất của biểu thức Q = x + 2y + 3z. Chứng minh. Từ giả thiết z(12xy − 21) ≥ 2x + 8y > 0 suy ra z ≥ 2x + 8y 12xy − 21 , x > 7 4y . Do đó Q ≥ x + 2y + 2x + 8y 4xy − 7 . Xét hàm số f(x) = x + 2x + 8y 4xy − 7 với x > 7 4y . Khi đó f (x) = 1 − 32y2 + 14 (4xy − 7)2 trên khoảng ( 7 4y , +∞) thì f (x) = 0 ⇔ x = x0 = 7 4y + 32y2 + 14 4y và f (x) đổi dấu từ âm qua dương khi x qua x0. Do vậy f(x) ≥ f(x0) = 2x0 − 5 4y . Suy ra Q ≥ f(x) + 2y ≥ 2x0 − 5 4y + 2y. Đặt g(y) = 2x0 − 5 4y + 2y = 2y + 9 4y + 32y2 + 14 2y . Khi đó g (y) = 0 ⇔ (8y2 − 9) 32y2 + 14 = 28. Đặt t = 32y2 + 14 thì t > 0 và ta có phương trình t3 − 50t − 112 = 0. Phương trình này chỉ có một nghiệm dương t = 8 từ đó y = y0 = 5 4 . Ta cũng có g(y) ≥ g(y0) = 15 2 .
  • 30. 30 Vậy Q ≥ g(y) ≥ g(y0) = 15 2 . Đẳng thức xảy ra khi x = 3, y = 5/4, z = 2/3. Vậy giá trị nhỏ nhất của Q là 15 2 . 2.3 Đặt biến phụ chuyển về đánh giá hàm số một biến Bài toán 19. Giả sử x, y là hai số thực không âm thỏa mãn x + y = 4. Tìm giá trị lớn nhất, giá trị nhỏ nhất của biểu thức P = (x3 − 1)(y3 − 1). Chứng minh. Ta có P = (x3 − 1)(y3 − 1) = x3 y3 − (x3 + y3 ) + 1 = (xy)3 − [(x + y)3 − 3xy(x + y)] + 1. Mà x + y = 4 nên f = (xy)3 − [(x + y)3 − 3xy(x + y)] + 1 = (xy)3 + 12xy − 63. Đặt t = xy ta đưa về hàm theo một biến t là f(t) = t3 + 12t − 63. Bây giờ ta cần tìm miền biến thiên của t như sau 0 ≤ t = xy ≤ ( x + y 2 )2 = 4. Vậy t ∈ [0, 4]. Dùng đạo hàm ta tìm được giá trị lớn nhất và giá trị nhỏ nhất của hàm f(t) = t3 + 12t − 63, trên đoạn [0, 4] ta có −63 ≤ f(t) ≤ 49 ∀t ∈ [0, 4]. Vậy giá trị lớn nhất, giá trị nhỏ nhất của biểu thức P = (x3 − 1)(y3 − 1) lần lượt là 49 khi t = 4, là −63 khi t = 0.
  • 31. 31 Bài toán 20. Giả sử x, y, z là hai số thực không âm thỏa mãn x2 + y2 + z2 = 3. Tìm giá trị lớn nhất của biểu thức P = xy + yz + zx + 5 x + y + z . Chứng minh. Đặt t = x + y + z. Khi đó t2 = x2 + y2 + z2 + 2(xy + yz + zx). Kết hợp với giả thiết x2 + y2 + z2 = 3 ta nhận được xy + yz + zx = t2 − 3 2 . Khi đó P = t2 2 + 5 t − 3 2 = f(t) Do 0 ≤ xy + yz + zx ≤ x2 + y2 + z2 = 3 nên √ 3 ≤ t ≤ 3. Ta thấy f (t) = t − 5 t2 > 0 ∀t ∈ [ √ 3, 3]. Suy ra f(t) đồng biến trên [ √ 3, 3]. Do đó f(t) ≤ f(3) = 14 3 . Dấu đẳng thức xảy ra khi t = 3 hay x = y = z = 1. Vậy giá trị lớn nhất của biểu thức P là 14 3 khi x = y = z = 1. Bài toán 21. Giả sử x, y ≥ 0 là hai số thực thỏa mãn x+y = 1. Tìm giá trị lớn nhất, giá trị nhỏ nhất của biểu thức P = (x2 − 1)(y2 − 1) − x2 + y2 + 1. Chứng minh. Đặt t = xy thì 0 ≤ t = xy ≤ ( x + y 2 )2 = 1 4 . Do vậy t ∈ [0, 1 4 ]. Khi đó P = f(t) = t2 + 2t − √ 2 − 2t. Ta có f (t) > 0 nên hàm số đồng biến. Suy ra max t∈[0, 1 4 ] f(t) = f( 1 4 ), min t∈[0, 1 4 ] f(t) = f(0). Vậy giá trị lớn nhất P là 9 16 − √ 3 2 đạt được khi x = y = 1 2 , giá trị nhỏ nhất của P là − √ 2 đạt được khi (x, y) = (0, 1), (1, 0).
  • 32. 32 Bài toán 22. Giả sử hai số x, y khác 0 thay đổi thỏa mãn (x + y)xy = x2 + y2 − xy. (2.15) Chứng minh rằng giá trị lớn nhất của 1 x3 + 1 y3 là 16. Chứng minh. Ta đặt x + y = u, xy = v. Khi đó (2.15) trở thành uv = u2 − 3v. Suy ra v = u2 u + 3 ( do u = 3). Ta có 1 x3 + 1 y3 = x3 + y3 (xy)3 = u3 − 3uv u3 = u(u2 − 3v) v3 = u2 v2 = ( u + 3 u )2 . Vì u2 ≥ 4v ⇒ u2 ≥ 4u2 u + 3 ⇒ 4 u + 3 ≤ 1 ⇒ u − 1 u + 3 ≥ 0. Điều này dẫn đến u ≥ 1 hoặc u ≤ −3. Vì từ (2.15) suy ra u+3 u > 0 nên ta chỉ cần chứng minh u − 1 u + 3 ≤ 4 với u ≥ 1 hoặc u ≤ −3. Xét hàm số f(u) = u + 3 u . Khi đó f (u) = −3 u2 < 0 suy ra f(x) nghịch biến trên mỗi khoảng (−∞, −3) và [1, +∞) và do đó f(u) ≤ f(1) = 4 ∀u ≥ 1.
  • 33. 33 Hơn nữa 0 < f(−3) < f(u) < 1 ∀u < −3. Do đó 1 x3 + 1 y3 ≤ 16. Ta thấy khi x = y = 1 2 thì 1 x3 + 1 y3 = 16. Vậy giá trị lớn nhất của 1 x3 + 1 y3 là 16. Bài toán 23. Giả sử x, y là các số thực thay đổi thỏa mãn điều kiện x2 + y2 + xy = 1. Tìm giá trị lớn nhất và giá trị nhỏ nhất của biểu thức S = x2 y − xy2 . Chứng minh. Ta có S2 = (xy)2 (x2 + y2 − 2xy) = (xy)2 (1 − 3xy). Đặt t = xy. Khi đó, từ x2 + y2 + xy = 1, ta có 1 − 3xy = (x − y)2 ≥ 0, (x + y)2 = 1 + xy. Điều này dẫn đến 1 3 ≥ t ≥ −1. Ta có S2 = f(t) = t2 (1 − 3t) với t ∈ [−1, 1 3 ]. Khi đó f (t) = 2t − 9t2 và do đó f (t) = 0 khi t = 0 hoặc t = 2 9 . Hơn nữa f(−1) = 4, f(0) = f( 1 3 ) = 0, f( 2 9 ) = 4 243 .
  • 34. 34 Vậy S2 ≤ 4. Hay −2 ≤ S ≤ 2. Ta thấy S = 2 khi x = −1, y = 1, và S = −2 khi x = 1, y = −1. Vậy giá trị lớn nhất và giá trị nhỏ nhất của S lần lượt là 2 và −2. Bài toán 24. Giả sử x, y ∈ R và x, y > 1. Tìm giá trị nhỏ nhất của P = (x3 + y3 ) − (x2 + y2 ) (x − 1)(y − 1) . Chứng minh. Biến đổi ta có P = (x + y)3 − (x + y)2 − xy(3(x + y) − 2) xy − (x + y) + 1 . Đặt t = x + y. Vì x, y > 1 nên t > 2. Ta được P = t3 − t2 − xy(3t − 2) xy − t + 1 . Áp dụng bất đẳng thức xy ≤ (x+y)2 4 , ta có xy ≤ t2 4 . Mặt khác xy > 0 và 3t − 2 > 0 nên P = t3 − t2 − xy(3t − 2) xy − t + 1 ≥ t3 − t2 − t2 4 (3t − 2) t2 4 − t + 1 = t2 t − 2 . Khảo sát hàm số f(t) = t2 t − 2 trên miền t ∈ (2, +∞) ta được kết quả min t∈(2,+∞) f(t) = 8 đạt được khi x = y = 2. Vậy giá trị nhỏ nhất của S là 8. Bài toán 25. (Trích đề thi Đại học khối A năm 2006) Giả sử hai số thực x, y = 0 thay đổi thỏa mãn (x + y)xy = x2 + y2 − xy. Tìm giá trị lớn nhất của biểu thức P = 1 x3 + 1 y3 .
  • 35. 35 Chứng minh. Từ giả thiết ta có 1 x + 1 y = 1 x2 + 1 y2 − 1 x 1 y . Đặt a = 1 x , b = 1 y . Ta được a + b = a2 + b2 − ab ⇒ a + b = (a + b)2 − 3ab. Đặt t = a + b thì ab = t2 − t 3 . Khi đó P = a3 + b3 = (a + b)3 − 3ab(a + b) = t3 − (t2 − t)t = t2 . Tìm miền của t a + b = (a + b)2 − 3ab ⇒ (a + b)2 − (a + b) = 3ab ≤ 3 (a + b)2 4 . Do vậy t2 − t ≤ 3t2 4 ⇒ 0 ≤ t ≤ 4. Suy ra P ≤ 16. Vậy giá trị lớn nhất của P là 16 đạt được khi x = y = 1 2 . Bài toán 26. Giả sử các số thực dương x, y, z thỏa mãn    x + y + z = 3 xy + yz + zx = 1. Tìm giá trị nhỏ nhất, giá trị lớn nhất của biểu thức Q = x4 + y4 + z4 . Chứng minh. Đặt s = xyz thì Q = 47 + s. Từ hệ (26) ta có    y + z = 3 − x yz = x2 − 3x + 1.
  • 36. 36 Vì (3 − x)2 ≥ 4(x2 − 3x + 1) nên 3x2 − 6x − 5 ≤ 0 kết hợp x ≥ 0 ta được x ∈ [0, 3 + 2 √ 6 3 ]. Khảo sát hàm số s = g(x) = x3 − 3x2 + x với x ∈ [0, 3 + 2 √ 6 3 ] và kết hợp với s ≥ 0 ta được 0 ≤ s ≤ 4 √ 6 − 9 9 . Vì Q = 47 + 12s nên 47 ≤ Q ≤ 105 + 16 √ 6 3 . Ta thấy với x = 0, (y, z) = ((3 + √ 5)/2, (3 − √ 5)/2), ((3 − √ 5)/2, (3 + √ 5)/2) thì Q = 47; với x = 3+2 √ 6 3 , y = z = (3 − √ 6)/3 thì Q = 105+16 √ 6 3 . Vậy giá trị nhỏ nhất của Q là 47, giá trị lớn nhất là 105+16 √ 6 3 . Bài toán 27. Giả sử các số thực x, y, z > 0 thỏa mãn    xy + yz + zx = 8 xyz = 4. Tìm giá trị nhỏ nhất, giá trị lớn nhất của biểu thức Q = x4 + y4 + z4 . Chứng minh. Từ giả thiết ta có    y + z = 1 x (8 − 4 x ) yz = 4 x . Vì (y + z)2 ≥ 4yz nên 1 x2 (8 − 4 x )2 ≥ 4 4 x ⇒ 1 x (8 − 4 x )2 ≥ 16 ( do x > 0). Hay x3 − 4x2 + 4x − 1 ≤ 0 ⇒ (x − 1)(x2 − 3x + 1) ≤ 0.
  • 37. 37 Kết hợp với x > 0 ta được x ∈ (0, 3 − √ 5 2 ] ∪ [1, 3 + √ 5 2 ]. Đặt s = x + y + z thì s = g(x) = x + 8 x − 4 x2 với x ∈ (0, 3 − √ 5 2 ] ∪ [1, 3 + √ 5 2 ]. Khảo sát hàm số g(x) với x ∈ (0, 3− √ 5 2 ] ∪ [1, 3+ √ 5 2 ] và chú ý s > 0 ta được 5 ≤ s ≤ 5 √ 5 − 1 2 . Mặt khác Q = f(s) = s4 − 32s2 + 16s + 128 với s ∈ [5, 5 √ 5 − 1 2 ]. Dùng đạo hàm lập bảng biến thiên hàm số f(s), ta được g(5) ≤ f(s) ≤ f( 5 √ 5 − 1 2 ). Vậy giá trị nhỏ nhất, giá trị lớn nhất của Q lần lượt là 33 và 271−75 √ 5 2 . Bài toán 28. Giả sử các số thực x, y, z > 0 thỏa mãn x + y + z ≤ 2. Tìm giá trị nhỏ nhất của biểu thức Q = 4x2 + 1 x2 + 4y2 + 1 y2 + 4z2 + 1 z2 . Chứng minh. Trước hết ta chứng minh Q ≥ 4(x + y + z)2 + ( 1 x + 1 y + 1 z )2. Thật vậy, áp dụng bất đẳng thức 1 x + 1 y + 1 z ≥ 9 x + y + z ta suy ra Q ≥ 4(x + y + z)2 + 81 (x + y + z)2 .
  • 38. 38 Đặt t = (x + y + z)2 thì t ∈ (0, 4]. Khảo sát hàm số f(t) = 4t + 81 t với t ∈ (0, 4]. ta được mint∈(0,4] f(t) = f(4) = 145 4 suy ra Q ≥ √ 145 2 . Đẳng thức xảy ra khi x = y = z = 2 3 . Vậy giá trị nhỏ nhất của Q là √ 145 2 . Bài toán 29. (Trích đề thi Đại học khối A năm 2003) Giả sử x, y, z > 0, x+y+z ≤ 1. Chứng minh rằng giá trị nhỏ nhất của x2 + 1 x2 + y2 + 1 y2 + z2 + 1 z2 là √ 82. Chứng minh. Đặt P = x2 + 1 x2 + y2 + 1 y2 + z2 + 1 z2 . Ta có P = x2 + 1 x2 + y2 + 1 y2 + z2 + 1 z2 ≥ (x + y + z)2 + ( 1 x + 1 y + 1 z )2 ≥ (3 3 √ xyz)2 + ( 3 3 √ xyz )2. Đặt t = 3 √ xyz suy ra P ≥ f(t) = 9t2 + 9 t2 Tìm miền của t 0 < t = 3 √ xyz ≤ x + y + z 3 ≤ 1 3 . Vậy t ∈ (0, 1 3 ].
  • 39. 39 Khảo sát hàm f(t) = 9t2 + 9 t2 với t ∈ (0, 1 3 ] ta dễ dàng suy ra được min P = min t∈(0, 1 3 ] f(t) = √ 82. Vậy giá trị nhỏ nhất của x2 + 1 x2 + y2 + 1 y2 + z2 + 1 z2 là √ 82 đạt được khi x = y = z = 1 3 . Bài toán 30. Giả sử ba số thực a, b, c > 0 thỏa mãn a + b + c ≤ 3 2 . Tìm giá trị nhỏ nhất của P = a2 + b2 + c2 + 1 a2b + 1 b2c + 1 c2a . Chứng minh. Ta đặt t = 3 √ abc khi đó 0 < t ≤ a+b+c 3 ≤ 1 2 . Đưa về hàm một biến P ≥ f(t) = 3t2 + 3 t3 với t ∈ (0, 1 2 ]. Khi đó f (t) = 6t − 9 t4 < 0 ∀t ∈ (0, 1 2 ]. Hay f(t) là hàm nghịch biến trên (0, 1 2 ], do đó f(t) ≥ f( 1 2 ) = 9 8 . Vậy giá trị nhỏ nhất của P là 9 8 đạt được khi a = b = b = 1 2 . Bài toán 31. (Thi thử đại 2012-2013. Trường THPT Kon Tum) Giả sử x, y, z là các số thực không âm thỏa x + y + z = 1. Tìm giá trị lớn nhất của biểu thức P = xy + yz + zx − 2xyz.
  • 40. 40 Chứng minh. Vì vai trò của x, y, z trong bài toán bình đẳng nên có thể giả sử x ≤ y ≤ z. Mặt khác x + y + z = 1 nên ta suy ra x ∈ [0, 1 3 ]. Ta thấy P = xy + yz + zx − 2xyz = x(y + z) + yz(1 − 2x). Ta có yz ≤ (y+z)2 4 mà x ∈ [0, 1 3 ] nên 1 − 2x > 0. Suy ra yz(1 − 2x) ≤ (y + z)2 4 (1 − 2x). Điều này dẫn đến P = x(1−x)+yz(1−2x) ≤ x(1−x)+ (y + z)2 4 (1−2x) = x(1−x)+ (1 − x)2 4 (1−2x). Khảo sát hàm f(x) = x(1 − x) + (1 − x)2 4 (1 − 2x) với x ∈ [0, 1 3 ]. Ta tìm được max P = max x∈[0, 1 3 ] f(x) = 7 27 khi x = y = z = 1 3 . Vậy giá trị lớn nhất của P là 7 27 . Bài toán 32. Giả sử x, y, z là các số thực không âm thỏa mãn x + y + z = 1. Tìm giá trị nhỏ nhất của biểu thức P = x3 + y3 + z3 + 15 4 xyz. Chứng minh. Vai trò x, yz bình đẳng nên ta có thể giả sử x ≤ y ≤ z. Từ giả thiết suy ra 0 ≤ x ≤ 1 3 và y + z = 1 − x. Áp dụng bất đẳng thức yz ≤ (y + z)2 4 và 27x 4 − 3 < 0 ta nhận được P = x3 + y3 + z3 + 15 4 xyz = x3 + (y + z)3 − 3yz(y + z) + 15 4 xyz
  • 41. 41 = x3 + (y + z)3 + yz[ 15x 4 − 3(y + z)] = x3 + (1 − x)3 + yz( 27x 4 − 3) ≥ x3 + (1 − x)3 + (y + z)2 4 ( 27x 4 − 3) = 1 16 (27x3 − 18x2 + 3x + 4). Khảo sát hàm f(x) = 1 16 (27x3 − 18x2 + 3x + 4) với 0 ≤ x ≤ 1 3 . Ta tìm được min P = min x∈[0, 1 3 ] f(x) = 1 4 khi (x, y, z) = (1 3 , 1 3 , 1 3 ) hoặc (x, y, z) = (0, 1 2 , 1 2 ) hoặc các hoán vị của nó. Vậy giá trị nhỏ nhất của P là 1 4 . Bài toán 33. (Trích đề thi thử đại học năm 2012-2013, trường THPT chuyên Nguyễn Tất Thành, Kon Tum) Giả sử x, y là các số thực không âm thay đổi và thỏa mãn điều kiện 4(x2 + y2 + xy) ≤ 1 + 2(x + y). Tìm giá trị lớn nhất của biểu thức P = xy + √ x + y − x2 − y2 . Chứng minh. Ta biến đổi P = xy+ √ x + y−(x2 +y2 ) = xy+ √ x + y−[(x+y)2 −2xy] = 3xy+ √ x + y−(x+y)2 . Đặt t = x + y. Khi đó P ≤ 3 (x + y)2 4 + √ x + y − (x + y)2 = √ x + y − (x + y)2 4 = f(t) = √ t − t2 4 . Từ giả thiết 4(x2 + y2 + xy) ≤ 1 + 2(x + y) ta suy ra 4((x + y)2 − xy) ≤ 1 + 2(x + y) và do đó 4((x + y)2 − (x + y)2 4 ) ≤ 4((x + y)2 − xy) ≤ 1 + 2(x + y). Hay 3t2 − 2t − 1 ≤ 0.
  • 42. 42 Từ đó − 1 3 ≤ t ≤ t. Mặt khác x, y không âm nên 0 ≤ t ≤ 1. Bây giờ ta đi tìm giá trị lớn nhất của f(t) = √ t − t2 4 trên miền 0 ≤ t ≤ 1. Bằng cách dùng đạo hàm và khảo sát hàm f(t) ta được max P = maxt∈[0,1] f(t) = 3 4 , khi x = y = 1 2 . Vậy giá trị lớn nhất của P là 3 4 . Bài toán 34. (Trích đề thi học sinh giỏi Toán 12, bảng A, tỉnh Nghệ An, năm 2012- 2013 ) Giả sử a, b, c là các số thực dương. Tìm giá trị nhỏ nhất của biểu thức P = 2 a + √ ab + 3 √ abc − 3 √ a + b + c . Chứng minh. Áp dụng bất đẳng thức AG-GM, ta có a + √ ab + 3 √ abc ≤ a + 1 2 √ a4b + 1 4 3 √ a4b16c ≤ a + 1 2 a + 4b 2 + 1 4 a + 4b + 16c 3 = 4 3 (a + b + c). Đẳng thức xảy ra khi và chỉ khi a = 4b = 16c. Suy ra P ≥ 3 2(a + b + c) − 3 √ a + b + c . Đặt t = a + b + c, t > 0. Khi đó ta có P ≥ f(t) = 3 2t − 3 √ t . Khảo sát hàm số f(t) = 3 2t − 3 √ t với t > 0. Ta tìm được min t>0 f(t) = − 3 2 khi và chỉ khi t = 1. Vậy giá trị nhỏ nhất của P là −3 2 đẳng thức xảy ra khi và chỉ khi a = 16 21 , b = 4 21 , c = 1 21 .
  • 43. 43 Bài toán 35. Giả sử x, y, z ∈ [1, 2]. Tìm giá trị lớn nhất của biểu thức P = (x + y + z)( 1 x + 1 y + 1 z ). Chứng minh. Ta có P = ( x y + y x ) + ( y z + z x ) + ( x z + z x ) + 3. Giả sử 1 ≤ x ≤ y ≤ z ≤ 2, suy ra (1 − x y )(1 − y z ) ≥ 0 và (1 − y x )(1 − z y ) ≥ 0. Suy ra P = ( x y + y x ) + ( y z + z x ) + ( x z + z x ) + 3 ≤ 5 + 2( x z + z x ). Đặt t = x z với t ∈ [1 2 , 1]. Khi đó P ≤ f(t) = 5 + 2(t + 1 t ) với t ∈ [1 2 , 1]. Dùng đạo hàm khảo sát hàm f(t) = 5 + 2(t + 1 t ) với t ∈ [1 2 , 1]. Ta thấy max t∈[1 2 ,1] f(t) = 10. Vậy giá trị lớn nhất của P là 10 đạt được khi x = 1, y = z = 2. Bài toán 36. Giả sử 1 ≤ x, y, z ≤ 3 và x + y + z = 6. Tìm giá trị lớn nhất của P = x2 + y2 + z2 . Chứng minh. Vai trò x, y, z trong bài toán bình đẳng nên có thể giả sử x = max{x, y, z}. Khi đó P = x2 + y2 + z2 ≤ x2 + y2 + z2 + 2(y − 1)(z − 1)
  • 44. 44 = x2 + (y + z)2 − 2(y + z) + 2 = x2 + (6 − x)2 − 2(6 − x) + 2. Đẳng thức xảy ra khi y = 1 hoặc z = 1. Xét hàm số f(x) = x2 + (6 − x)2 − 2(6 − x) + 2 với 2 ≤ x ≤ 3. Ta dễ dàng tìm được giá trị lớn nhất của f(x) bằng 14 khi x = 2 hoặc x = 3. Vậy giá trị lớn nhất của P là 14, chẳng hạn khi x = 3, y = 1, z = 2. 2.4 Đánh giá gián tiếp thông qua biểu thức bậc nhất Nếu bài toán có dạng sau cho n ∈ N và các số a1, a2, . . . an ∈ D thỏa mãn a1 + a2 + · · · + an = nα, với α ∈ D. Hàm số y = f(x) trên khoảng D không lồi và cũng không lõm trên D nhưng đồ thị vẫn “nằm trên” tiếp tuyến của nó tại D. Trong bài này không thể áp dụng được BĐT hàm lồi được nhưng vẫn có thể dùng phương pháp “tiếp tuyến” để giải quyết bài toán. Sau đây xin được trình bày một số bài toán minh họa cho phương pháp trên được trích dẫn từ một số đề thi Olympic của nước ta và các nước trên thế giới. Trong một số bài toán có thể chúng ta phải sử dụng linh hoạt các giả thiết và tính chất của các biểu thức trong bài toán để vận dụng phương pháp một cách hiệu quả nhất. Bài toán 37. ( Olimpic 30/4- 2006). Giả sử a, b, c là các số thực dương. Tìm giá trị lớn nhất của biểu thức Q = a(b + c) (b + c)2 + a2 + b(c + a) (c + a)2 + b2 + c(a + b) (a + b)2 + c2 . Chứng minh. Do Q có tính thuần nhất nên chỉ xét giá trị của Q với a + b + c = 1. Ta có Q = a(1 − a) (1 − a)2 + a2 + b(1 − b) (1 − b)2 + b2 + c(1 − c) (1 − c)2 + c2 = f(a) + f(b) + f(c) trong đó f(x) = x(1 − x) (1 − x)2 + x2 = x − x2 1 − 2x + 2x2 với x ∈ (0, 1).
  • 45. 45 Khi đó tiếp tuyến tại x = 1 3 có phương trình y = 27 25 (x − 1 3 ) + 2 5 = 27 25 x + 1 25 . Mặc dầu trong khoảng (0, 1) đồ thị (C) của hàm số y = f(x) không lồi, nhưng vẫn có f(x) ≤ 27 25 x + 1 25 ∀x ∈ (0, 1). (2.16) Thật vậy (2.16) tương đương với x − x2 2x2 − 2x + 1 ≤ 27x + 1 25 ⇒ 54x3 − 27x2 + 1 ≥ 0. Xét hàm số g(x) = 54x3 − 27x2 + 1 với x ∈ (0, 1). Khi đó g (x) = 54x(3x − 1). Lập bảng biến thiên của hàm số y = g(x) ta được kết quả g(x) ≥ 0 ∀x ∈ (0, 1). Áp dụng BĐT (2.16) cho các số a, b, c ∈ (0, 1) ta có f(a) + f(b) + f(c) ≤ 27 25 (a + b + c) + 3 25 = 6 5 . Vậy giá trị lớn nhất của biểu thức Q = a(b + c) (b + c)2 + a2 + b(c + a) (c + a)2 + b2 + c(a + b) (a + b)2 + c2 là 6 5 đẳng thức xảy ra khi a = b = c. Bài toán 38. (Hồng Kong, 2005). Giả sử a, b, c, d là các số dương thỏa mãn a + b + c + d = 1. Chứng minh rằng giá trị nhỏ nhất của 6(a3 + b3 + c3 + d3 ) − (a2 + b2 + c2 + d2 ) là 1 8 .
  • 46. 46 Chứng minh. Từ giả thiết ta có a, b, c, d ∈ (0, 1). Bây giờ ta chứng minh rằng f(a) + f(b) + f(c) + f(d) ≥ 1 8 (2.17) trong đó f(x) = 6x3 − x2 . Xét f(x) với x ∈ (0, 1). Tiếp tuyến của đồ thị hàm số y = f(x) tại x = 1 4 có phương trình là y = 5 8 x − 1 8 . Mặt khác f(x) − ( 5 8 x − 1 8 ) = 6x3 − x2 − ( 5 8 x − 1 8 ) = 1 8 (4x − 1)2 (3x + 1) ≥ 0 với mọi x ∈ (0, 1) hay f(x) ≥ 5 8 x − 1 8 . Từ đó suy ra f(a) + f(b) + f(c) + f(d) ≥ 5 8 (a + b + c + d) − 4 1 8 = 1 8 . Đẳng thức xảy ra khi a = b = c = d = 1 4 . Vậy giá trị nhỏ nhất của 6(a3 + b3 + c3 + d3 ) − (a2 + b2 + c2 + d2 ) là 1 8 . Bài toán 39. ( Mở rộng bài toán thi Olimpic Ba Lan, 1996 và Olimpic 30-4, 1999) Giả sử các số thực a, b, c thỏa mãn a + b + c = 1. Tìm giá trị lớn nhất của biểu thức a 1 + a2 + b 1 + b2 + c 1 + c2 . Chứng minh. Đặt f(x) = x 1 + x2 . Ta sẽ chứng minh rằng f(a) + f(b) + f(c) ≤ 9 10 . (2.18) Thật vậy, có f (x) = 1 − x2 (1 + x2)2 ,
  • 47. 47 hơn nữa f (x) = 0 khi x = −1 hoặc x = 1. Xét các trường hợp xảy ra Trường hợp 1. Có một số, giả sử a ∈ (−∞, −3] nên b + c ≥ 4 nên có một số, giả sử b ≥ 2. Khi đó ta có f(a) + f(b) + f(c) < 0 + 2 5 + 1 2 = 9 10 . Trường hợp 2. Có một số, giả sử a ∈ (−3, −1 3 ] . Khi đó f(a) + f(b) + f(c) ≤ − 3 10 + 1 2 + 1 2 = 7 10 < 9 10 . Trường hợp 3. Cả ba số a, b, c ∈ (−1 3 , +∞). Khi đó tiếp tuyến của đồ thị y = f(x) tại x = 1 3 có phương trình y = 18 25 x + 3 50 . Ta có f(x) − ( 18 25 x + 3 50 ) = x 1 + x2 − ( 18 25 x + 3 50 ) = (3x − 1)2 (4x + 3) 50(1 + x2) ≤ 0 ∀x > − 1 3 . Áp dụng bất đẳng thức này cho các số a, b, c > −1 3 và a + b + c = 1 ta có f(a) + f(b) + f(c) ≤ 18 25 + 3 3 50 = 9 10 . Vậy trong mọi trường hợp bất đẳng thức (2.18) đều đúng. Đẳng thức xảy ra khi a = b = c = 1 3 . Vậy giá trị lớn nhất của biểu thức a 1 + a2 + b 1 + b2 + c 1 + c2 là 9 10 . Bài toán 40. (Rumania, 2005). Giả sử các số thực dương a, b, c thỏa mãn a+b+c = 3. Chứng minh rằng giá trị nhỏ nhất của 1 a2 + 1 b2 + 1 c2 − (a2 + b2 + c2 ) là 0. Chứng minh. Ta cần chứng minh rằng 1 a2 + 1 b2 + 1 c2 − (a2 + b2 + c2 ) ≥ 0. (2.19)
  • 48. 48 Theo giả thiết a, b, c > 0 ta được a2 + b2 + c2 < (a + b + c)2 = 9. Từ đó nếu có một trong ba số, giả sử a < 1 3 thì 1 a2 + 1 b2 + 1 c2 > 9 > a2 + b2 + c2 nên (2.19) đúng. Bây giờ ta chỉ cần xét trường hợp a, b, c ≥ 1 3 . Vì a + b + c = 3 nên a, b, c ≤ 7 3 . Vậy a, b, c ∈ [ 1 3 , 7 3 ]. Xét hàm số f(x) = 1 x2 − x2 trên [ 1 3 , 7 3 ]. Tiếp tuyến của đồ thị hàm số y = f(x) tại x = 1 là y = −4x+4. Do g(x) = x2 −2x−1 = (x − 1)2 − 2 ≤ (4 3 )2 − 2 < 0 trên [1 3 , 7 3 ], nên ta có f(x) − (−4x + 4) = − (x − 1)2 (x2 − 2x − 1) x2 ≥ 0 ∀x ∈ [ 1 3 , 7 3 ]. Hay f(x) ≥ −4x + 4 ∀x ∈ [ 1 3 , 7 3 ]. Áp dụng cho các số a, b, c ∈ [1 3 , 7 3 ] ta có f(a) + f(b) + f(c) ≥ −4(a + b + c) + 4.3 = 0. Nên bất đẳng thức (2.19) được chứng minh. Đẳng thức xảy ra khi và chỉ khi a = b = c = 1. Vậy giá trị nhỏ nhất của 1 a2 + 1 b2 + 1 c2 − (a2 + b2 + c2 ) là 0. Bài toán 41. (Trung Quốc, 2005). Giả sử các số không âm a, b, c thỏa mãn a+b+c = 1. Chứng minh rằng giá trị nhỏ nhất của 10(a3 + b3 + c3 ) − 9(a5 + b5 + c5 ) (2.20) là 1.
  • 49. 49 Chứng minh. Đặt f(x) = 10x3 − 9x5 . Nên để chứng minh (2.20) ta sẽ chứng minh rằng f(a) + f(b) + f(c) ≥ 1. (2.21) Để chứng minh điều này, ta xét 2 trường hợp. Trường hợp 1. Trong ba số a, b, c có một số, giả sử a ≥ 9 10 . Khi đó thì a ∈ [ 9 10 , 1], b, c ∈ [0, 1 10 ]. Xét hàm số f(x) trên đoạn [ 9 10 , 1] ta có f (x) = 30x2 − 45x4 = 15x2 (2 − 3x2 ) ≤ 0 với mọi x ∈ [ 9 10 , 1]. Vậy f(x) nghịch biến trên đoạn này và từ đó f(a) ≥ f(1) = 1 với a ∈ [ 9 10 , 1]. Hơn nữa với b, c ∈ [0, 1 10 ] thì f(b) = 10b3 − 9b5 ≥ 0 và f(c) = 10c3 − 9c5 ≥ 0 nên f(a) + f(b) + f(c) ≥ 1 + 0 + 0 = 1 hay (2.21) đúng. Trường hợp 2. Các số a, b, c ∈ [0, 9 10 ]. Khi đó tiếp tuyến của đồ thị hàm số y = f(x) tại x = 1 3 có phương trình y = 25 9 x − 16 27 . Ta có f(x)−( 25 9 x− 16 27 ) = 10x3 −9x5 −( 25 9 x− 16 27 ) = − 1 27 (3x−1)2 (27x3 +18x2 −21x−16)). Xét hàm số g(x) = 27x3 + 18x2 − 21x − 16 trên đoạn [0, 9 10 ]. Ta có g (x) = 81x2 + 36x − 21
  • 50. 50 g (x) = 0 khi x = 1 3 hoặc x = −7 9 . Ta thấy trên đoạn [0, 9 10 ] thì g(x) < 0 nên f(x) − ( 25 9 x − 16 27 ) ≥ 0 hay f(x) ≥ 25 9 x − 16 27 ∀x ∈ [0, 9 10 ]. Áp dụng cho các số a, b, c ∈ [0, 9 10 ] và a + b + c = 1 ta có f(a) + f(b) + f(c) ≥ 25 9 (a + b + c) − 3 16 27 = 1. Hay (2.21) đúng. Vậy giá trị nhỏ nhất của 10(a3 + b3 + c3 ) − 9(a5 + b5 + c5 ) là 1. Đẳng thức xảy ra khi a = b = c = 1 3 hoặc (a, b, c) là một hoán vị bất kì của bộ (1, 0, 0). Bài toán 42. (Moldova,2005) . Giả sử các số dương a, b, c thỏa mãn a4 + b4 + c4 = 3. Tìm giá trị lớn nhất của biểu thức 1 4 − ab + 1 4 − bc + 1 4 − ca . Chứng minh. Vì ab ≤ a2+b2 2 nên 1 4 − ab ≤ 2 8 − (a2 + b2) do đó 1 4 − ab + 1 4 − bc + 1 4 − ca ≤ 2 8 − (a2 + b2) + 2 8 − (b2 + c2) + 2 8 − (c2 + a2) . Để vận dụng giả thiết a4 + b4 + c4 = 3 ta đặt x = (b2 + c2 )2 , y = (c2 + a2 )2 , z = (a2 + b2 )2 thì ta có x, y, z > 0 và x + y + z = (b2 + c2 )2 + (c2 + a2 )2 + (a2 + b2 )2 ≤ 4(a4 + b4 + c4 ) = 12. Ta sẽ chứng minh 1 8 − √ x + 1 8 − √ y + 1 8 − √ z ≤ 1 2 . (2.22)
  • 51. 51 Xét hàm số f(t) = 1 8 − √ t với t ∈ (0, 12). Phương trình tiếp tuyến của đồ thị y = f(t) tại t = 4 có phương trình y = 1 144 t + 5 36 . Hơn nữa ta có 1 8 − √ t − ( 1 144 t + 5 36 ) = − 1 144 ( √ t − 2)2 (4 − √ t) ≤ 0 với mọi t ∈ (0, 12). Vậy f(t) ≤ 1 144 t + 5 36 . Từ đó f(x) + f(y) + f(z) ≤ 1 144 (x + y + z) + 3 5 36 ≤ 1 144 12 + 3 5 36 = 1 2 . Vậy giá trị lớn nhất của biểu thức 1 4 − ab + 1 4 − bc + 1 4 − ca là 1 2 . Đẳng thức xảy ra khi a = b = c = 1. 2.5 Phương pháp sử dụng tính chất của hàm lồi, hàm lõm Bài toán 43. (Bất đẳng thức Karamata). Cho hai dãy số {xk, yk ∈ I(a, b), k = 1, 2, . . . , n} thỏa mãn các điều kiện x1 ≥ x2 ≥ · · · ≥ xn, y1 ≥ y2 ≥ · · · ≥ yn và    x1 ≥ y1 x1 + x2 ≥ y1 + y2 . . . . . . x1 + x2 + . . . xn−1 ≥ y1 + y2 + . . . yn−1 x1 + x2 + . . . xn = y1 + y2 + . . . yn (2.23)
  • 52. 52 Khi đó, ứng với mọi hàm lồi f(x) (f (x) ≥ 0) trên I(a, b), ta đều có f(x1) + f(x2) + · · · + f(xn) ≥ f(y1) + f(y2) + · · · + f(yn). (2.24) Ta cũng có phát biểu tương tự đối với hàm lõm bằng cách đổi chiều dấu bất đẳng thức. Chứng minh. Sử dụng biểu diễn đối với hàm lồi f(x1) + f(x2) + · · · + f(xn) = min t1,...,tn∈I(a,b) [ n i=1 f(t1) n i=1 f(xi − t1)f (ti)]. (2.25) Không mất tính tổng quát, ta giả thiết bộ số t1, . . . , tn ∈ I(a, b) cũng là một bộ số giảm, tức là t1 ≥ t1 ≥ ... ≥ tn. Khi đó, để chứng minh (2.25), ta chỉ cần chứng minh rằng x1f (t1) + x2f (t2) + · · · + xn1f (tn) ≥ ≥ y1f (t1) + y2f (t2) + · · · + ynf (tn). (2.26) Sử dụng biến đổi Abel x1f (t1) + x2f (t2) + · · · + xn1f (tn) = = S1[f (t1) − f (t2)] + S2[f (t2) − f (t3)] + · · · + + Sn−1[f (tn−1 − f (tn)] + Snf (tn) (2.27) với Sk(x) := x1 + x2 + · · · + xk. Vì rằng f (x) ≥ 0 nên f (xk) ≤ f (xk−1). Mặt khác, do Sk(x) ≥ Sk(y) (k = 1, 2, . . . , n − 1) và Sn(x) = Sn(y), ta thu được ngay (2.26). Chứng minh được hoàn thành. Hệ quả 2.1 (Bất đẳng thức Jensen). Với mọi hàm lồi f(x) trên I(a, b) và với mọi xi ∈ I(a, b) (i = 1, 2, . . . , n), ta luôn có bất đẳng thức f(x1) + f(x2) + · · · + f(xn) n ≥ f( x1 + x2 + · · · + xn n ).
  • 53. 53 Chứng minh. Do tính chất đối xứng, không mất tính tổng quát, ta có thể giả sử x1 ≥ x2 ≥ · · · ≥ xn. Khi đó, ta có    x1 ≥ x x1 + x2 ≥ 2x . . . . . . x1 + x2 + · · · + xn−1 ≥ (n − 1)x x1 + x2 + · · · + xn = nx (2.28) trong đó x = x1 + x2 + · · · + xn n . Theo bất đẳng thức Karamata, ta có f(x1) + f(x2) + · · · + f(xn) ≥ nf( x1 + x2 + · · · + xn n ). Chứng minh được hoàn thành. Ở phần tiếp theo, luận văn trình bày một số áp dụng của bất đẳng thức Karamata và các hệ quả của nó. Bài toán 44. Cho 2n số thực dương ai, bi (i = 1, 2, . . . , n) thỏa mãn các điều kiện    a1 ≥ a2 ≥ · · · ≥ an b1 ≥ b2 ≥ · · · ≥ bn a1 ≥ b1, a1a2 ≥ b1b2, . . . , a1a2 . . . an = b1b2 . . . bn. Chứng minh rằng a1 + a2 + · · · + an ≥ b1 + b2 + · · · + bn. Chứng minh. Đặt xi = ln ai, yi = ln bi (i = 1, 2, . . . , n). Với các điều kiện đã cho, ta
  • 54. 54 có    x1 ≥ y1 x1 + x2 ≥ y1 + y2 . . . . . . x1 + x2 + . . . xn−1 ≥ y1 + y2 + . . . yn−1 x1 + x2 + . . . xn = y1 + y2 + . . . yn Xét hàm số f(x) = ex với x ∈ (0, +∞). Ta có f (x) = ex > 0 ∀x ∈ (0, +∞) nên hàm số f(x) lồi trên khoảng (0, +∞). Khi đó, theo bất đẳng thức Karamata, ta có ex1 + ex2 + · · · + exn ≥ ey1 + ey2 + · · · + eyn hay a1 + a2 + · · · + an ≥ b1 + b2 + · · · + bn. Chứng minh được hoàn thành. Bài toán 45. Giả sử các số thực a, b, c thỏa mãn    0 ≤ c ≤ b ≤ a ≤ 8 a + b ≤ 13 a + b + c = 15. Tìm giá trị lớn nhất và nhỏ nhất của biểu thức M = a2 + b2 + c2 . Chứng minh. Từ giả thiết, ta có    0 ≤ a ≤ 8 a + b ≤ 8 + 5 a + b + c = 8 + 5 + 2. Xét hàm số f(x) = x2 , ta có f (x) = 2 > 0 ∀x ∈ R nên hàm số f(x) lồi thực sự trên R. Do đó, theo bất đẳng thức Karamata, ta có f(a) + f(b) + f(c) ≤ f(8) + f(5) + f(2)
  • 55. 55 hay a2 + b2 + c2 ≤ 64 + 25 + 4 = 93. Đẳng thức xảy ra khi a = 8, b = 5, c = 2 Vậy max M = 93 đạt được khi a = 8, b = 5, c = 2. Áp dụng bất đẳng thức B-C-S cho hai bộ số (a, b, c) và (1, 1, 1), ta có (1a + 1b + 1c)2 ≤ (12 + 12 + 12 )(a2 + b2 + c2 ). Suy ra (a + b + c)2 ≤ 3(a2 + b2 + c2 ). Nên ta được 152 ≤ 3M hay M ≥ 75. Đẳng thức xảy ra khi a = b = c = 5. Vậy min M = 75 đạt được khi a = b = c = 5. Bài toán 46. Giả sử A, B, C là 3 góc của một tam giác nhọn. Chứng minh rằng giá trị lớn nhất của cos A + cos B + cos C là 3 2 . Chứng minh. Không mất tính tổng quát, ta coi A ≥ B ≥ C. Khi đó A ≥ π 3 , C ≤ π 3 và A + B = π − C ≥ 2π 3 nên    A ≥ π 3 A + B ≥ π 3 + π 3 A + B + C = π 3 + π 3 + π 3 .
  • 56. 56 Xét hàm số f(x) = cos x với x ∈ [0, π 2 ]. Ta có f (x) = − cos x < 0 ∀x ∈ [0, π 2 ] nên hàm số f(x) lõm trên đoạn [0, π 2 ]. Khi đó, theo bất đẳng thức Karamata, ta có f(A) + f(B) + f(C) ≤ 3f( π 3 ) hay cos A + cos B + cos C ≤ 3 2 . Vậy giá trị lớn nhất của cos A + cos B + cos C là 3 2 đạt được khi A = B = C = π 3 . Bài toán 47. Giả sử tam giác ABC không nhọn. Chứng minh rằng giá trị nhỏ nhất của tan A 2 + tan B 2 + tan C 2 là 2 √ 2 − 1. Chứng minh. Không mất tính tổng quát, ta coi A ≥ B ≥ C. Khi đó    A ≥ π 2 A + B ≥ π 2 + π 4 A + B + C = π 2 + π 4 + π 4 . hay    A 2 ≥ π 4 A 2 + B 2 ≥ π 4 + π 8 A 2 + B 2 + C 2 = π 4 + π 8 + π 8 . Xét hàm số f(x) = tan x với x ∈ (0, π 2 ). Ta có f (x) = 2 sin x cos3 x ≥ 0 ∀x ∈ (0, π 2 ) nên hàm số f(x) lồi trên khoảng (0, π 2 ). Khi đó, áp dụng bất đẳng thức Karamata, ta được tan A 2 + tan B 2 + tan C 2 ≥ tan π 4 + tan π 8 + tan π 8 .
  • 57. 57 Để ý rằng tan π 8 = √ 2 − 1 nên tan π 4 + tan π 8 + tan π 8 = 2 √ 2 − 1. Vậy tan A 2 + tan B 2 + tan C 2 ≥ 2 √ 2 − 1. Vậy giá trị nhỏ nhất của tan A 2 + tan B 2 + tan C 2 là 2 √ 2 − 1 đạt được khi (A, B, C) = (π 2 , π 4 , π 4 ) và các hoán vị của nó. Bài toán 48. (IMO 2000). Giả sử các số dương a, b, c thỏa mãn điều kiện abc = 1. Chứng minh rằng (a − 1 + 1 b )(b − 1 + 1 c )(c − 1 + 1 a ) ≤ 1. Chứng minh. Vì abc = 1 nên ta đặt a = x y , b = y z , c = z x với x, y, z > 0. Ta viết bất đẳng thức đã cho theo x, y, z ta có ( x y − 1 + z y )( y z b − 1 + x z )( z x − 1 + y x ) ≤ 1. Hay (x − y + z)(y − z + x)(z − x + y) ≤ xyz. Để ý rằng (x−y+z)+(y−z+x) = 2x > 0 do đó trong ba số x−y+z, y−z+x, z−x+y không thể có trường hợp hai số cùng âm. Nếu trong ba số trên có một hoặc ba số âm, hiển nhiên ta có bất đẳng thức cần chứng minh. Trường hợp cả ba số đó đều dương, bằng cách lấy lôgarit hai vế với cơ số e, ta được ln(x − y + z) + ln(y − z + x) + ln(z − x + y) ≤ ln x + ln y + ln z. Không mất tính tổng quát, ta coi x ≥ y ≥ z. Khi đó, ta có x + y − z ≥ x, (x+y−z)+(x−y+z) = 2x ≥ x+y, (x+y−z)+(x−y+z)+(z −x+y) = x+y+z.
  • 58. 58 Xét hàm số f(x) = ln x với x > 0. Ta có f (x) = − 1 x2 < 0 ∀x > 0 nên hàm số f(x) lõm trên khoảng (0, +∞). Khi đó theo bất đẳng thức Karamata, ta có ln(x − y + z) + ln(y − z + x) + ln(z − x + y) ≤ ln x + ln y + ln z. Đẳng thức xảy ra khi và chỉ khi x = y = z hay a = b = c.
  • 59. Chương 3 Cực trị hàm nhiều biến 3.1 Cực trị tự do Sau đây, luận văn xin trình bày về cực trị tự do của hàm nhiều biến được tham khảo trong [3]. Giả sử z = f(x1, . . . , xn) là một hàm xác định và liên tục ở trong miền D mở, M(a1, . . . , an) ∈ D. Ta nói rằng hàm f(x1, . . . , xn) đạt được giá trị cực đại (cực tiểu) tại M nếu tại mọi điểm (x1, . . . , xn) thuộc một lân cận nào đó của M(a1, . . . , an) thì f(x1, . . . , xn) ≤ f(a1, . . . , an) ( tương ứng f(x1, . . . , xn) ≥ f(a1, . . . , an)). Giá trị cực đại và giá trị cực tiểu của hàm f(x1, . . . , xn) được gọi là cực trị của hàm số. Tại M(a1, . . . , an) mà hàm đạt được cực trị gọi là điểm cực trị của hàm số. Định lý 3.1. (Điều kiện cần của cực trị [3]) Nếu hàm z = f(x1, . . . , xn) đạt được cực trị tại M(a1, . . . , an) và tại đây hàm số có các đạo hàm riêng hữu hạn, fxj (a1, . . . , an), j = 1, 2, . . . , n thì các đạo hàm riêng đó phải triệt tiêu fxj (a1, . . . , an) = 0 với mọi j = 1, 2, . . . , n. Định lý 3.2. (xem [3]). Giả sử M(x0, y0) là điểm thỏa mãn zx(x0, y0) = 0, zy(x0, y0) = 0 của hàm z = f(x, y) và tại đây hàm z = f(x, y) có các đạo hàm riêng cấp 2 liên tục 59
  • 60. 60 và ta gọi A = ∂2 z ∂x2 (x0, y0), B = ∂2 z ∂x∂y (x0, y0), C = ∂2 z ∂y2 (x0, y0). 1. Nếu B2 − AC < 0 thì z = f(x, y) có cực trị tại M(x0, y0). Hơn nữa hàm z = f(x, y) đạt cực đại tại M(x0, y0) nếu A < 0, z = f(x, y) đạt cực tiểu tại M(x0, y0) nếu A > 0. 2. Nếu B2 − AC > 0 thì z = f(x, y) không có cực trị tại M(x0, y0). 3. Nếu B2 − AC = 0: chưa kết luận được cực trị của hàm z = f(x, y) tại M(x0, y0). Bài toán 49. Tìm giá trị nhỏ nhất của các hàm số sau z = x3 + y3 − 3xy trong đó 0 ≤ x, y ≤ 2. Chứng minh. Ta thấy    zx = 3x2 − 3y = 0 zy = 3y2 − 3x = 0. Tương đương với    x2 = y y2 = x. Hay (x, y) = (0, 0), (1, 1). Tọa độ các điểm dừng là M1(1, 1), M2(0, 0). Hơn nữa    zxx = 6x zxy = −3 zyy = 6y. Do vậy A1 = 6.1 = 6 > 0, A2 = 0, B1 = −3, B2 = −3, C1 = 6, C2 = 0. Ta có B2 1 − A1C1 = 9 − 36 = −27 < 0 hàm số đạt cực tiểu tại M1(1, 1). Vậy giá trị nhỏ nhất của z là z(M1) = −1. Dễ thấy tại biên của D = {(x, y) : 0 ≤ x, y ≤ 2}, thì z ≥ −1. Vậy giá trị nhỏ nhất của các hàm số z = x3 + y3 − 3xy trong đó 0 ≤ x, y ≤ 2 là −1 đạt được khi x = y = 1.
  • 61. 61 Bài toán 50. Tìm giá trị nhỏ nhất và lớn nhất của hàm số sau z = x3 + 2y3 − 3x − 6y trong đó − 2 ≤ x, y ≤ 2. Chứng minh. Ta thấy    zx = 3x2 − 3 = 0 zy = 6y2 − 6 = 0. Tương đương với    x2 = 1 y2 = 1. Hay (x, y) = (1, 1), (−1, −1), (1, −1), (−1, 1). Tọa độ các điểm dừng là M1(1, 1), M2(−1, −1), M3(−1, 1), M4(1, −1). Hơn nữa    zxx = 6x zxy = −3 zyy = 6y. Do vậy, tại M1 có A1 = 6.1 = 6 > 0, B1 = 0, C1 = 12. Ta có B2 1 − A1C1 = −72 < 0 hàm số đạt cực tiểu tại M1. Tại M2 có A2 = −6 < 0, B2 = 0, C2 = −12. Ta có B2 2 − A2C2 = −72 < 0 hàm số đạt cực đại tại M3. Tại M3 có A3 = −6 < 0, B3 = 0, C3 = 12.
  • 62. 62 Ta có B2 3 − A3C3 = 72 > 0 suy ra M3 không là điểm cực trị. Tại M4 có A4 = 6 > 0, B4 = 0, C4 = −12. Ta có B2 4 − A4C4 = 72 > 0 suy ra M4 không là điểm cực trị. Dễ thấy tại biên của tập D = {(x, y) : −2 ≤ x, y ≤ 2}, tức x hoặc y thuộc {2, −2} thì −6 ≤ z ≤ 6. Vậy giá trị nhỏ nhất và lớn nhất của z = x3 + 2y3 − 3x − 6y lần lượt là là −6 và 6. Bài toán 51. Tìm giá trị lớn nhất và nhỏ nhất của hàm số z = 8x2 + 3y2 + 1 − (2x2 + y2 + 1)2 trong miền tròn đóng D xác định bởi x2 + y2 ≤ 1. Chứng minh. Rõ ràng z liên tục với mọi x, y nên nó đạt giá trị lớn nhất M và giá trị nhỏ nhất m trên miền D. Ta có    zx = 16x − 2(2x2 + y2 + 1)4x = 8x(1 − 2x2 − y2 ) = 0 zy = 6y − 2(2x2 + y2 + 1)2y = 2y(1 − 4x2 − 2y2 ) = 0. Hay (x, y) = (0, 0), (0, 1√ 2 ), (0, −1√ 2 ), ( 1√ 2 , 0), (−1√ 2 , 0). Tọa độ các điểm dừng là O(0, 0), A1(0, 1 √ 2 ), A2(0, −1 √ 2 ), A3( 1 √ 2 , 0), A4( −1 √ 2 , 0), và cả 5 điểm dừng này đều nằm trong miền D. Tính giá trị của z tại các điểm ấy ta được z(O) = 0, z(A1) = z(A2) = 1 4 , z(A3) = z(A4) = 1. Bây giờ, ta xét giá trị của z trên biên của miền D. Trên biên ấy x2 + y2 = 1, vậy y2 = 1 − x2 , do đó z = 8x2 + 3(1 − x2 ) + 1 − (2x2 + 1 = x2 + 1)2 = x2 (1 − x2 ) trong đó −1 ≤ x ≤ 1. Hàm này bằng 0 khi x = 1, −1 và đạt giá trị lớn nhất bằng 1 4 khi x = 1√ 2 , −1√ 2 . Vậy hàm số đạt giá trị nhỏ nhất m = 0 tại gốc O và đạt giá trị lớn nhất M = 1 tại các điểm A3, A4.
  • 63. 63 3.2 Cực trị có điều kiện Xét bài toán: Tìm cực trị của hàm số f(x1, . . . , xn) với điều kiện φj(x1, . . . , xn) = 0, j = 1, . . . , m. Phương pháp làm như sau (xem [3]): Xét hàm Lagrange L(x1, . . . , xn, λ1, . . . , λm) = f(x1, . . . , xn) + m j=1 λjφj(x1, . . . , xn). Giải hệ    Lxj (x1, . . . , xn, λ1, . . . , λm) = 0 ∀j = 1, . . . , n φj(x1, . . . , xn) = 0, j = 1, . . . , m để tìm các điểm dừng. Sau đó xét dấu của dạng vi phân cấp 2 là d2 L để tìm cực trị của hàm số ban đầu. Bài toán 52. Tìm giá trị lớn nhất và nhỏ nhất của hàm số u = x − 2y + 2z với điều kiện x2 + y2 + z2 − 1 = 0. Chứng minh. Rõ ràng u liên tục với mọi x, y, z nên nó đạt giá trị lớn nhất M và giá trị nhỏ nhất m trên miền D. Ta lập hàm Lagrange L(x, y, z, λ) = u = x − 2y + 2z + λ(x2 + y2 + z2 − 1). Xét hệ phương trình    Lx = 1 + 2λx = 0 Ly = −2 + 2λy = 0 Lz = 2 + 2λz = 0 x2 + y2 + z2 − 1 = 0. Hay    x 1 = y −2 = z 2 x2 + y2 + z2 = 1.
  • 64. 64 Từ đây, ta tìm được 2 điểm dừng là M1(1 3 , −2 3 , 2 3 ) ứng với λ = −3 2 và M2(−1 3 , 2 3 , −2 3 ) ứng với λ = 3 2 . Tính d2 L = Lxxdx2 + Lyydy2 + Lzzdz2 + 2Lxydxdy + 2Lyzdydz + 2Lzxdzdx trong đó Lxx = 2λ, Lyy = 2λ, Lzz = 2λ, Lxy = Lyz = Lzx = 0. Do đó d2 L = 2λ(dx2 + dy2 + dz2 ). Từ đó suy ra giá trị lớn nhất và nhỏ nhất của hàm số u = x − 2y + 2z với điều kiện x2 + y2 + z2 − 1 = 0 lần lượt là 1 3 ( đạt được khi (x, y, z) = (1 3 , −2 3 , 2 3 )) và −1 3 ( đạt được khi (x, y, z) = (−1 3 , 2 3 , −2 3 )). Bài toán 53. Tìm giá trị nhỏ nhất của hàm số u = x2 + y2 với điều kiện x + y = 1. Chứng minh. Rõ ràng z liên tục với mọi x, y nên nó đạt giá trị nhỏ nhất m trên miền D. Lập hàm Lagrange L(x, y) = x2 + y2 + λ(x + y − 1). Xét hệ phương trình    Lx = 2x + λ = 0 Ly = 2y + λ = 0 x + y − 1 = 0 ta tìm được điểm dừng là M(1 2 , 1 2 ) với λ = −1. Tính d2 L( 1 2 , 1 2 , −1) = Lxxdx2 + 2Lxydxdy + Lyydy2 |( 1 2 , 1 2 ,−1) trong đó Lxx = 2, Lxy = 0, Lyy = 2. Do đó d2 L( 1 2 , 1 2 , −1) = 2dx2 + 2dy2 > 0. Vậy giá trị nhỏ nhất u = x2 + y2 với điều kiện x + y = 1 là 1 2 .
  • 65. 65 Bài toán 54. Tìm giá trị lớn nhất của hàm số u = sin x sin y sin z với điều kiện x + y + z = π 2 , x, y, z ≥ 0. Chứng minh. Rõ ràng u liên tục với mọi x, y, z nên nó đạt giá trị lớn nhất m trên miền D. Lập hàm Lagrange L(x, y, z, λ) = ln sin x + ln sin y + ln sin z + λ(x + y + z − π 2 ). Ta thấy    Lx = cot x + λ = 0 Ly = cot y + λ = 0 Lz = cot z + λ = 0 x + y + z − π 2 = 0 ta tìm được điểm dừng là M(π 6 , π 6 , π 6 ) với λ = − √ 3 2 . Tính d2 L = −( dx2 sin2 x + dy2 sin2 y + dz2 sin2 z ) < 0. Do đó tại điểm (π 6 , π 6 , π 6 ) hàm số đạt cực đại có điều kiện là 1 8 . Vậy giá trị lớn nhất u = sin x sin y sin z với điều kiện x + y + z = π 2 , x, y, z ≥ 0 là 1 8 . Bài toán 55. Tìm giá trị lớn nhất của hàm số u = xyz với điều kiện x2 + y2 + z2 = 1, x + y + z = 0. Chứng minh. Rõ ràng u liên tục với mọi x, y, z nên nó đạt giá trị lớn nhất m trên miền D. Lập hàm Lagrange L(x, y, z, λ) = xyz − λ1(x2 + y2 + z2 − 1) − λ2(x + y + z). Ta thấy    Lx = yz − 2λ1x − λ2 = 0 Ly = xz − 2λ1y − λ2 = 0 Lz = xy − 2λ1z − λ2 = 0 x2 + y2 + z2 = 1 x + y + z = 0
  • 66. 66 ta tìm được các điểm dừng là M1( 1 √ 6 , 1 √ 6 , − 2 √ 6 ), M2( 1 √ 6 , − 2 √ 6 , 1 √ 6 ), M3(− 2 √ 6 , 1 √ 6 , 1 √ 6 ) với λ2 = − 1 2 √ 6 , M4(− 1 √ 6 , − 1 √ 6 , 2 √ 6 ), M5(− 1 √ 6 , 2 √ 6 , − 1 √ 6 ), M6( 2 √ 6 , − 1 √ 6 , − 1 √ 6 ) với λ2 = 1 2 √ 6 . Tiếp tục tìm vi phân bậc 2 của hàm Lagrange d2 L = −2λ1(dx2 + dy2 + dz2 ) + 2zdxdy + 2ydxdz + 2xdydz trong đó dx, dy, dz liên hệ với nhau bởi 2 hệ thức xdx + ydy + zdz = 0, dx + dy + dz = 0. Tại các điểm M1, M4 thì x = y = −2λ1, z = 4λ1. Khi đó xdx + ydy + zdz = −2λ1dx − 2λ1dy + 4λ1dz = 0 hay dz = 1 2 (dx + dy). Thay vào biểu thức của d2 L tại M1 ta có d2 L(M1) = 1 √ 6 (dx2 + dy2 + dz2 ) + 1 √ 6 (dx − dy)2 > 0. Vậy u(M1) = umin = − 1 3 √ 6 , d2 L(M4) = − 1 √ 6 (dx2 + dy2 + dz2 ) − 1 √ 6 (dx − dy)2 < 0. Vậy u(M4) = umax = 1 3 √ 6 . Tương tự u(M5) = u(M6) = umax = 1 3 √ 6 , u(M2) = u(M3) = umin = − 1 3 √ 6 . Vậy giá trị nhỏ nhất và lớn nhất của u = xyz với điều kiện x2 +y2 +z2 = 1, x+y+z = 0 lần lượt là − 1 3 √ 6 và 1 3 √ 6 .
  • 67. 67 Bài toán 56. Tìm giá trị lớn nhất của hàm số u = xy + yz với điều kiện x2 + y2 = 2, y + z = 2, x, y, z ≥ 0. Chứng minh. Rõ ràng u liên tục với mọi x, y, z nên nó đạt giá trị lớn nhất m trên miền D. Lập hàm Lagrange L(x, y, z, λ1, λ2) = xy + yz + λ1(x2 + y2 − 2) − λ2(y + z − 2). Ta thấy    Lx = y + 2λ1x = 0 Ly = x + z + 2λ1y + λ2 = 0 Lz = y + λ2 = 0 x2 + y2 = 2 y + z = 2 ta tìm được điểm dừng là M(1, 1, 1) với λ1 = −1 2 , λ2 = −1. Tiếp tục tìm vi phân bậc 2 của hàm Lagrange d2 L = 2λ1(dx2 + dy2 ) + 2dxdy + 2dydz và thay λ1 = −1 2 ta nhận được d2 L(1, 1, 1, − 1 2 ) = −(dx2 + dy2 ) + 2dxdy + 2dydz. Từ phương trình y + z = 2 ta suy ra dy = −dz và từ 2xdx + 2ydy = 0 với x = y = 1 ta có dx = −dy. Vậy nên d2 L(1, 1, 1, − 1 2 ) = −(dx2 + dy2 ) − 2dy2 − 2dz2 = −dx2 − 3dy2 − 2dz2 < 0. Vậy u(1, 1, 1) = umax = 2. Vậy giá trị lớn nhất của u = xy + yz với điều kiện x2 + y2 = 2, y + z = 2, x, y, z ≥ 0 là 2.
  • 68. Kết luận Luận văn đề cập tới nghiên cứu một số phương pháp đạo hàm để tìm giá trị lớn nhất và giá trị nhỏ nhất của hàm số với ứng dụng vào giải quyết những bài toán khác nhau. Luận văn đã trình bày các vấn đề sau: - Phương pháp khảo sát trực tiếp hàm số trên miền xác định - Phương pháp khảo sát hàm số theo từng biến - Phương pháp đặt biến phụ - Phương pháp đánh giá thông qua biểu thức bậc nhất - Phương pháp sử dụng tính chất của hàm lồi, hàm lõm - Cực trị tự do của hàm nhiều biến, và cực trị có điều kiện của hàm nhiều biến. 68
  • 69. Tài liệu tham khảo [1] Phạm Văn Dũng, Phương pháp sử dụng đạo hàm chứng minh bất đẳng thức. [2] Phạm Kim Hùng, Sáng tạo bất đẳng thức, NXB tri thức, 2006. [3] Trần Đức Long, Nguyễn Đình Sang, Hoàng Quốc Toàn, Giáo trình giải tích 1, NXB ĐHQG Hà Nội, 2004. [4] Nguyễn Văn Mậu, Bất đẳng thức: Định lý và áp dụng, NXB Giáo dục, 2006. [5] Trần Phương, Vẻ đẹp Bất đẳng thức trong các kì thi Olympic Toán học, NXB ĐHQG Hà Nội, 2010. [6] Trần Phương, Những viên kim cương trong bất đẳng thức Toán học, 2009. [7] Nguyễn Minh Tuấn, Lý thuyết cơ sở của hàm lồi và các bất đẳng thức cổ điển, NXB ĐHQG Hà Nội, 2013. 69